You are on page 1of 57

Constitutional Law Review

12. BILL OF RIGHTS (Article III)


Cases

Section 1.
Ang Tibay vs CIR
The SC also outlined that administrative bodies like the CIR,
although not strictly bound by the Rules of Court must also make
sure that they comply to the requirements of due process. For
administrative bodies, due process can be complied with by
observing the following:
(1) The right to a hearing which includes the right of the party interested
or affected to present his own case and submit evidence in support
thereof.
(2) Not only must the party be given an opportunity to present his case
and to adduce evidence tending to establish the rights which he
asserts but the tribunal must consider the evidence presented.
(3) While the duty to deliberate does not impose the obligation to
decide right, it does imply a necessity which cannot be disregarded,
namely, that of having something to support its decision. A decision
with absolutely nothing to support it is a nullity, a place when
directly attached.
(4) Not only must there be some evidence to support a finding or
conclusion but the evidence must be substantial. Substantial
evidence is more than a mere scintilla It means such relevant
evidence as a reasonable mind might accept as adequate to
support a conclusion.
(5) The decision must be rendered on the evidence presented at the
hearing, or at least contained in the record and disclosed to the
parties affected.
(6) The administrative body or any of its judges, therefore, must act on
its or his own independent consideration of the law and facts of the
controversy, and not simply accept the views of a subordinate in
arriving at a decision.
(7) The administrative body should, in all controversial questions, render
its decision in such a manner that the parties to the proceeding can
know the various issues involved, and the reasons for the decisions
rendered. The performance of this duty is inseparable from the
authority conferred upon it.

People vs Cayat

(1) must rest on substantial distinctions;


(2) must be germane to the purposes of the law;
(3) must not be limited to existing conditions only; and
(4) must apply equally to all members of the same class.
Act No. 1639 satisfies these requirements. The classification rests
on real or substantial, not merely imaginary or whimsical,
distinctions. It is not based upon accident of birth or parentage.
The law, then, does not seek to mark the non-Christian tribes as an
inferior or less capable race. On the contrary, all measures thus far
adopted in the promotion of the public policy towards them rest
upon a recognition of their inherent right to equality in the
enjoyment of those privileges now enjoyed by their Christian
brothers. But as there can be no true equality before the law, if
there is, in fact, no equality in education, the government has
endeavored, by appropriate measures, to raise their culture and
civilization and secure for them the benefits of their progress, with
the ultimate end in view of placing them with their Christian
brothers on the basis of true equality.

Villegas vs Hiu Chiong


ISSUE: Whether or not there is undue delegation to the Mayor of
Manila.
HELD: The decision of Judge Arca is affirmed. Ordinance No. 6537
does not lay down any criterion or standard to guide the Mayor in
the exercise of his discretion. It has been held that where an
ordinance of a municipality fails to state any policy or to set up any
standard to guide or limit the mayors action, expresses no purpose
to be attained by requiring a permit, enumerates no conditions for
its grant or refusal, and entirely lacks standard, thus conferring
upon the Mayor arbitrary and unrestricted power to grant or deny
the issuance of building permits, such ordinance is invalid, being an
undefined and unlimited delegation of power to allow or prevent an
activity per se lawful. Ordinance No. 6537 is void because it does
not contain or suggest any standard or criterion to guide the mayor
in the exercise of the power which has been granted to him by the
ordinance. The ordinance in question violates the due process of
law and equal protection rule of the Constitution.

Kwong Sing vs City of Manila


WON the enforcement of Ord.No.532 is an act beyond the scope of
police power?
WON the enforcement of the same is a class legislation that
infringes property rights

Reasonable restraints of a lawful business for such


purposes are permissible under the police power. The
police power of the City of Manila to enact Ordinance
no.532 is based on Sec. 2444, par.(I) and (ee) of the
Admin. Code as amended by Act. 2744, authorizes the
municipality board of the City of Manila, with the approval
of the mayor of the city:

(I) To regulate and fix the amount of the license fees for
the following: xxxx xxxx laundries xxxx.

ISSUE: Whether or not the said Act violates the equal protection
clause.
HELD: No. The SC ruled that Act 1639 is valid for it met the
requisites of a reasonable classification. The
SC emphasized that it is not enough that the members of a group
have the characteristics that distinguish them from others. The
classification must, as an indispensable requisite, not be arbitrary.
The requisites to be complied with are;

(ee) to enact all ordinances it may deem necessary and


proper for the sanitation and safety, the furtherance of the
prosperity, and the promotion of the morality, peace, good
order, comfort, convenience, and general welfare of the
city and its inhabitants.

The court held that the obvious of the ordinance no. 532 was to
avoid disputes between laundrymen and their patrons and to
protect customers of laundries who are not able to decipher
Chinese characters from being defrauded.

In whether the ordinance is class legislation, the court held


that the ordinance invades no fundamental right, and impairs no
personal privilege. Under the guise of police regulation, an attempt
is not made to violate personal property rights. The ordinance is
neither discriminatory nor unreasonable in its operation. It applies
to all public laundries without distinction, whether they belong to
Americans, Filipinos, Chinese, or any other nationality. All, without
exception, and each every of them without distinction, must comply
with the ordinance. The obvious objection for the implementation of
the ordinance based in sec. 2444 (ee) of the Administrative Code.
Although, an additional burden will be imposed on the business and
occupation affected by the ordinance such as that of the appellant
by learning even a few words in Spanish or English, but mostly
Arabic numbers in order to properly issue a receipt, it seems that
the same burdens are cast upon them. Yet, even if private rights of
person or property are subjected to restraint, and even if loss will
result to individuals from the enforcement of the ordinance, this is
not sufficient ground for failing to uphold the power of the
legislative body. The very foundation of the police power is the
control of private interests for the public welfare.

Held: There was no specific offense mentioned. There is a need to


mention only one specific offense. Otherwise, it will be considered a
general warrant.
EXCLUSIONARY RULE
STONEHILL VS. DIOKNO
The court issued several warrants, 42 in all, to search the
houses and business addresses of Stonehill and of the members of
his board. Stonehill was known to engage in various illegal
activities. Unfortunately, all the warrants were declared null and
void for the reason that they were general warrants. The warrant
stated that the objects to seize were to be used as evidence for
violations of the Tariff Laws, Internal Revenue Laws, Central Bank
Laws, RPC, and etc.
Held: The court ruled that evidences obtained from Stonehills
residences cannot be held admissible in any proceeding against
him. The court however, declared that evidences obtained from his
offices can be used against him. The reason for this is section 2 can
only be invoked by the person whose rights have been invaded.
Evidences obtained from the corporation, being a different person
and entity from Stonehill, can be held admissible in court.

Bache Co. vs. Ruiz L-32409


February 27 1971
Facts:

International School Alliance


vs Quisumbing
HELD: There is violation of equal protection. Equal pay for equal
work, persons who work with substantially equal qualifications,
skillsm effort, and responsibility under similar conditions should be
paid similar salaries. If an employer accords the same rank and
position, the presumption is that they perform equal work. Here,
both groups have similar functions which they perform under
similar conditions. There is no evidence that foreign hires perform
25% more efficient than local hires. The dislocation factor and
tenure are properly accorded by the benefits they received.

Commissioner of Internal Revenue, wrote a letter addressed to


respondent Judge Vivencio M. Ruiz requesting the issuance of a
search warrant against petitioners for violation of Section 46(a) of
the National Internal Revenue Code. Revenue Examiner Rodolfo de
Leon and Arturo Logronio went to CFI with proper documents. Judge
Vivencio Ruiz asked his secretary to take the deposition and when
done stenographer read it to the judge. Logronio took the oath ans
was warned by judge that he may be charged with perjury if found
lying. Search warrant was issued and served. Petitioners lawyers
protested the search on the ground that no formal complaint or
transcript of testimony was attached to the warrant. The agents
nevertheless proceeded with their search which yielded six boxes of
documents. BIR based on the documents seized. Petitioner contend
that judged failed to personally examine the complainant and
witnesses.
Issue: Whether or not search warrant is null and void on the ground
of no personal examination of the jusge?

SECTION 2
STONEHILL VS. DIOKNO
(discussed by Ms. Jumao-as General Warrant)
Note: This case was also in the exclusionary rule.
Stated in the warrant: The above items are subject to the offense,
stolen or embezzled, or intended to be used as a means to commit
offenses violating CB laws, tariffs and customs laws, The Internal
Revenue Code, and the RPC.

Decision:
This cannot be consider a personal examination. If there was an
examination at all of the complainant and his witness, it was the
one conducted by the Deputy Clerk of Court. But, as stated, the
Constitution and the rules require a personal examination by the
judge. It was precisely on account of the intention of the delegates
to the Constitutional Convention to make it a duty of the issuing
judge to personally examine the complainant and his witnesses that
the question of how much time would be consumed by the judge in
examining them came up before the Convention, as can be seen
from the record of the proceedings quoted above. The reading of
the stenographic notes to respondent Judge did not constitute
sufficient compliance with the constitutional mandate and the rule;
for by that manner respondent Judge did not have the opportunity
to observe the demeanor of the complainant and his witness, and

to propound initial and follow-up questions which the judicial mind,


on account of its training, was in the best position to conceive.
These were important in arriving at a sound inference on the allimportant question of whether or not there was probable cause.

Roan vs.
71410

Gonzales

Gr

Held: The court said that the judge need not personally conduct
the investigation because it will duly burdensome on the part of the
judges if they will be required to do so. They will not have enough
time to conduct the trial, which is their main responsibility.

no. LIM VS. FELIX

In Roan vs. Gonzales, G.R. No. 71410, November 25, 1986, the
Court declared the deposition-taking so defective that it rendered
the assailed search warrant invalid. In this case, the depositions
taken showed that they were mainly a restatement of the
witnesses allegations in their affidavit, except that they were made
in the form of answers to the questions put to them by the
respondent judge. Significantly, the meaningful remark made by
one of the witnesses that they were suspicious of the petitioner
because he was a follower of the opposition candidate in the
forthcoming election did not excite the respondent judges
suspicions. The respondent judge almost unquestioningly received
the witnesses statement that they saw eight men deliver arms to
the petitioner in his house. This was supposedly done overtly, and
the witness said he saw everything through an open window of the
house while he was near the gate. He could even positively say that
six of the weapons were .45 caliber pistols and two were .38 caliber
revolvers. The Court wondered why it did not occur to the
respondent judge to ask how the witness could be so certain even
as to the caliber of the guns, or how far he was from the window, or
whether it was on the first floor or a second floor, or why his
presence was not noticed at all, or if the acts related were really
done openly, in the full view of the witnesses, considering that
these acts were against the law. These would have been judicious
questions but they were injudiciously omitted. Instead, the
declarations of the witnesses were readily accepted and the search
warrant sought was issued forthwith.
Warrantless Searches
There are recognized exceptions where a search may be validly
made without warrant and articles may be taken validly as a result
of that search. These include a warrantless search made
incidental to a lawful arrest, as when the person being arrested is
frisked for weapons he may otherwise be able to use against the
arresting officer. Motor cars may be inspected at borders to prevent
smuggling of aliens and contraband and even in the interior upon a
showing of probable cause. Vessels and aircraft are also
traditionally removed from the operation of the rule because of
their mobility and their relative ease in fleeing the state's
jurisdiction. The individual may knowingly agree to be searched or
waive objections to an illegal search. And it has also been held that
prohibited articles may be taken without warrant if they are open to
eye and hand and the peace officer comes upon them
inadvertently. ROAN vs. GONZALES, et al. [G.R. No. 71410.
November 25, 1986.]

SOLIVEN VS. MAKASIAR


This is the famous libel case filed by former Pres. Aquino against
Luis Beltran. Beltran questioned the issuance of the warrant. He
said the judge did not personally examine the witnesses, which is
unconstitutional. In an arrest warrant, the judge may rely on the
evidences. He need not call the witnesses.

The killing of Congressman Espinosa happened in Masbate.


The fiscal conducted a preliminary investigation to find out if there
was probable cause to hold the accused for trial. And he found out
that there was. During the pendency of the case before the RTC of
Masbate, the SC approved a petition for a change in venue to
Makati. The judge, on the certificate alone, which was issued by the
prosecutor, determined that there was probable cause. The accused
challenged the validity of the warrant. He argued that the judge
should make a personal determination as to the existence of
probable cause to issue the warrant.
ISSUE: Whether the judge can issue a warrant based on the
certificate alone. Can it be considered valid?
Held: The court ruled that the warrant is void. Before the fiscal can
conclude that there is probable cause, he should first examine all
the evidences, documents presented to him, and all of those should
be attached on the certificate stating that there is sufficient ground
that the accused should be put on trial. In this case, there was
nothing attached to the certificate stating that there is sufficient
ground that the accused should be put on trial. In this case, there
was nothing attached to the certificate. The Constitution
underscores that the judge should determine if there is probable
cause basing on all the facts and evidences, and not on the
certificate alone.
In the case of arrest warrant, the judge need not call the
witnesses for questioning. He may rely on the record submitted to
him by the fiscal. But he must not rely solely on the resolution or
certification of the fiscal. The resolution must be supported by other
evidence which led to the filing of the case. In the case of a search
warrant, the judge must personally examine the witnesses himself
to determine probable cause.

PANGANDAMAN VS. CESAR


There was a shooting incident which killed 5 people. After the
incident and after preliminary investigation, the Lanao judge issued
a warrant for the arrest of 14 individuals and 50 John Does. Not one
of the 50 can be identified by the witnesses, but it seems that the
number was about the number of people who participated in the
shooting.
ISSUE: Is a John Doe warrant valid?
Held: According to the court, a John Doe warrant is void. The
Constitution requires that there must be specific description or
designation of the person to be arrested. John Does warrants are
general warrants.

ANIAG VS. COMELEC

COMELEC issued resolutions for implementation of the gun ban


during the election period in preparation for the coming elections as
well as the authority for the COMELEC to establish checkpoints.
Consequently, the Congress sergeant-at-arms ordered the return of
2 firearms issued to Aniag. In compliance, Aniag ordered his driver
to pick up the guns at his residence and deliver the same to the
sergeant-at-arms. PNP established a checkpoint 30 meters from
the Batasan Complex, so when the car passed by, it was flagged
down. The car was inspected and extensively searched. The trunk
was opened and a bag was found containing guns neatly packed in
their cases. The driver was apprehended and a case was filed for
violation of the resolution and PD 1866.
Held: In searches made of moving vehicle, extensive search can
only be done if there is existence of probable cause. In this case,
the SC ruled that there was no probable cause to conduct a more
extensive search. The driver was not acting suspiciously. He did
not become fidgety. The guns were even hidden. They could not
be seen by mere visual search. They have to open the trunk and
the bag for the guns to be apparent

People vs. Tabar


I.. . . IN CONVICTING ACCUSED-APPELLANTS (sic) OF VIOLATION OF
SECTION 4, ARTICLE II, REPUBLIC ACT 6425 AS AMENDED WHEN
THE EVIDENCE DOES NOT WARRANT IT.
II.. . . IN ADMITTING EVIDENCE SEIZED WITHOUT ANY SEARCH
WARRANT. 14
As to the first assigned error, the appellant claims that the
prosecution presented no evidence that she sold marijuana and
since there exists no convincing, positive and conclusive proof of
conspiracy between her and her co-accused, Rommel Arriesgado,
she cannot be held liable for violation of Section 4, Article II of R.A.
No. 6425, as amended.
In support of the second assigned error, the appellant maintains
that the marijuana cigarettes seized from her are inadmissible in
evidence because they were obtained in violation of the
constitutional guarantee against unreasonable search and seizure.
After a careful perusal of the records and evaluation of the
evidence, this Court is inclined to agree with the appellant that she
should not be convicted under Section 4, Article II of R.A. No, 6425.
We rule, however, that she is liable under Section 8, Article II of the
said Act. Her conviction by the trial court under Section 4 is
primarily based on its conclusion that the appellant "actually
employed her nephew Rommel Arriesgado to sell marijuana from
her store and that she had been in that illicit business for quite
sometime now." This conclusion is based on the trial court's
sweeping statement that "talks in the community where the
accused lives is rife with accusations that she is indeed engaged
with members of the family, in the sale and distribution of
prohibited drugs such as marijuana." We find no evidence on record
to sustain this charge. It may thus be said that such a conclusion is
not based on established facts but on "talks in the community." If
indeed such was the fact, it would not have been difficult for the
prosecution to provide the court with overwhelming evidence. Yet, it
presented only Pat. Trangia who, rather unfortunately, did not even
testify or volunteer information that the main target of the busybust operation was the appellant. He did not also disclose in his
testimony that the appellant was among the reported "pushers" in
Punta Princesa, Cebu City. It may be recalled that the buy-bust
operation on 8 February 1989 was conducted because, as he
alleged:

A.
Before that time we have already received information from
the community of Punta Princesa regarding marijuana pushers in
that place. 15
Nevertheless, the prosecutor who conducted the direct-examination
of Trangia did not ask further as to identity of the pushers such that
it was not proven that the appellant was one of them. The Solicitor
General, however, maintains that there was conspiracy, established
by circumstancial evidence, between accused Rommel Arriesgado
who was caught in flagrante selling three (3) sticks of handrolled
marijuana to the informant and accepting the marked money. We
are not persuaded since the evidence for the prosecution does not
show that (a) the appellant was in the mind of the members of the
team when they planned the buy-bust operation and when they
carried out such plan, (b) the three (3) sticks of handrolled
marijuana came from the appellant, and (c) the appellant used
Rommel as her agent to sell the three (3) sticks to the informant.
Moreover, if indeed the prosecution truly believed that such
conspiracy existed, it should not have willingly given its conformity
to Rommel's plea to the lesser offense of illegal possession of
prohibited drugs under Section 8, Article II of R.A. No. 6425, as
amended. Having been caught in flagrante for selling marijuana, it
was not difficult to prove Rommel's culpability under Section 4,
Article II of the Act. Yet it readily consented to his offer to plead
guilty to the said lesser offense.
It was, however, established beyond any shadow of doubt and,
therefore, with moral certainty, that the appellant kept in her
possession handrolled sticks of marijuana placed in empty Hope,
Philip Morris and Mark cigarrette packs. 16 She does not have any
authority to possess them. She may have acquired them with the
intention to sell them for profit; but without proof of sale, she
cannot be held liable under Section 4, Article II of the Dangerous
Drugs Act. For such possession, her liability is covered by Section 8
of the said Article which penalizes possession or use of prohibited
drugs. The last paragraph thereof reads:
xxx

xxx

xxx

The penalty of the imprisonment ranging from six years and one
day to twelve years and a fine ranging from six thousand to twelve
thousand pesos shall be imposed upon any person who, unless
authorized by law, shall possess or use of Indian hemp. Indian
hemp is otherwise known as Marijuana. 17
Appellant, therefore, may specifically be penalized under the
aforesaid last paragraph of Section 8,
Article II of the Act. Applying the Indeterminate Sentence Law, 18
the penalty of eight (8) years as Minimum to twelve (12) years as
Maximum and a fine of P10,000.00 may then be imposed upon her.
The second assigned error is without merit. The evidence for the
prosecution discloses that the appellant placed the packs of
marijuana sticks under the rolled pair of pants which she was then
carrying at the time she hurriedly left her shanty after noticing the
arrest of Rommel. When she was asked to spread it out, which she
voluntary did, the package containing the packs of marijuana sticks
were thus exposed in plain view to the member of the team. A
crime was thus committed in the presence of the policemen.
Pursuant to Section 5, Rule 113 and Section 12 Rule 126 of the
Revised Rules of Court, she could lawfully be arrested and searched
for anything which may be used as proof of the commission of an
offense without the corresponding arrest and search warrants. Her
own counsel on crossexamination of prosecution witness Josephus
Trangia further obtained a affirmation of these facts, thus:

Even assuming ex gratia argumenti that the seach and seizure were
without a warrant, the appellant had effectively waived her
constitutional right relative thereto by voluntarily submitting to the
seach and seizure. In People vs. Malasugui, 20 this Court ruled:

Held: The search was not incident to a lawful arrest because the
marijuana was not obtained in the person of the accused nor in the
place within his immediate control. It would be valid if Musa were in
the kitchen when the bag was found.

When one voluntarily submits to a search and consent to have it


made of his person or premises, he is precluded from later
complaining thereof (Cooley, Constitutional Limitations, 8th ed., vol.
I, pages 631). The right to be secure from unreasonable seach may,
like every right, be waived and such waiver may be made either
expressly or impliedly.

PEOPLE VS. MUSA

The exclusionary rule relied upon by the appellant does not provide
her safe refuge.
Before We close this case, a final observation for the guidance of
trial judges must be made.
For the violation of Section 4, Article II of R. A. No. 6425, as
amended, the trial court imposed the penalty of reclusion perpetua.
The penalty provided for therein is "life imprisonment to death and
a fine ranging from twenty thousand to thirty thousand pesos." In
view of Section 19(1), Article III of the 1987 Constitution which
prohibits the imposition of the death penalty, the maximum penalty
then imposable thereunder would only be life imprisonment. Life
imprisonment, however, is not synonymous with reclusion
perpetua. We have reiterated this time and again 21 and
admonished judges to employ the proper legal terminology in the
imposition of imprisonment penalties because of their different
accompanying legal accessories and effects. 22
IN THE LIGHT OF THE FOREGOING, judgment is hereby rendered
modifying the challenged Decision of Branch 15 of the Regional Trial
Court of Cebu in Criminal Case No. CBU-14863 dated 17 December
1990 and, as modified, finding appellant CARMELINA TABAR y
CARMILOTES guilty beyond reasonable doubt of illegal possession of
marijuana under Section 8, Article II of R. A. No. 6425, otherwise
known as the Dangerous Drugs Act of 1972, as amended, and,
applying the Inderterminate Sentence Law, she is sentenced to
suffer imprisonment of eight (8) years as minimum to twelve (12)
years as maximum and to pay a fine of Ten Thousand Pesos
(P10,000.00).
Costs against the appellant.
SO ORDERED.

This was a buy-bust operation. Information was obtained that a


certain Musa was selling marijuana. A test-buy was conducted and
true enough marijuana was brought the following day. A buybust
was arranged. Sgt Ani posed as a prospective buyer. After Musa
gave him marijuana and after getting the marked P20.00 bill, Musa
was arrested. In the living room, they could not find the P20.00 bill.
Hence, they proceeded to the other parts of the house and found in
the kitchen a plastic bag which when opened contained dried
marijuana leaves.

Held:
No valid search. Although it was a valid arrest, the
subsequent search in the kitchen was not. It cannot be considered
as a surrounding within his immediate control. The prosecutions
contention that it was evidence in plain view is untenable because
the evidence of illegality is not apparent. They have to open the
plastic bag to know what was contained therein. The marijuana
there in the plastic bag is inadmissible in evidence.
PEOPLE VS. MUSA
(as discussed by Ms. Jumao-as under plain view)
- (People vs. Musa, supra)
Held: The court ruled that this was not a search in plain view. The
police did not come across the evidences inadvertently. There was
prior justification for the intrusion but it is limited only within the
immediate vicinity. Also, the illegality was not readily apparent
since it was inside a plastic bag.

People vs. Valdez


FACTS: The accused was arrested for bank robbery. After four days,
the police investigator took down his extrajudicial confession and
called a lawyer who conferred with the accused for ten minutes and
executed his confession.
HELD: The confession is inadmissible. The moment the accused
was arrested and detained, he was already under custodial
investigation. The lawyer was called only on the 4th day of
detention when the accused was about to put down his confession
in writing.

PEOPLE VS. MUSA


The police conducted a buy-bust operation, an instance where in
a warrantless arrest is considered as legal (Selling marijuana is a
valid ground for a policeman to make an arrest, even without a
warrant). The incident took place in the sala of the accused. The
policeman used marked money during the operation. While they
were looking for the money, they found a plastic bag in the kitchen
which after opening, contained marijuana.
ISSUE: Can the arresting officers raise the issue that this was
incident to a lawful arrest? Since the accused was a dealer of
marijuana. Therefore the marijuana found should be admissible as
evidence.

People
188691

vs.

Mariacos

PEOPLE OF THE PHILIPPINES vs BELEN MARIACOS


GR NO. 188611 June 16 2010
FACTS:

gr.

October 27, 2005 in Brgy Balbalayang, PO2 Pallayoc met with secret
agent of the Barangay Intelligence Network who informed him that
a baggage of marijuana had been loaded in a passenger jeepney
that was about to leave for the poblacion. The agent mentioned 3
bags and 1 plastic bag. Further, the agent described a backpack
bag with O.K. marking. PO2 Pallayoc boarded the said jeepney and
positioned himself on top thereof. He found bricks of marijuana
wrapped in newspapers. He them asked the other passengers about
the owner of the bag, but no one know.

outset of the search. Thus, given the factual milieu of the case,
there is a need to determine whether the police officers had
probable cause to arrest appellant. Although probable cause eludes
exact and concrete definition, it ordinarily signifies a reasonable
ground of suspicion supported by circumstances sufficiently strong
in themselves to warrant a cautious man to believe that the person
accused is guilty of the offense of which he is charged. People vs.
Jack Racho y Raquero, G.R. No.
186529, August 3, 2010.

When the jeepney reached the poblacion, PO2 Pallayoc alighted


together with other passengers. Unfortunately, he did not noticed
who took the black backpack from atop the jeepney. He only
realized a few moments later that the said bag and 3 other bags
were already being carried away by two (2) women. He caught up
with the women and introduced himself as a policeman. He told
them that they were under arrest, but on the women got away.

PEOPLE V. COMPACION

DOCTRINES:
ARTICLE III, SECTION 2 OF THE PHILIPPINE CONSTITUTION
PROVIDES: The right of the People to be secure in their persons,
houses, papers, and effects against unreasonable searches and
seizures of whatever nature and for any purpose shall be inviolable,
and no search warrant or warrant of arrest shall issue except upon
probable cause to be determined personally by the Judge after
examination under oath or affirmation of the complainant and the
witnesses he may produce, and particularly describing the place to
be searched and the persons or things to be seized.
Purpose: MOVING VEHICLE (WARRANTLESS SEARCH)
This has been justified on the ground that the mobility of motor
vehicles makes it possible for the vehicle to be searched to move
out of the locality or jurisdiction in which the warrant must be
sought.
This is no way, however, gives the police officers unlimited
discretion to conduct warrantless searches of automobiles in the
absence of probable cause when a vehicle is stopped and subjected
to an extension search, such a warrantless search has been held to
be valid only as long as officers conducting the search have
reasonable or probable cause to believe before the search that they
will find the instrumentality or evidence pertaining to a crime, in the
vehicle to be searched.
MALUM PROHIBITUM
When an accused is charged with illegal possession or
transportation of prohibited drugs, the ownership thereof is
immaterial. Consequently, proof of ownership of the confiscated
marijuana is not necessary.
Appellants alleged lack of knowledge does not constitute a valid
defence. Lack of criminal intent and good faith are not exempting
circumstances where the crime charge is malum prohibitum

People vs. Jack Racho

Gr no. 186529

aug. 3 2010
Arrest; probable cause. Recent jurisprudence holds that in
searches incident to a lawful arrest, the arrest must precede the
search; generally, the process cannot be reversed. Nevertheless, a
search substantially contemporaneous with an arrest can precede
the arrest if the police has probable cause to make the arrest at the

Based on a tip supplied by an informant, the police


conducted a surveillance of the residence and saw 2 tall marijuana
plants in the backyard of accused in San Carlos City. On July 12,
1995, the police applied for a warrant in Bacolod City but the judge
told them that he had no territorial jurisdiction over the matter.
They went to San Carlos City but the judge there told them to go
back the following day, it was already night-time. At about 1:30 in
the morning of the following day, without a search warrant, the
police arrived at the house of accused who let them into the gate
without objection. The police found the marijuana. Was there a
waiver of the right against warrantless searches?

Held: No. While the right against unreasonable searches and


seizures may be waived expressly or impliedly, such waiver must
be made voluntarily, knowingly and intelligently. The acts of the
accused in allowing the police to enter the premises and his
consequent silence during the search should not be construed as
voluntary submission or and implied acquiescence especially so
when members of the raiding team were numerous. His implied
consent, if any, could not have been more than mere passive
conformity given under coercive circumstances, and is, thus, no
consent at all.
Consequently, his lack of objection is not
tantamount to waiver of a constitutional right.

People vs Solayao

gr no 119220 sep 20

1996
The issue of whether or not an admission in criminal cases is
adequate to prove beyond reasonable doubt the commission of the
crime charged has been settled in the case of People vs. Solayao 4
G.R. No. 119220, September 20, 1996, 262 SCRA 255.where this
Court made the following pronouncements:
". . . By its very nature, an "admission is the mere acknowledgment
of a fact or of circumstances from which guilt may be inferred,
tending to incriminate the speaker, but not sufficient of itself to
establish his guilt." In other words it is a "statement by defendant of
fact or facts pertinent to issues pending, in connection with proof of
other facts or circumstances, to prove guilt, but which is, of itself,
insufficient to authorize conviction." From the above principles, this
Court can infer that an admission in criminal cases is insufficient to
prove beyond reasonable doubt the commission of the crime
charged. 5 Ibid. p. 264

Malacat vs CA gr no 123595 dec

12 1997

Facts:
On August 29, 1990 at about 6:30 in the evening, allegedly in
response to bomb threats reported seven days earlier, Rodolfo Yu of
the Western Police District, Metropolitan Police Force of the
Integrated National Police, Police Station No. 3, Quiapo, Manila, was
on foot patrol with three other police officers
(all of them in uniform) along Quezon Boulevard, Quiapo, Manila,
near the Mercury Drug store at Plaza Miranda. They chanced upon
two groups of Muslim-looking men, with each group, comprised of
three to four men, posted at opposite sides of the corner of stop
and frisk, where a warrant and seizure can be effected without
necessarily being preceded by an arrest and whose object is
either to maintain the status quo momentarily while the police
officer seeks to obtain more information; and that the seizure of
the grenade from Malacat was incidental to a lawful arrest. The trial
court thus found Malacat guilty of the crime of illegal possession of
explosives under Section 3 of PD 1866, and sentenced him to suffer
the penalty of not less than 17 years, 4 months and 1 day of
Reclusion Temporal, as minimum, and not more than 30 years of
Reclusion Perpetua, as maximum. On 18 February 1994, Malacat
filed a notice of appeal indicating that he was appealing to the
Supreme Court. However, the record of the case was forwarded to
the Court of Appeals (CA-GR CR 15988). In its decision of 24 January
1996, the Court of Appeals affirmed the trial court. Manalili filed a
petition for review with the Supreme Court.
Issue:
Whether the search made on Malacat is valid, pursuant to the
exception of stop and frisk.
Ruling:
The trial court ruled that the warrantless search and seizure of
petitioner was akin to a stop and frisk, where a warrant and
seizure can be effected without necessarily being preceded by an
arrest and whose object is either to maintain the status quo
momentarily while the police officer seeks to obtain more
information. Probable cause was not required as it was not certain
that a crime had been committed, however, the situation called for
an investigation, hence to require probable cause would have been
premature. The RTC emphasized that Yu and his companions were
confronted with an emergency, in which the delay necessary to
obtain a warrant, threatens the destruction of evidence and the
officers had to act in haste, as petitioner and his companions
were acting suspiciously, considering the time, place and reported
cases of bombing. Further, petitioners group suddenly ran away
in different directions as they saw the arresting officers approach,
thus it is reasonable for an officer to conduct a limited search, the
purpose of which is not necessarily to discover evidence of a crime,
but to allow the officer to pursue his investigation without fear of
violence. The trial court then ruled that the seizure of the grenade
from petitioner was incidental to a lawful arrest, and since
petitioner later voluntarily admitted such fact to the police
investigator for the purpose of bombing the Mercury Drug Store,
concluded that sufficient evidence existed to establish petitioners
guilt beyond reasonable doubt.
WHEREFORE, the challenged decision of the Seventeenth Division
of the Court of Appeals in CA-G.R. CR No. 15988 is SET ASIDE for
lack of jurisdiction on the part of said Court and, on ground of
reasonable doubt, the decision of 10 February 1994 of Branch 5 of
the Regional Trial Court of Manila is REVERSED and petitioner
SAMMY MALACAT y MANDAR is hereby ACQUITTED and ORDERED
immediately released from detention, unless his further detention is
justified for any other lawful cause.

People vs Jayson

Gr no 120320 November

18,1997
I.
Although not raised as an error by the accused-appellant, it is
pertinent to consider the circumstances surrounding accusedappellants arrest and the seizure from him of the firearm in question
considering that both were made without any warrant from a court.
With respect to the arrest, SPO1 Loreto Tenebro[11] testified that
at around 10:00 in the evening of March 16, 1991, while he and
Patrolmen Camotes and Reinerio Racolas were patrolling in their
car, they received a radio message from their camp directing them
to proceed to the Ihaw-Ihaw on Bonifacio Street where there had
been a shooting. Accordingly, they proceeded to the place and there
saw the victim, Nelson Jordan. Bystanders pointed to accusedappellant as the one who had shot Jordan. They then arrested
accused-appellant. Seized from him was a .38 caliber revolver with
serial number 91955. The firearm was covered by a mission order
and memorandum receipt. Considering these facts, we hold that the
warrantless arrest and search were valid.
Rule 113, 5(b) of the Revised Rules of Criminal Procedure
provides:
Sec. 5. Arrest without warrant; when lawful. A peace officer or
private person may, without a warrant, arrest a person: . . . .
(b) When an offense has in fact just been committed, and he has
personal knowledge of facts indicating that the person to be
arrested has committed it.
In the case at bar there was a shooting. The policemen
summoned to the scene of the crime found the victim. Accusedappellant was pointed to them as the assailant only moments after
the shooting. In fact accused-appellant had not gone very far (only
ten meters away from the Ihaw-Ihaw), although he was then fleeing.
The arresting officers thus acted on the basis of personal knowledge
of the death of the victim and of facts indicating that accusedappellant was the assailant.
This Court has upheld a warrantless arrest under analogous
circumstances. In People v. Tonog, Jr.,[12] the police found the lifeless
body of a person with several stab wounds. An informer pointed to
the accused as the person who had killed the victim. That
afternoon, police officers arrested the accused. On their way to the
police station, a policeman noticed bloodstains on the accuseds
pants which, when examined, was found to be the same blood type
O found on the fatal knife. The Court upheld the warrantless arrest
and ruled that the blood-stained pants, having been seized as an
incident of a lawful arrest, was admissible in evidence.
In People v. Gerente,[13] the police arrested the accused three
hours after the victim had been killed. They went to the scene of the
crime where they found a piece of wood and a concrete hollow
block used by the killers in bludgeoning the victim to death. A
neighbor of the accused who witnessed the killing, pointed to him
as one of the assailants. The warrantless arrest was held valid under
Rule 113, 5(b).
In People v. Acol,[14] a group held up a passenger jeepney.
Policemen immediately responded to the report of the crime. One of
the victims saw four persons walking towards Fort Bonifacio, one of
whom was wearing his jacket. He pointed them to the policemen.
When the group saw the policemen coming, they ran in different
directions. Three were caught and arrested. Each was found in
possession of an unlicensed revolver and charged with illegal
possession of firearms. The accused claimed that the warrantless
seizure of firearms was illegal. The Court rejected their plea and
held that the search was a valid incident of a lawful arrest.

The subsequent search of accused-appellants person and the (c) When the person to be arrested is a prisoner who escaped from a
seizure from him of the firearm was likewise lawful. Rule 126, 12
penal establishment or place where he is serving final judgment or
states:
temporarily confined while his case is pending, or has escaped
while being transferred from one confinement to another.
Sec. 12. Search incident to lawful arrest. A person lawfully arrested
may be searched for dangerous weapons or anything which may be
used as proof of the commission of an offense, without a search
warrant.
In People v. Lua,[15] a buy-bust operation was conducted
against the accused. After accused had gone inside his house and
returned with the three tea bags of marijuana and received the
marked money, the designated poseur-buyer gave the signal to his
fellow police officers who closed in and arrested the accused. In the
course of the arrest, a police officer noticed something bulging at
accuseds waistline, which turned out to be an unlicensed .38 caliber
paltik with two live bullets. Accused was charged with illegal
possession of firearm. The search was held to be a valid incident of
a lawful arrest.

People vs Manlangit gr no 189806


Ruling:
Contrary to accused-appellants challenge to the validity of the buybust operation, the Court categorically stated in Quinicot v. People
that a prior surveillance or test buy is not required for a valid buybust operation, as long as the operatives are accompanied by their
informant, thus:
Settled is the rule that the absence of a prior surveillance or test
buy does not affect the legality of the buy-bust operation. There is
no textbook method of conducting buy-bust operations. The Court
has left to the discretion of police authorities the selection of
effective means to apprehend drug dealers. A prior surveillance,
much less a lengthy one, is not necessary, especially where the
police operatives are accompanied by their informant during the
entrapment. Flexibility is a trait of good police work. We have held
that when time is of the essence, the police may dispense with the
need for prior surveillance. In the instant case, having been
accompanied by the informant to the person who was peddling the
dangerous drugs, the policemen need not have conducted any prior
surveillance before they undertook the buy-bust operation.[14]
(Emphasis supplied.)
Furthermore, accused-appellants contention that the buy-bust
team should have procured a search warrant for the validity of the
buy-bust operation is misplaced. The Court had the occasion to
address this issue in People v. Doria:[15]

Under Section 5 (a), as above-quoted, a person may be arrested


without a warrant if he has committed, is actually committing, or is
attempting to commit an offense. Appellant Doria was caught in the
act of committing an offense. When an accused is apprehended in
flagrante delicto as a result of a buy-bust operation, the police are
not only authorized but duty-bound to arrest him even without a
warrant.
The Court reiterated such ruling in People v. Agulay:[16]
Accused-appellant contends his arrest was illegal, making the
sachets of shabu allegedly recovered from him inadmissible in
evidence. Accused-appellants claim is devoid of merit for it is a
well-established rule that an arrest made after an entrapment
operation does not require a warrant inasmuch as it is considered a
valid warrantless arrest, in line with the provisions of Rule 113,
Section 5(a) of the Revised Rules of Court, to wit:
Section 5. Arrest without warrant; when lawful.A peace officer or a
private person may, without a warrant, arrest a person:
(a) When, in his presence, the person to be arrested has
committed, is actually committing, or is attempting to commit an
offense.
A buy-bust operation is a form of entrapment which in recent years
has been accepted as a valid and effective mode of apprehending
drug pushers. In a buy-bust operation, the idea to commit a crime
originates from the offender, without anybody inducing or prodding
him to commit the offense. If carried out with due regard for
constitutional and legal safeguards, a buy-bust operation deserves
judicial sanction

Fajardo vs. People gr no 190889 jan. 2011


First, we rule on the admissibility of the receivers. We hold that the
receivers were seized in plain view, hence, admissible.
No less than our Constitution recognizes the right of the people to
be secure in their persons, houses, papers, and effects against
unreasonable searches and seizures. This right is encapsulated in
Article III, Section 2, of the Constitution, which states:

Sec. 5. Arrest without warrant; when lawful.A peace officer or a


private person may, without a warrant, arrest a person:

Sec. 2. The right of the people to be secure in their persons,


houses, papers, and effects against unreasonable searches and
seizures of whatever nature and for any purpose shall be inviolable,
and no search warrant or warrant of arrest shall issue except upon
probable cause to be determined personally by the judge after
examination under oath or affirmation of the complainant and the
witnesses he may produce, and particularly describing the place to
be searched and the persons or things to be seized.

(a) When, in his presence, the person to be arrested has committed, is


actually committing, or is attempting to commit an offense;

Complementing this provision is the exclusionary rule embodied in


Section 3(2) of the same article

(b) When an offense has in fact just been committed, and he has
personal knowledge of facts indicating that the person to be
arrested has committed it; and

(2) Any evidence obtained in violation of this or the preceding


section shall be inadmissible for any purpose in any proceeding.

We also hold that the warrantless arrest of accused-appellant Doria


is not unlawful. Warrantless arrests are allowed in three instances
as provided by Section 5 of Rule 113 of the 1985 Rules on Criminal
Procedure, to wit:

There are, however, several well-recognized exceptions to the


foregoing rule. Thus, evidence obtained through a warrantless

search and seizure may be admissible under any of the following


circumstances: (1) search incident to a lawful arrest; (2) search of a
moving motor vehicle; (3) search in violation of custom laws; (4)
seizure of evidence in plain view; and (5) when the accused himself
waives his right against unreasonable searches and seizures.[18]
Under the plain view doctrine, objects falling in the plain view of an
officer, who has a right to be in the position to have that view, are
subject to seizure and may be presented as evidence.[19] It applies
when the following requisites concur: (a) the law enforcement
officer in search of the evidence has a prior justification for an
intrusion or is in a position from which he can view a particular
area; (b) the discovery of the evidence in plain view is inadvertent;
and (c) it is immediately apparent to the officer that the item he
observes may be evidence of a crime, contraband, or otherwise
subject to seizure. The law enforcement officer must lawfully make
an initial intrusion or properly be in a position from which he can
particularly view the area. In the course of such lawful intrusion, he
came inadvertently across a piece of evidence incriminating the
accused. The object must be open to eye and hand, and its
discovery inadvertent.[20]
Tested against these standards, we find that the seizure of the two
receivers of the .45 caliber pistol outside petitioners house falls
within the purview of the plain view doctrine.
First, the presence of SPO2 Nava at the back of the house and of
the other law enforcers around the premises was justified by the
fact that petitioner and Valerio were earlier seen respectively
holding .45 caliber pistols before they ran inside the structure and
sought refuge. The attendant circumstances and the evasive
actions of petitioner and Valerio when the law enforcers arrived
engendered a reasonable ground for the latter to believe that a
crime was being committed. There was thus sufficient probable
cause for the policemen to cordon off the house as they waited for
daybreak to apply for a search warrant.
Secondly, from where he was situated, SPO2 Nava clearly saw, on
two different instances, Valerio emerge on top of the subject
dwelling and throw suspicious objects. Lastly, considering the
earlier sighting of Valerio holding a pistol, SPO2 Nava had
reasonable ground to believe that the things thrown might be
contraband items, or evidence of the offense they were then
suspected of committing. Indeed, when subsequently recovered,
they turned out to be two (2) receivers of .45 caliber pistol.
The ensuing recovery of the receivers may have been deliberate;
nonetheless, their initial discovery was indubitably inadvertent. It is
not crucial that at initial sighting the seized contraband be
identified and known to be so. The law merely requires that the law
enforcer observes that the seized item may be evidence of a crime,
contraband, or otherwise subject to seizure.
Hence, as correctly declared by the CA, the two receivers were
admissible as evidence. The liability for their possession, however,
should fall only on Valerio and not on petitioner.

FACTS:
Atty. Tito Pintor and his client Manuel Montebon were
discussing the terms for the withdrawal of the complaint for direct
assault filed with the Office of the City Fiscal of Cebu against
Leonardo Laconico after demanding P 8,000.00 from him. This
demand was heard by Atty. Gaanan through a telephone extension
as requested by Laconico so as to personally hear the proposed
conditions for the settlement.
Atty. Pintor was subsequently arrested in an entrapment operation
upon receipt of the money. since Atty. Gaanan listened to the
telephone
conversation
without
complainant's
consent,
complainant charged Gaanan and Laconico with violation of the
Anti- Wiretapping Act (RA 4200).
ISSUE:
Whether or not an extension telephone is among the prohibited
devices in Sec. 1 of RA 4200 such that its use to overheard a
private conversation would constitute an unlawful interception of
communication between two parties using a telephone line.
HELD:
No. An extension telephone cannot be placed in the same
category as a dictaphone or dictagraph, or other devvices
enumerated in Sec. 1 of the law as the use thereof cannot be
considered as "tapping" the wire or cable of a telephone line. this
section refers to instruments whose installation or presence cannot
be presumed by the party or parties being overheard because, by
their very nature, they are of common usage and their purpose is
precisely for tapping, intercepting, or recording a telephone
conversation. The telephone extension in this case was not installed
for that purpose. It just happened to be there for ordinary office
use.
Furthermore, it is a general rule that penal statutes must be
construed strictly in favor of the accused. Thus in the case of doubt
as in this case, on whether or not an extension telephone is
included in the phrase "device or arrangement" the penal statute
must be construed as not including an extension telephone.
A perusal of the Senate Congressional Record shows that our
lawmakers intended to discourage, through punishment, persons
such as government authorities or representatives of organized
groups from installing devices in order to gather evidence for use in
court or to intimidate, blackmail or gain some unwarranted
advantage over the telephone users. Consequently, the mere act of
listeneing , in order to be punishable must strictly be with the use of
the enumerated devices in RA 4200 or other similar nature.

Salcedo- Ortanez vs CA Gr no 100662


aug 4 1994
Facts:

Section 3
Gaanan
vs.
Intermediate
Appellate Court
[GR L-69809, 16 October 1986]

Private respondent Rafael Ortanez filed with the Quezon City RTC a
complaint for annulment of marriage with damages against
petitioner Teresita Salcedo-Ortanez, on grounds of lack of marriage
license and/or psychological incapacity of the petitioner.
Among the exhibits offered by private respondent were three (3)
cassette tapes of alleged telephone conversations between
petitioner and unidentified persons.

Teresita submitted her Objection/Comment to Rafaels oral offer of


evidence. However, the trial court admitted all of private
respondents offered evidence and later on denied her motion for
reconsideration, prompting petitioner to file a petition for certiorari
with the CA to assail the admission in evidence of the
aforementioned cassette tapes.
These tape recordings were made and obtained when private
respondent allowed his friends from the military to wire tap his
home telephone.
CA denied the petition because (1) Tape recordings are not
inadmissible per se. They and any other variant thereof can be
admitted in evidence for certain purposes, depending on how they
are presented and offered and on how the trial judge utilizes them
in the interest of truth and fairness and the even handed
administration of justice; and (2) A petition for certiorari is
notoriously inappropriate to rectify a supposed error in admitting
evidence adduced during trial. The ruling on admissibility is
interlocutory; neither does it impinge on jurisdiction. If it is
erroneous, the ruling should be questioned in the appeal from the
judgment on the merits and not through the special civil action of
certiorari. The error, assuming gratuitously that it exists, cannot be
anymore than an error of law, properly correctible by appeal and
not by certiorari.
Petitioner then filed the present petition for review under Rule 45 of
the Rules of Court.
Issue: W/N the recordings of the telephone conversations are
admissible in evidence

phone conversation between then-President Gloria Arroyo and


Comelec Commissioner Virgilio Garcillano, respondent DOJ
Secretary Gonzales warned reporters that those who had copies of
the CD and those broadcasting or publishing its contents could be
held liable under the Anti-Wiretapping Act. He also stated that
persons possessing or airing said tapes were committing a
continuing offense, subject to arrest by anybody. Finally, he stated
that he had ordered the National Bureau of Investigation to go after
media organizations found to have caused the spread, the playing
and the printing of the contents of a tape.
Meanwhile, respondent NTC warned in a press release all radio
stations and TV network owners/operators that the conditions of the
authorization and permits issued to them by government like the
Provisional Authority and/or Certificate of Authority explicitly
provides that they shall not use their stations for the broadcasting
or telecasting of false information or willful misrepresentation. The
NTC stated that the continuous airing or broadcast of the Hello
Garci taped conversations by radio and
TV stations is a continuing violation of the Anti-Wiretapping Law and
the conditions of the Provisional Authority and/or Certificate of
Authority.
It warned that their broadcast/airing of such false
information and/or willful misrepresentation shall be a just cause for
the suspension, revocation and/or cancellation of the licenses or
authorizations issued to the said media establishments.
Subsequently, a dialogue was held between the NTC and the
Kapisanan ng mga Brodkaster sa Pilipinas (KBP) which resulted in
the issuance of a Joint Press Statement which stated, among others,
that the supposed wiretapped tapes should be treated with
sensitivity and handled responsibly.

Held:
No. Rep. Act No. 4200 entitled An Act to Prohibit and Penalize Wire
Tapping and Other Related Violations of the Privacy of
Communication, and for other purposes expressly makes such
tape recordings inadmissible in evidence thus:

Petitioner Chavez filed a petition under Rule 65 against respondents


Secretary Gonzales and the NTC directly with the Supreme Court.
II.

THE ISSUES

1. Will a purported violation of law such as the Anti-Wiretapping Law


Sec. 1. It shall be unlawful for any person, not being authorized by
justify straitjacketing the exercise of freedom of speech and of the
all the parties to any private communication or spoken word, to tap
press?
any wire or cable, or by using any other device or arrangement, to
secretly overhear, intercept, or record such communication or 2. Did the mere press statements of respondents DOJ Secretary and the
NTC constitute a form of content-based prior restraint that has
spoken word by using a device commonly known as a dictaphone
transgressed the Constitution?
or dictagraph or detectaphone or walkie-talkie or tape-recorder, or
however otherwise described. . . .
III. THE RULING
Sec. 4. Any communication or spoken word, or the existence,
[The Court voted 10-5 (CJ Puno, joined by JJ. Quisumbing, Ynarescontents, substance, purport, or meaning of the same or any part
Santiago, Sandoval-Gutierrez, Carpio, Austria-Martinez, Carpio
thereof, or any information therein contained, obtained or secured
Morales, Azcuna, Reyes and Tinga in the majority, as against JJ.
by any person in violation of the preceding sections of this Act shall
Corona, ChicoNazario, Nachura, Leonardo-De Castro and Velasco in
not be admissible in evidence in any judicial, quasijudicial,
the minority)
in granting the petition insofar as respondent
legislative or administrative hearing or investigation.
Secretary Gonzalezs press statement was concerned. Likewise, it
voted 10-5 (CJ Puno, joined by JJ. Quisumbing, Ynares-Santiago,
Absent a clear showing that both parties to the telephone
Sandoval-Gutierrez, Carpio, Austria-Martinez, Carpio Morales,
conversations allowed the recording of the same, the inadmissibility
Azcuna, Reyes and Velasco in the majority, as against JJ. Corona,
of the subject tapes is mandatory under Rep. Act No. 4200.
Chico-Nazario, Nachura, Leonardo-De Castro and Tinga in the
minority) in granting the same insofar as NTCs press statement
was concerned.]

Chavez vs Secretary Gonzales


(Hello Garci)
I.

THE FACTS

As a consequence of the public release of copies of the Hello


Garci compact disc audiotapes involving a wiretapped mobile

1.
NO, a purported violation of law such as the Anti-Wiretapping
Law will NOT justify straitjacketing the exercise of freedom of
speech and of the press.

A governmental action that restricts freedom of speech or of the


press based on content is given the strictest scrutiny, with the
government having the burden of overcoming the presumed
unconstitutionality by the clear and present danger rule. This rule
applies equally to all kinds of media, including broadcast media.
Respondents, who have the burden to show that these acts do not
abridge freedom of speech and of the press, failed to hurdle the
clear and present danger test. [T]he great evil which government
wants to prevent is the airing of a tape recording in alleged
violation of the anti-wiretapping law. The records of the case at bar
however are confused and confusing, and respondents evidence
falls short of satisfying the clear and present danger test. Firstly,
the various statements of the Press Secretary obfuscate the identity
of the voices in the tape recording. Secondly, the integrity of the
taped conversation is also suspect. The Press Secretary showed to
the public two versions, one supposed to be a complete version
and the other, an altered version. Thirdly, the evidence of the
respondents on the whos and the hows of the wiretapping act is
ambivalent, especially considering the tapes different versions. The
identity of the wire-tappers, the manner of its commission and
other related and relevant proofs are some of the invisibles of this
case. Fourthly, given all these unsettled facets of the tape, it is
even arguable whether its airing would violate the anti-wiretapping
law.
We rule that not every violation of a law will justify straitjacketing
the exercise of freedom of speech and of the press. Our laws are of
different kinds and doubtless, some of them provide norms of
conduct which[,] even if violated[,] have only an adverse effect on a
persons private comfort but does not endanger national security.
There are laws of great significance but their violation, by itself and
without more, cannot support suppression of free speech and free
press. In fine, violation of law is just a factor, a vital one to be sure,
which should be weighed in adjudging whether to restrain freedom
of speech and of the press. The totality of the injurious effects of
the violation to private and public interest must be calibrated in
light of the preferred status accorded by the Constitution and by
related international covenants protecting freedom of speech and of
the press. In calling for a careful and calibrated measurement of the
circumference of all these factors to determine compliance with the
clear and present danger test, the Court should not be
misinterpreted as devaluing violations of law.
By all means,
violations of law should be vigorously prosecuted by the State for
they breed their own evil consequence. But to repeat, the need to
prevent their violation cannot per se trump the exercise of free
speech and free press, a preferred right whose breach can lead to
greater evils. For this failure of the respondents alone to offer proof
to satisfy the clear and present danger test, the Court has no option
but to uphold the exercise of free speech and free press. There is no
showing that the feared violation of the anti-wiretapping law clearly
endangers the national security of the State.
2.
YES, the mere press statements of respondents DOJ Secretary
and the NTC constituted a form of content-based prior restraint that
has transgressed the Constitution.
[I]t
is
not
decisive that the press statements made by
respondents were not reduced in or followed up with formal orders
or circulars. It is sufficient that the press statements were made by
respondents while in the exercise of their official functions.
Undoubtedly, respondent Gonzales made his statements as
Secretary of Justice, while the NTC issued its statement as the
regulatory body of media. Any act done, such as a speech uttered,
for and on behalf of the government in an official capacity is
covered by the rule on prior restraint. The concept of an act does
not limit itself to acts already converted to a formal order or official
circular. Otherwise, the non formalization of an act into an official

order or circular will result in the easy circumvention of the


prohibition on prior restraint. The press statements at bar are acts
that should be struck down as they constitute impermissible forms
of prior restraints on the right to free speech and press.

US vs Gutz

(Fruit of the poisonous tree)

The "fruit of the poisonous tree" doctrine is an offspring of the


Exclusionary Rule. The exclusionary rule mandates that evidence
obtained from an illegal arrest, unreasonable search, or coercive
interrogation must be excluded from trial.

Mancusi vs De Forte

(Reasonable

expectation)
Mancusi v. DeForte, 392 U.S. 364, is a 1968 decision of the United
States Supreme Court on privacy and the Fourth Amendment. It
originated in the lower courts as United States ex rel. Frank
DeForte, appellant v. Vincent R. Mancusi, Warden of Attica Prison,
Attica, New York, appellee, a petition for a writ of habeas corpus by
a prisoner who had exhausted all his state appeals. By a 63 margin
the Court affirmed the United States Court of Appeals for the
Second Circuit's reversal of a district court denial of the petition.
The prisoner, Frank DeForte, was one of several labor union officials
on Long Island who had been convicted of racketeering-related
charges connected to a scheme in which they attempted to
monopolize the juke box market in the New York Metropolitan area.
Early in the investigation, local prosecutors had issued a subpoena
duces tecum for records from the union officials. When they refused
to comply, the prosecutors went to the union offices themselves
and seized the records from the officials' desks themselves. DeForte
had been present and voiced his objections. The state later
admitted the action was illegal but the documents, which formed
the bulk of the case against the officials, were not suppressed at
trial. Both the state's appellate court and the New York State Court
of Appeals sustained the verdict, and all the defendants went to
prison. There they began filing habeas petitions to the federal
courts. The first, alleging that the court's orders to the jury to
continue deliberating after they had done so for almost 24 hours
and twice asked for a break constituted coercion, was denied.
DeForte's second, arguing as he had at trial and on his state appeal,
that the search of his desk violated his reasonable expectation of
privacy and thus his Fourth Amendment rights, was the one the
Supreme Court heard. Justice John Marshall Harlan II wrote for the
majority that under the Court's recent holding in Katz v. United
States, DeForte had a reasonable expectation of privacy over the
papers he kept at work even though they were not his personal
property and he shared the office with his co-defendants. Nor did
the subpoena authorize the prosecutor to act as he might with a
search warrant, since the subpoena was not subject to independent
judicial review before its execution. In dissent, Hugo Black, who had
also dissented in Katz, said he could not find why the Court chose to
depart from previous holdings that documents in the possession of
one's employer enjoyed no Fourth Amendment protection, and was
misreading the cases it relied on.
The case is seen as a seminal case in privacy law, since it extended
it for the first time to a nonresidential space. Lower courts have
used it to guide them in distinguishing Fourth Amendment claims
into the present day. The Supreme Court has, in later holdings,

extended it to include public employees during administrative


investigations and considered its application in the context of
modern telecommunications.

Ramirez vs CA
Issue: Does the anti-wiretapping law, RA 4200, allow parties to a
conversation to tape it without the consent of all those involved?
What was construed: The word any in Sec. 1 of RA 4200: It shall
be unlawful for ANY person, not being authorized by all the parties
to any private communication or spoken word, to tap any wire or
cable, or by using any other device or arrangement, to secretly
overhear, intercept, or record such communication or spoken word
by using a device commonly known as a Dictaphone or dictagraph
or detectaphone or walkie-talkie or tape recorder, or however
otherwise described.
Facts of the case:
Soccoro Ramirez was scolded by Ester Garcia inside Garcias office.
Ramirez taped the conversation and later filed charges against
Garcia for insulting and humiliating her, using as evidence the
transcript of the conversation, based on the tape recording.
Garcia filed criminal charges against Ramirez for violating the antiwire tapping act, because it was done without her knowledge and
consent. Ramirez claimed that what the law forbids is for other
parties, who are not part of the conversation, to record it using the
instruments enumerated in the law (there was an earlier case that
was dismissed because the instrument used was not mentioned in
the law).
The trial court ruled in favor of Ramirez, granting a motion to quash
on the ground that the facts charged do not constitute an offense,
but the Court of Appeals reversed it.
Ratio: First, the court noted that the provision makes it clear that it
is illegal for any person to secretly record a conversation, unless
authorized by all parties involved.
The law makes no distinction as to whether the party sought to be
penalized by the statute ought to be a party other than or different
from those involved in the private communication.
The congressional records also showed that the intent was that
permission must be sought from all parties in the conversation.
This is a complete ban on tape recorded conversations taken
without the authorization of all the parties, Sen. Tanada said during
the deliberations.
The provision seeks to penalize even those privy to the private
communications. Where the law makes no distinctions, one does
not distinguish.
Decision: Petition denied. Decision of CA affirmed. Costs against
Ramirez.

G.R. No. 81561 January 18, 1991


FACTS:
August 14, 1957, the appellant and his common-law wife, Sherly
Reyes, went to the booth of the
Manila Packing and Export Forwarders carrying Four (4) wrapped
packages. The appellant informed Anita Reyes that he was sending
the packages to a friend in Zurich, Switzerland. Anita Reyes asked if
she could examine and inspect the packages. She refused and
assures her that the packages simply contained books, cigars, and
gloves.
Before the delivery of appellants box to the Bureau of Customs and
Bureau of Posts, Mr. Job Reyes (Proprietor), following the standard
operating procedure, opened the boxes for final inspection. A
peculiar odor emitted from the box and that the gloves contain
dried leaves. He prepared a letter and reported to the NBI and
requesting a laboratory examinations. The dried marijuana leaves
were found to have contained inside the cellophane wrappers.
The accused appellant assigns the following errors: The lower
court erred in admitting in evidence the illegality of search and
seized objects contained in the four (4) parcels.
ISSUE:
Whether or not the seizing of illegal objects is legal?
HELD:
Yes, appellant guilty beyond reasonable doubt.
RATIONALE:
Article III, Sections 2 and 3, 1987 Constitution
Mapp vs Ohio, exclusionary rule
Stonehill vs Diokno, declared as inadmissible any evidence
obtained by virtue of a defective search warrant, abandoning in the
process the ruling earlier adopted in Mercado vs Peoples Court.
The case at the bar assumes a peculiar character since the
evidence sought to be excluded was primarily discovered and
obtained by a private person, acting in a private capacity and
without the intervention and participation of state authorities.
Under the circumstances, can accused / appellant validly claim that
his constitutional right against unreasonable search and seizure.
The contraband in this case at bar having come into possession of
the government without the latter transgressing appellants rights
against unreasonable search and seizure, the Court sees no cogent
reason why the same should not be admitted.
FACTUAL CONSIDERATIONS Readily foreclose the proportion that
NBI agents conducted an illegal search and seizure of the prohibited
merchandise, clearly that the NBI agents made no search and
seizure much less an illegal one, contrary to the postulate of
accused / appellant.
CHADWICK vs STATE, having observed that which is open, where no
trespass has been committed in aid thereof

People vs Marti
PEOPLE OF THE PHILIPPINES vs ANDRE MARTI

13.

BILL OF RIGHTS

The protection of fundamental liberties in the essence of


constitutional democracy, protection against whom, protection
against the STATE.

Pollo vs Constantino

David

Gr

no

181881 Oct. 18 2011


Facts
Respondent CSC Chair Constantino-David received an anonymous
letter complaint alleging of an anomaly taking place in the Regional
Office of the CSC. The respondent then formed a team and issued a
memo directing the team to back up all the files in the computers
found in the Mamamayan Muna (PALD) and Legal divisions.
Several diskettes containing the back-up files sourced from the hard
disk of PALD and LSD computers were turned over to Chairperson
David. The contents of the diskettes were examined by the CSCs
Office for Legal Affairs (OLA). It was found that most of the files in
the 17 diskettes containing files copied from the computer assigned
to and being used by the petitioner, numbering about 40 to 42
documents, were draft pleadings or lettersin connection with
administrative cases in the CSC and other tribunals. On the basis of
this finding, Chairperson David issued the Show-Cause Order,
requiring the petitioner, who had gone on extended leave, to
submit his explanation or counter-affidavit within five days from
notice.
In his Comment, petitioner denied the accusations against him and
accused the CSC Officials of fishing expedition when they
unlawfully copied and printed personal files in his computer.
He was charged of violating R.A. No. 6713 (Code of Conduct and
Ethical Standards for Public Officials and
Employees). He assailed the formal charge and filed an Omnibus
Motion ((For Reconsideration, to Dismiss and/or to Defer) assailing
the formal charge as without basis having proceeded from an illegal
search which is beyond the authority of the CSC Chairman, such
power pertaining solely to the court.
The CSC denied the omnibus motion and treated the motion as the
petitioners answer to the charge. In view of the absence of
petitioner and his counsel, and upon the motion of the prosecution,
petitioner was deemed to have waived his right to the formal
investigation which then proceeded ex parte.
The petitioner was dismissed from service. He filed a petition to the
CA which was dismissed by the latter on the ground that it found no
grave abuse of discretion on the part of the respondents. He filed a
motion for reconsideration which was further denied by the
appellate court. Hence, this petition.

Issue
WON the search conducted by the CSC on the computer of the
petitioner constituted an illegal search and was a violation of his
constitutional right to privacy
Ruling
The search conducted on his office computer and the copying of his
personal files was lawful and did not violate his constitutional right.
Ratio Decidendi
In this case, the Court had the chance to present the cases
illustrative of the issue raised by the petitioner.

Katz v. United States 389 U.S. 437 (1967), the US Supreme Court
held that the act of FBI agents in electronically recording a
conversation made by petitioner in an enclosed public telephone
booth violated his right to privacy and constituted a search and
seizure. Because the petitioner had a reasonable expectation of
privacy in using the enclosed booth to make a personal telephone
call, the protection of the Fourth Amendment extends to such area.
Moreso, the concurring opinion of Mr. Justice Harlan noted that the
existence of privacy right under prior decisions involved a two-fold
requirement: first, that a person has exhibited an actual
(subjective) expectation of privacy; and second, that the
expectation be one that society is prepared to recognize as
reasonable (objective).
Mancusi v. DeForte 392 U.S. 364, 88 S.Ct. 2120, 20 L.Ed2d 1154
(1968),thus recognized that employees may have a reasonable
expectation of privacy against intrusions by police.
OConnor v. Ortega 480 U.S. 709 (1987), the Court categorically
declared that [i]ndividuals do not lose
Fourth Amendment rights merely because they work for the
government instead of a private employer. In OConnor the Court
recognized that special needs authorize warrantless searches
involving public employees for work-related reasons. The Court thus
laid down a balancing test under which government interests are
weighed against the employees reasonable expectation of privacy.
This reasonableness test implicates neither probable cause nor the
warrant requirement, which are related to law enforcement.
Social Justice Society (SJS) v. Dangerous Drugs Board G.R. Nos.
157870, 158633 and 161658, November 3, 2008, 570 SCRA 410,
427, (citing Ople v. Torres, G.R. No. 127685, July 23, 1998, 293
SCRA 141, 169), recognized the fact that there may be such
legitimate intrusion of privacy in the workplace.
The Court ruled that the petitioner did not have a reasonable
expectation of privacy in his office and computer files.
As to the second point of inquiry, the Court answered in the
affirmative. The search authorized by the CSC Chair, the copying of
the contents of the hard drive on petitioners computer reasonable
in its inception and scope.
The Court noted that unlike in the case of Anonymous LetterComplaint against Atty. Miguel Morales,
Clerk of Court, Metropolitan Trial Court of Manila A.M. Nos. P-082519 and P-08-2520, November 19, 2008, 571 SCRA 361, the case
at bar involves the computer from which the personal files of the
petitioner were retrieved is a government-issued computer, hence
government property the use of which the CSC has absolute right to
regulate and monitor.

Capuccino vs Apolonio T-041771 Sep 5


2011
THE COURTS RULING
The issue in an administrative case is not essentially about the
wrong inflicted on the complainant by the respondent; the main
question is whether the accused employee breached the norms and
standards of service in the judiciary.[26] We resolve this case based
on this perspective and not on the basis of whether respondents
Taguba, Apolonio and Santiago violated the Anti-Wire Tapping Act.

Taguba denied that he was motivated by malice in bringing


Valencias deposit of funds to Judge Rosetes attention and in filing a
complaint against Duque based on the taped conversation. He
believed that the taping was for the good of the service; all he
wanted was to ferret out the truth. He insisted that Atty. Capuchino
has no cause to file the complaint against them because the
criminal case of his client had already been terminated. Santiago
denied any participation in the taping, insisting that she was
implicated because she was the owner of the tape recorder used. It
was borrowed from her by somebody whom she could no longer
remember. On her part, Apolonio, together with Aspiras, maintained
that the accusation against them cannot prosper because the
matters covered are matters of public interest the interest of the
Judiciary itself.
The Court finds the respondents contentions without merit. Their
concerted acts of leading Atty. Capuchino and Valencia into the
court sala, engaging them in conversation regarding the money
deposited with Duque, taping their conversation without
Capuchinos & Valencias knowledge, and later using the taped
conversation as basis of the complaint they filed against Duque
constitute misconduct. Santiagos claim that she forgot who
borrowed her tape recorder and for what purpose it was borrowed is
not credible.
The Court observes that there exists animosity among the judges
and employees of the court. When the present case was referred to
Judge Plata for investigation, he inhibited himself on the ground
that the respondents had filed a complaint against him and that he
had also filed a criminal case against all of them. The filing of the
complaint against Duque was instigated by Taguba. Initially signed
only by Taguba, he prevailed upon the other respondents to co-sign
his letter addressed to then Chief Justice Hilario G. Davide, Jr., which
was later docketed as A.M. No. P-05-1958. He introduced as
evidence in this complaint the tape recorded conversation.
Although Duque was penalized for simple misconduct, the Court
found that there was no evidence that she was moved by evident
bad faith, dishonesty or hatred[27] in receiving Valencias money for
safekeeping. We cannot say the same of Tagubas actions in the
animosity-ridden atmosphere apparently obtaining in the MTCC of
Santiago City.
Making false accusations and sowing intrigues are acts
unbecoming of a public servant. They run against the principles of
public service envisioned by the 1987 Constitution and by the Code
of Conduct and Ethical Standards for Public Officials & Employees
(Republic Act No. 6713). These acts divert the attention of public
employees and the courts from their more important tasks, and
result in undue wastage of government resources; they cannot be
tolerated if we are to demand the highest degree of excellence and
professionalism among public employees, and if we are to preserve
the integrity and dignity of our courts.[28]
Misconduct, on the other hand, is a transgression of some definite
or established rule of action; more particularly, it is unlawful
behavior by the public officer and refers as well to wrongful or
improper behavior under applicable provisions of the Code of
Ethics. The term gross connotes something out of all measure;
beyond allowance; flagrant; shameful such conduct as is not be
excused.[29] For administrative liability to attach, it must be
established that the respondent was moved by bad faith,
dishonesty, hatred or other similar motives.[30]
Clearly, substantial evidence exists in this case to hold Taguba
guilty of gross misconduct punishable by
dismissal from the service even for the first offense. Not only did he
disregard the terms of the AntiWiretapping Act within court
premises where the public should feel most secure about their

personal liberties. He undertook the act to secure evidence against


a co-employee; he obtained and used the taped conversation as
basis for a complaint against Duque who was penalized for the
deposit she had accepted. We cannot accept, under these
circumstances, any claimed absence of bad faith after considering
the devious method Taguba employed and the purpose that it
served, however lofty Taguba thought his purpose had been.
Unfortunately, we can no longer impose the penalty of dismissal on
Taguba because he has retired from the service on disability
effective September 1, 2006. Additionally, we recently found
Taguba guilty of gross misconduct in another case A.M. No. MTJ-081727, entitled Milagros Villaceran and Omar T. Miranda v. Judge
Maxwel Rosete and Process Server Eugenio Taguba, etc.[31] for
soliciting P25,000.00 from the defendant in a pending case with the
promise that he would work for the defendants acquittal. In lieu of
the dismissal that at that point we could no longer impose because
of his previous retirement, the Court given the gravity of
respondent Tagubas offense ordered the forfeiture of Tagubas
disability retirement benefits. While we therefore find Taguba
administratively liable in the present case, we have run out of
administrative penalties to impose on him. Nothing, however, can
stop us from holding and declaring him liable for the gross
misconduct that he stands charged with.
For their participation in the illegal tape recording of the
complainant and his client, the Court finds respondents Apolonio
and Santiago guilty of simple misconduct. We so rule given the
evidence that they merely followed the lead of Taguba. Under the
Uniform Rules on Administrative Cases in the Civil
Service, simple misconduct is a less grave offense punishable by
suspension for one (1) month and one
(1) day to six (6) months for the first offense, and dismissal for the
second offense.[32]
Since the penalty of dismissal can no longer be imposed on
respondent Taguba, we can only reiterate the directive in A.M. No.
MTJ-08-1727[33] ordering the forfeiture of his remaining retirement
benefits.
Respondent Maripi A. Apolonio has previously been found guilty of
simple misconduct for gambling during office hours, together with
respondents Taguba and Andres, in A.M. No. P-01-1517.[34] They
were suspended for one (1) month and one (1) day. Since this is
Maripi A. Apolonios second offense, the penalty of dismissal should
be imposed. We opt, however, to merely order her SUSPENSION
from the service for one (1) year effective immediately, in light of
our recognition that her present act is different in nature from her
first offense; the elements of perversity and impenitence that are
considered in a repetition of the same offense are not necessarily
present. Thus, we accord her the benefit of the doubt.
This is respondent Ana Gracia E. Santiagos first offense; thus, the
Court hereby imposes on her a lighter penalty and orders her
SUSPENSION from the service for only six (6) months.
WHEREFORE, the Court finds respondent Eugenio P. Taguba guilty
of GROSS MISCONDUCT, and respondents Maripi A. Apolonio and
Ana Gracia E. Santiago guilty of SIMPLE MISCONDUCT.
Maripi A. Apolonio is ordered SUSPENDED for one year effective
immediately, with the warning that any similar or graver offense at
any time in the future shall merit the penalty of outright dismissal.
Ana Gracia E. Santiago is hereby ordered SUSPENDED for six (6)
months effective immediately, with the warning that any similar or
graver offense at any time in the future shall merit the penalty of
outright dismissal.

Let a copy of the records of OMB-L-C-03-0619-E be returned to the


Office of the Ombudsman and a copy of this Decision be furnished
the said Office, for appropriate action with respect to the criminal
aspect of the case. SO ORDERED.

Section 4
AYER vs. CAPULONG
Facts: Hal McElroy, an Australian film-maker, wanted to join the
Peoples Power Revolution in a movie entitled The 4-day
Revolution. The movie was a mixture of fiction and history to that
in addition to a love story, prominent personalities, like Enrile, had
to be portrayed. While the production was in progress, Enrile
obtained a court injunction to stop it. He argued that the film
violated his right to privacy. TAU MU

Issue (1): WON Sec. 4 also protects foreigners


Ruling (1): The court held that the Freedom of Expression protects
not only citizens of the Philippines but also foreigners in our
country. Sec.4 also extends to commercial media. Even if they did it
for profit, they are also protected. The reason is that most media is
privately owned and operates for profit. To prohibit them would
render Sec. 4 useless. Nobody can say anything anymore.
Issue (2): WON it violated Enriles right to privacy
Ruling (2): As to the contention of Enrile that it violated his right to
privacy. The SC looked at the particular circumstances and did not
apply any formula to decide on the issue of which shall prevail:
right to privacy or the freedom of expression. The court ruled that
the events that were portrayed were of public interest and Enrile is
also a public figure. And because of this, the SC is constrained to
rule that his Right to Privacy shall give way to Freedom of
Expression. Only the balancing of interest was used by the SC in
deciding(during this time, Enrile was a Senator). What is protected
by the Right to Privacy is unwarranted publicity and wrongful
publicizing of private affairs. The trial judge should not have issued
an injunction beforehand because of the preferred character of the
Right of Freedom of Speech and of Expression. And while production
was still in progress, no one knew whether the final outcome would
pose a clear and present danger. There should have been no prior
restraint because there was no basis yet.
NOTE: Private individuals have more rights than public figures.
The decision did not, however, define what a public figure is. The
right to privacy comes out of the shadows of the other rights in the
Constitution. There is no textual grant of the Right of Privacy found
in the Constitution.

MTRCB vs ABS CBN


01/17/05
G.R. No. 155282 January 17, 2005
Facts:

Gr

no.

155282

On October 15, 1991, at 10:45 in the evening, respondent ABS-CBN


aired Prosti-tuition, an episode of the television (TV) program
The Inside Story produced and hosted by respondent Legarda. It
depicted female students moonlighting as prostitutes to enable
them to pay for their tuition fees. In the course of the program,
student prostitutes, pimps, customers, and some faculty members
were interviewed. The Philippine Womens University (PWU) was
named as the school of some of the students involved and the
facade of PWU Building at Taft Avenue, Manila conspicuously served
as the background of the episode.
The showing of The Inside Story caused uproar in the
PWU community. Dr. Leticia P. de Guzman, Chancellor and Trustee
of the PWU, and the PWU Parents and Teachers Association filed
letter-complaints with petitioner MTRCB. Both complainants alleged
that the episode besmirched the name of the PWU and resulted in
the harassment of some of its female students.
Acting on the letter-complaints, the MTRCB Legal Counsel initiated a
formal complaint with the MTRCB Investigating Committee, alleging
among others, that respondents did not submit The Inside Story
to petitioner for its review and exhibited the same without its
permission, thus, violating Section 7 of Presidential Decree (P.D.)
No. 1986 and Section 3, Chapter III and Section 7, Chapter IV of the
MTRCB Rules and Regulations.
On February 5, 1993, after hearing and submission of the parties
memoranda, the MTRCB Investigating Committee rendered a
Decision, the decretal portion of which reads:
WHEREFORE, the aforementioned premises, the respondents are
ordered to pay the sum of TWENTY THOUSAND PESOS (P20,000.00)
for non-submission of the program, subject of this case for review
and approval of the MTRCB.
Heretofore, all subsequent programs of the The Inside Story and
all other programs of the ABS-CBN Channel 2 of the same category
shall be submitted to the Board of Review and Approval before
showing; otherwise the Board will act accordingly.
Respondents then filed a special civil action for certiorari with the
Regional Trial Court (RTC), Branch 77, Quezon City. It seeks to:
declare as unconstitutional Sections 3(b), 3(c), 3(d), 4, 7, and 11 of
P. D. No.
1986 and Sections 3, 7, and 28 (a) of the MTRCB Rules and
Regulations; (in the alternative) exclude the
The Inside Story from the coverage of the above cited provisions;
and annul and set aside the MTRCB Decision dated March 12, 1993
and Resolution dated April 14, 1993. Respondents averred that the
above-cited provisions constitute prior restraint on respondents
exercise of freedom of expression and of the press, and, therefore,
unconstitutional. Furthermore, the above cited provisions do not
apply to the The Inside Story because it falls under the category
of public affairs program, news documentary, or socio-political
editorials governed by standards similar to those governing
newspapers. The RTC rendered a decision in favour of the
respondent.

Petitioner MTRCB through the Solicitor General, contends inter alia:


first, all television programs, including public affairs programs,
news documentaries, or socio-political editorials, are subject to
petitioners power of review under Section 3 (b) of P.D. No. 1986
and pursuant to this Courts ruling in Iglesia ni Cristo vs. Court of
Appeals; second, television programs are more accessible to the
public than newspapers, thus, the liberal regulation of the latter
cannot apply to the former; third, petitioners power to review
television programs under Section 3(b) of P. D. No. 1986 does not
amount to prior restraint; and fourth, Section 3(b) of P. D. No.
1986 does not violate respondents constitutional freedom of
expression and of the press.
Issue:
Whether the MTRCB has the power or authority to review the
The Inside Story prior to its exhibition or broadcast by television?
Ruling:
Settled is the rule in statutory construction that where the
law does not make any exception, courts may not except something
therefrom, unless there is compelling reason apparent in the law to
justify it. Ubi lex non distinguit nec distinguere debemos. Thus,
when the law says all television programs, the word all covers
all television programs, whether religious, public affairs, news
documentary, etc. The principle assumes that the legislative body
made no qualification in the use of general word or expression.
It bears stressing that the sole issue here is whether
petitioner MTRCB has authority to review The Inside Story.
Clearly, we are not called upon to determine whether petitioner
violated Section 4, Article III (Bill of Rights) of the Constitution
providing that no law shall be passed abridging the freedom of
speech, of oppression or the press. Petitioner did not disapprove or
ban the showing of the program. Neither did it cancel respondents
permit. Respondents were merely penalized for their failure to
submit to petitioner The Inside Story for its review and approval.
Therefore, we need not resolve whether certain provisions of P. D.
No. 1986 and the MTRCB Rules and Regulations specified by
respondents contravene the Constitution.
Consequently, we cannot sustain the RTCs ruling that Sections 3 (c)
(d), 4, 7 and 11 of P. D. No. 1986 and Sections 3, 7 and 28 (a) of the
MTRCB Rules and Regulations are unconstitutional. It is settled that
no question involving the constitutionality or validity of a law or
governmental act may be heard and decided by the court unless
there is compliance with the legal requisites for judicial inquiry,
namely: (1) that the question must be raised by the proper party;
(2) that there must be an actual case or controversy; (3) that the
question must be raised at the earliest possible opportunity; and,
(4) that the decision on the constitutional or legal question must be
necessary to the determination of the case itself.
WHEREFORE, the instant petition is GRANTED. The assailed RTC
Decision dated November 18, 1997 and Order dated August 26,
2002 are hereby REVERSED. The Decision dated March 12, 1993 of
petitioner MTRCB is AFFIRMED. Costs against respondents.

Chavez vs Gonzales Gr no. 168338


CHAVEZ vs. GONZALES
555 SCRA 441
Facts:

Press Secretary Bunye told reporters that the opposition was


planning to destabilize the administration by releasing an audiotape
of a mobile phone conversation allegedly between the PGMA and a
highranking official of the COMELEC.
The conversation was
audiotaped allegedly through wire-tapping. Later, in a Malacaang
press briefing, Secretary Bunye produced two versions of the tape,
one supposedly the complete version, and the other, a spliced,
doctored or altered version, which would suggest that the
President had instructed the COMELEC official to manipulate the
election results in the Presidents favor. It seems that Secretary
Bunye admitted that the voice was that of President Arroyo, but
subsequently made a retraction.
Counsel of ERAP subsequently released an alleged
authentic tape recording of the wiretap.
Included in the tapes were purported conversations of the
President, the First Gentleman Jose Miguel
Arroyo, COMELEC Commissioner Garcillano, and the late Senator
Barbers. Respondent DOJ Secretary Raul Gonzales warned reporters
that those who had copies of the compact disc (CD) and those
broadcasting or publishing its contents could be held liable under
the Anti-Wiretapping Act. These persons included Secretary Bunye
and Atty. Paguia, counsel of ERAP. He also stated that persons
possessing or airing said tapes were committing a continuing
offense, subject to arrest by anybody who had personal knowledge
if the crime was committed or was being committed in their
presence. Secretary Gonzales ordered the NBI to go after media
organizations found to have caused the spread, the playing and
the printing of the contents of a tape of an alleged wiretapped
conversation involving the President about fixing votes in the 2004
national elections.
Thereafter, the NTC issued a press release which states
that: NTC GIVES FAIR WARNING TO
RADIO AND TELEVISION OWNERS/OPERATORS TO OBSERVE ANTIWIRETAPPING LAW AND PERTINENT CIRCULARS ON PROGRAM
STANDARDS. Thereafter, NTC held a dialogue with the Board of
Directors of the Kapisanan ng mga Brodkaster sa Pilipinas (KBP).
NTC allegedly assured the KBP that the press release did not violate
the constitutional freedom of speech, of expression, and of the
press, and the right to information.
Petitioner Chavez filed a petition against respondents
Secretary Gonzales and the NTC alleging that the acts of
respondents are violations of the freedom of expression and of the
press, and the right of the people to information on matters of
public concern. Respondents denied that the acts transgress the
Constitution, and questioned petitioners legal standing to file the
petition.
Issue: WON the mere press statements of the Secretary of Justice
and of the NTC in question constitute a form of content-based prior
restraint that has transgressed the constitutionally protected
freedoms of speech, of expression and of the press.
Ruling:
The SC held that it is not decisive that the press statements made
by respondents were not reduced in or followed up with formal
orders or circulars. It is sufficient that the press statements were
made by respondents while in the exercise of their official functions.
Undoubtedly, respondent Gonzales made his statements as
Secretary of Justice, while the NTC issued its statement as the
regulatory body of media. Any act done, such as a speech uttered,
for and on behalf of the government in an official capacity is
covered by the rule on prior restraint. The concept of an act does
not limit itself to acts already converted to a formal order or official
circular. Otherwise, the non formalization of an act into an official
order or circular will result in the easy circumvention of the

prohibition on prior restraint. The press statements at bar are acts


that should be struck down as they constitute impermissible forms
of prior restraints on the right to free speech and press.
There is enough evidence of chilling effect of the
complained acts on record. The warnings given to media came
from no less the NTC, a regulatory agency that can cancel the
Certificate of Authority of the radio and broadcast media. They also
came from the Secretary of Justice, the alter ego of the Executive,
who wields the awesome power to prosecute those perceived to be
violating the laws of the land.
After the warnings, the KBP
inexplicably joined the NTC in issuing an ambivalent Joint Press
Statement. After the warnings, petitioner Chavez was left alone to
fight this battle for freedom of speech and of the press. This silence
on the sidelines on the part of some media practitioners is too
deafening to be the subject of misinterpretation.
The constitutional imperative to strike down unconstitutional acts
should always be exercised with care and in light of the distinct
facts of each case. For there are no hard and fast rules when it
comes to slippery constitutional questions and the limits and
construct of relative freedoms are never set in stone, issues
revolving on their construct must be decided on a case to case
basis, always based on the peculiar shapes and shadows of each
case. But in cases where the challenged acts are patent invasions
of a constitutionally protected right, the SC must be swift in striking
them down as nullities per se. A blow too soon struck for freedom is
preferred than a blow too late.

Analyzed for meaning and weighed in its consequences, the article


written bybthe accused, cannot fail to impress thinking persons that
it seeks to sow the seeds of sedition and strife. The infuriating
language is not a sincere effort to persuade, what with the writer's
simulated suicide and false claim to martyrdom and what with is
failure to particularize. When the use irritating language centers not
on persuading the readers but on creating disturbances, the
rationable of free speech cannot apply and the speaker or writer is
removed from the protection of the constitutional guaranty.
If it be argued that the article does not discredit the entire
governmental structure but only President Roxas and his men, the
reply is that article 142 punishes not only all libels against the
Government but also "libels against any of the duly constituted
authorities thereof." The "Roxas people" in the Government
obviously refer of least to the President, his Cabinet and the
majority of legislators to whom the adjectives dirty, Hitlers and
Mussolinis were naturally directed. On this score alone the
conviction could be upheld.
Regarding the publication, it suggests or incites rebellious
conspiracies or riots and tends to stir up people against the
constituted authorities, or to provoke violence from opposition who
may seek to silence the writer. Which is the sum and substance of
the offense under consideration.
The essence of seditious libel may be said to its immediate
tendency to stir up general discontent to the pitch of illegal
courses; that is to say to induce people to resort to illegal methods
other than those provided by the Constitution, in order to repress
the evils which press upon their minds.

Espuelas vs People
G.R. No. L-2990 December 17, 1951
Facts:
On June 9 and June 24, 1947, both dates inclusive, in the town of
Tagbilaran, Bohol, Oscar Espuelas y Mendoza had his picture taken,
making it to appear as if he were hanging lifeless at the end of a
piece of rope suspended form the limb of the tree, when in truth
and in fact, he was merely standing on a barrel. After securing
copies of his photograph, Espuelas sent copies of same to Free
Press, the Evening News, the Bisayas, Lamdang of general
circulation and other local periodicals in the Province of Bohol but
also throughout the Philippines and abroad, for their publication
with a suicide note or letter, wherein he made to appear that it was
written by a fictitious suicide, Alberto Reveniera and addressed to
the latter's supposed wife translation of which letter or note, stating
his dismay and administration of President Roxas, pointing out the
situation in Central Luzon and Leyte, and directing his wife his dear
wife to write to President Truman and Churchill of US and tell them
that in the Philippines the government is infested with many Hitlers
and Mussolinis.
Issue:
Whether the accused is liable of seditious libel under Art. 142 of the
RPC against the Government of the Philippines?
Held:
Yes. The accused must therefore be found guilty as charged. And
there being no question as to the legality of the penalty imposed on
him, the decision will be affirmed with costs.

New Sound Broadcasting vs


Cesar Dy Gr no 170270 April 2, 2009
Facts:
Petitioners operate and run Bombo Radyo DZNC Cauayan (DZNC),
an AM radio broadcast station, and
Star FM DWIT Cauayan, an FM radio broadcast station, in Cauayan
Citry, Isabela. Back in 1996, Newsounds commenced relocation of
its broadcasting station, management office, and transmitters on
propery located in Minante 2, Cauayan City, Isabela.
On July 1996, the Housing & Land Use Regulatory Board (HLURB)
and Office of the Municipal Planning and Development Coordinator
(OMPDC) affirmed and certified that the commercial structure to be
constructed conformed to local zoning regulations, noting as well
that the location is classified as a commercial area. The radio
station was able to fully operate smoothly thereafter.
In 2002 however, when petitioners applied for a renewal of mayors
permit, City Zoning AdministratiorDesignate Bagnos Maximo
refused to issue zoning clearance on the grounds that petitioners
were not able to submit conversion papers showing that the
agricultural land was converted to commercial land. Petitioners
asked the court to compel the issuance of mayors permit but the
court denied the action. In the meantime, the Department of
Agrarian Reform (DAR) Region II office issued to petitioners a formal
recognition of conversion of the property from agricultural to
commercial.
In 2003, petitioners again filed their application for renewal of
mayors permit, attaching the DAR Order. Respondent Felicisimo

Meer, acting City Administrator of Cauayan City denied the same,


claiming that it was void on the grounds that they did not have
record of the DAR Order.
The deadline lapsed on Febuary 15, 2004, and respondents Meer
and Racma Fernandez-Garcia, City Legal Officer of Cauayan City,
closed the radio station. Due to the prvosion of Omnibus Election
Code which prohibits the closure of radio station during the
pendency of election period, COMELEC issued an order allowing the
petitioners to operate before Febuary 17, 2004, but was barred
again by respondent Mayor Ceasar Dy on the grounds that the radio
station had no permit. Nonetheless, COMELEC allowed them to run
again until June 10, 2004 after elections.
Petitioners filed the case to the RTC and CA for the issuance of
mayors permit but both courts denied the petition.
A municipal or city mayor is likewise authorized under the LGC to
issue licenses and permits, and suspend or revoke the same for any
violation of the conditions upon which said licenses or permits had
been issued, pursuant to law or ordinance. In case of Cauayan City,
the authority to require a mayors permit was enacted through
Ordinance No. 92-004, enacted in 1993. However, nothing in the
ordinance requires an application for a mayors permit to submit
either an approved land conversion papers from DAR, showing that
its property was converted from prime agricultural land or an
approved resolution from the Sangguniang Bayan or Sangguniang
Panglungsod authorizing the reclassification of property from
agricultural to commercial land.
In 1996, the HLURB issued a zoning decision that classified the
property as commercial. Petitioners are also armed with several
certifications stating that the property is indeed a commercial area.
Also, petitioners paid real property taxes based on the classification
of property as commercial without objections raised by the
respondents.
Petitioners argued that this consistent recognition by the local
government of Cauayan of the commercial character of the
property constitutes estoppels against respondents from denying
the fact before the courts. The lower courts had ruled that the
government of Cauayan City is not bound by estoppels, but
petitioners classified that this concept is understood to only refer to
acts and mistakes of its official especially to those which are
irregular.
Issue:
Whether the lower court is correct in contending that the
government of Cauayan City is not bound by estoppels on the
grounds that the state is immune against suits.
Held:
No. While it is true that the state cannot be put in estoppels by
mistake or error of its officials or agents, there is an exception.
Estoppels against the public are little favored. They should not be
invoked except in rare and unusual circumstances, and may not be
invoked where they would operate to defeat the effective operation
of a policy adopted to protect the public. They must be applied with
circumspection and should be applied only in those special cases
where the interests of justice clearly require it. Nevertheless, the
government must not be allowed to deal dishonorably or
capriciously with its citizens, and must not play an ignoble part or
do a shabby thing; and subject to limitations . . ., the doctrine of
equitable estoppel may be invoked against public authorities as
well as against private individuals

Thus, when there is no convincing evidence to prove irregularity or


negligence on the part of the government official whose acts are
being disowned other than the bare assertion on the part of the
State, the Supreme Court have declined to apply State immunity
from estoppel. Herein, there is absolutely no evidence other than
the bare assertions of the respondents that the Cauayan City
government had previously erred when it certified that the property
had been zoned for commercial use. The absence of any evidence
other than bare assertions that the 1996 to 2001 certifications were
incorrect lead to the ineluctable conclusion that respondents are
estopped from asserting that the previous recognition of the
property as commercial was wrong.
Respondents were further estopped from disclaiming the previous
consistent recognition by the Cauayan City government that the
property was commercially zoned unless they had evidence, which
they had none, that the local officials who issued such certifications
acted irregularly in doing so. It is thus evident that respondents had
no valid cause at all to even require petitioners to secure approved
land conversion papers from the DAR showing that the property
was converted from prime agricultural land to commercial land.
Respondents closure of petitioners radio stations is clearly tainted
with ill motvies. Petitioners have been aggressive in exposing the
widespread election irregularities in Isabela that appear to have
favored respondent Dy and his political dynasty. Such statement
manifests and confirms that respondents denial of the renewal
applications on the ground that property is commercial and merely
a pretext, and their real agenda is to remove petitioners from
Cauayan City and suppress the latters voice. This is a blatant
violation of constitutional right to press freedom.
WHEREFORE, the petitions are GRANTED. The assailed decisions of
the Court of Appeals and the Regional Trial Court of Cauayan City,
Branch 24, are hereby REVERSED and SET ASIDE. The instant
petition for mandamus is hereby GRANTED and respondents are
directed to immediately issue petitioners zoning clearances and
mayors permits for 2004 to petitioners.

David vs Arroyo [Right to Assembly]


RANDOLF DAVID, ET AL. VS. GLORIA MACAPAGAL-ARROYO, ET AL.
G.R. No. 171396, 171409, 171485, 171483, 171400, 171489 &
171424
May 3, 2006
Facts:
On February 24, 2006, as the nation celebrated the 20th
Anniversary of the Edsa People Power I,
President Arroyo issued PP 1017 declaring a state of national
emergency and call upon the Armed Forces of the Philippines (AFP)
and the Philippine National Police (PNP), to prevent and suppress
acts of terrorism and lawless violence in the country. The Office of
the President announced the cancellation of all programs and
activities related to the 20th anniversary celebration of Edsa People
Power I; and revoked the permits to hold rallies issued earlier by the
local governments and dispersal of the rallyists along EDSA. The
police arrested (without warrant) petitioner Randolf S. David, a
professor at the University of the Philippines and newspaper
columnist. Also arrested was his companion, Ronald Llamas,
president of party-list Akbayan.

In the early morning of February 25, 2006, operatives of the


Criminal Investigation and Detection Group (CIDG) of the PNP, on
the basis of PP 1017 and G.O. No. 5, raided the Daily Tribune offices
in Manila and attempt to arrest was made against representatives
of ANAKPAWIS, GABRIELA and BAYAN MUNA whom suspected of
inciting to sedition and rebellion. On March 3, 2006, President
Arroyo issued PP 1021 declaring that the state of national
emergency has ceased to exist. Petitioners filed seven (7) certiorari
with the Supreme Court and three (3) of those petitions impleaded
President Arroyo as respondent questioning the legality of the
proclamation, alleging that it encroaches the emergency powers of
Congress and it violates the constitutional guarantees of freedom of
the press, of speech and assembly.
Issue:
1.) Whether or
unconstitutional?

not

Presidential

Proclamation

No.

1017

is

seizure of its articles for publication and other materials, are


declared unconstitutional because there was no clear and present
danger of a substantive evil that the state has a right to prevent.
3.) It is not proper to implead President Arroyo as respondent.
Settled is the doctrine that the President, during his tenure of office
or actual incumbency, may not be sued in any civil or criminal case,
and there is no need to provide for it in the Constitution or law.
4.) This Court adopted the direct injury test in our jurisdiction. In
People v. Vera, it held that the person who impugns the validity of a
statute must have a personal and substantial interest in the case
such that he has sustained, or will sustain direct injury as a result.
Therefore, the court ruled that the petitioners have a locus standi,
for they suffered direct injury resulting from illegal arrest and
unlawful search committed by police operatives pursuant to PP
1017.

3.) Whether or not proper to implead President Gloria Macapagal


Arroyo as respondent in the petitions?

5.) Under Article XII Section 17 of the 1987 Philippine Constitution,


in times of national emergency, when the public interest so
requires, the President may temporarily take over a privately owned
public utility or business affected with public interest only if there is
congressional authority or approval. There must enactment of
appropriate legislation prescribing the terms and conditions under
which the

4.) Whether or not the petitioners have a legal standing in


questioning the constitutionality of the proclamation?

President may exercise the powers that will serves as the best
assurance that due process of law would be observed.

2.) Whether or not the warantless arrest of Randolf S. David and


Ronald Llamas and the dispersal of KMU and NAFLU-KMU members
during rallies were valid?

5.) Whether or not the concurrence of Congress is necessary


whenever the alarming powers incident to Martial Law are used?

Section 5

Ruling:
1.) The Court finds and so holds that PP 1017 is constitutional
insofar as it constitutes a call by the President for the AFP to
prevent or suppress lawless violence whenever becomes necessary
as prescribe under Section 18, Article VII of the Constitution.
However, there were extraneous provisions giving the President
express or implied power

AMERICAN BIBLE vs. CITY OF


MANILA
101 PHIL. 386 (1957)

(A) To issue decrees; (" Legislative power is peculiarly within the


province of the Legislature. Section 1,
Article VI categorically states that "[t]he legislative power shall be
vested in the Congress of the
Philippines which shall consist of a Senate and a House of
Representatives.")
(B) To direct the AFP to enforce obedience to all laws even those
not related to lawless violence as well as decrees promulgated
by the President[The absence of a law defining "acts of
terrorism" may result in abuse and oppression on the part of the
police or military]; and
(C) To impose standards on media or any form of prior restraint on
the press, are ultra vires and unconstitutional. The Court also
rules that under Section 17, Article XII of the Constitution, the
President, in the absence of legislative legislation, cannot take
over privately-owned public utility and private business affected
with public interest. Therefore, the PP No. 1017 is only partly
unconstitutional.
2.) The warrantless arrest of Randolf S. David and Ronald Llamas;
the dispersal and warrantless arrest of the KMU and NAFLU-KMU
members during their rallies are illegal, in the absence of proof that
these petitioners were committing acts constituting lawless
violence, invasion or rebellion and violating BP 880; the imposition
of standards on media or any form of prior restraint on the press, as
well as the warrantless search of the Tribune offices and whimsical

Facts: American Bible was a religious corporation, selling bibles


and other religious articles. They were required to obtain a license
and pay the corresponding fee for being engaged in the sale of
merchandise.
Ruling: The Court ruled that the Ordinance 2925 violated freedom
of religion. The government cannot impose some form of tax on
somebody for the exercise of his religion. American Bible was a
missionary organization which sells bibles as part of propagating
their faith. It was only making a small profit from the sale of bibles.
If they were making a lot of profit, then that would be the time that
the government should impose a tax on them since it would not
anymore be part of propagating the faith.

Tolentino
Finance

vs

Secretary

of

Facts:
Tolentino et al is questioning the constitutionality of RA 7716
otherwise known as the Expanded Value

Added Tax (EVAT) Law. Tolentino averred that this revenue bill did
After all, the determination of whether a certain ritual is or is not a
not exclusively originate from the House of Representatives as
religious ceremony must rest with the courts. It cannot be left to a
required by Section 24, Article 6 of the Constitution. Even though
religious group or sect, much less to a follower of said group or
RA 7716 originated as HB 11197 and that it passed the 3 readings
sect; otherwise, there would be confusion and misunderstanding for
in the HoR, the same did not complete the 3 readings in Senate for
there might be as many interpretations and meanings to be given
after the 1st reading it was referred to the Senate Ways & Means
to a certain ritual or ceremony as there are religious groups or sects
Committee thereafter Senate passed its own version known as
or followers.
Senate Bill 1630. Tolentino averred that what Senate could have
done is amend HB 11197 by striking out its text and substituting it 1. The freedom of religious belief guaranteed by the Constitution does
not and cannot mean exemption form or non-compliance with
w/ the text of SB 1630 in that way the bill remains a House Bill and
reasonable and non-discriminatory laws, rules and regulations
the Senate version just becomes the text (only the text) of the HB.
promulgated by competent authority. In enforcing the flag salute on
Tolentino and co-petitioner Roco [however] even signed the said
the petitioners, there was absolutely no compulsion involved, and
Senate Bill.
for their failure or refusal to obey school regulations about the flag
ISSUE: Whether or not EVAT originated in the HoR.
salute they were not being persecuted. Neither were they being
criminally prosecuted under threat of penal sacntion. If they chose
HELD: By a 9-6 vote, the SC rejected the challenge, holding that
not to obey the flag salute regulation, they merely lost the benefits
such consolidation was consistent with the power of the Senate to
of public education being maintained at the expense of their fellow
propose or concur with amendments to the version originated in the
citizens, nothing more. According to a popular expression, they
HoR. What the Constitution simply means, according to the 9
could take it or leave it. Having elected not to comply with the
justices, is that the initiative must come from the HoR. Note also
regulations about the flag salute, they forfeited their right to attend
that there were several instances before where Senate passed its
public schools.
own version rather than having the HoR version as far as revenue
and other such bills are concerned. This practice of amendment by 2. The Filipino flag is not an image that requires religious veneration;
substitution has always been accepted. The proposition of Tolentino
rather it is symbol of the Republic of the Philippines, of sovereignty,
concerns a mere matter of form. There is no showing that it would
an emblem of freedom, liberty and national unity; that the flag
make a significant difference if Senate were to adopt his over what
salute is not a religious ceremony but an act and profession of love
has been done.
and allegiance and pledge of loyalty to the fatherland which the
flag stands for; that by authority of the legislature, the Secretary of
Education was duly authorized to promulgate Department Order No.
8, series of 1955; that the requirement of observance of the flag
ceremony or salute provided for in said Department Order No. 8,
does not violate the Constitutional provision about freedom of
religion and exercise of religion; that compliance with the nondiscriminatory and reasonable rules and regulations and school
106 Phil 2 Aug. 12, 1959
discipline, including observance of the flag ceremony is a
FACTS:
prerequisite to attendance in public schools; and that for failure and
refusal to participate in the flag ceremony, petitioners were
1. Petitioners belong to the Jehovas Witness whose children were
properly excluded and dismissed from the public school they were
expelled from their schools when they refused to salute, sing the
attending.
anthem, recite the pledge during the conduct of flag ceremony.
DO No. 8 issued by DECS pursuant to RA 1265 which called for
the manner of conduct during a flag ceremony. The petitioners
wrote the Secretary of Education on their plight and requested to
reinstate their children. This was denied.

Gerona vs. Secretary

2.

As a result, the petitioners filed for a writ of preliminary


injunction against the Secretary and Director of Public Schools to
restrain them from implementing said DO No. 8.

3.

The lower court (RTC) declared DO 8 invalid and contrary to the


Bill of Rights.
ISSUE: Whether or not DO 8 is valid or constitutional
Ruling:

DO 8 is valid. Saluting the flag is not a religious ritual and it is for


the courts to determine, not a religious group, whether or not a
certain practice is one.
The court held that the flag is not an image but a symbol of the
Republic of the Philippines, an emblem of national sovereignty, of
national unity and cohesion and of freedom and liberty which it and
the Constitution guarantee and protect. Considering the complete
separation of church and state in our system of government, the
flag is utterly devoid of any religious significance. Saluting the flag
consequently does not involve any religious ceremony.

Ebralinag Division
Schools of Cebu

of

Supt.

G.R. No. 95770 March 1, 1993


Facts:
All the petitioners in these two cases were expelled from their
classes by the public school authorities in Cebu for refusing to
salute the flag, sing the national anthem and recite the patriotic
pledge as required by Republic Act No. 1265 and DECS Department
Order No. 8 which stipulated compulsory flag ceremonies in all
educational institutions.
Jehovah's Witnesses admittedly teach their children not to salute
the flag, sing the national anthem, and recite the patriotic pledge
for they believe that those are "acts of worship" or "religious
devotion" which they "cannot conscientiously give . . . to anyone or
anything except God". They consider the flag as an image or idol
representing the State.
They allege that the action of the local authorities in compelling the
flag salute and pledge transcends constitutional limitations on the

State's power and invades the sphere of the intellect and spirit
which the Constitution protects against official control. In requiring
school pupils to participate in the flag salute, the State thru the
Secretary of Education is not imposing a religion or religious belief
or a religious test on said students. It is merely enforcing a nondiscriminatory school regulation applicable to all alike.
Under the Administrative Code of 1987, Any teacher or student or
pupil who refuses to join or participate in the flag ceremony may be
dismissed after due investigation. In 1989, the DECS Regional Office
in Cebu received complaints about teachers and pupils belonging to
the Jehovah's Witnesses, and enrolled in various public and private
schools, who refused to sing the Philippine national anthem, salute
the Philippine flag and recite the patriotic pledge. Cebu school
officials resorted to a number of ways to persuade the children of
Jehovah's Witnesses to obey the memorandum.
In the Buenavista Elementary School, the children were asked to
sign an Agreement promising to sing the national anthem, place
their right hand on their breast until the end of the song and recite
the pledge of allegiance to the flag. However, things took a turn for
the worst. In the Daan Bantayan District, the District Supervisor,
Manuel F. Biongcog, ordered the "dropping from the rolls" of
students who "opted to follow their religious belief which is against
the Flag Salute Law" on the theory that "they forfeited their right to
attend public schools." 43 students were subsequently expelled
after refusing to sing. The petition in G.R. No. 95887 was filed by 25
students who were similarly expelled because Dr. Pablo Antopina,
who succeeded Susana Cabahug as Division Superintendent of
Schools, would not recall the expulsion orders of his predecessor.
Instead, he verbally caused the expulsion of some more children of
Jehovah's Witnesses. The petitioning students filed on account of
grave abuse of discretion on the part of the respondents in violating
their due process and their right to education. They alleged for the
nullity of the expulsion or dropping from the rolls of petitioners from
their respective schools, prohibiting respondents from further
barring the petitioners from their classes, and compelling the
respondent and all persons acting for him to admit and order the readmission of petitioners to their respective schools. They also
prayed for a TRO. On November 27, 1990, the Court issued a TRO
and a writ of preliminary mandatory injunction commanding the
respondents to immediately re-admit the petitioners to their
respective classes until further orders from this Court. The OSG
commented on the defense of the expulsion orders and claimed
that the flag salute was devoid of any religious significance and the
State had compelling interests to expel the children.
Issue:
Whether school children who are members of Jehovah's
Witnesses may be expelled from school (both public and private),
for refusing, on account of their religious beliefs, to take part in the
flag ceremony which includes playing or singing the Philippine
national anthem, saluting the Philippine flag and reciting the
patriotic pledge.

Held:
No. Religious freedom is a fundamental right which is entitled to the
highest priority and the amplest protection among human rights,
for it involves the relationship of man to his Creator. The right to
religious profession and worship has a two-fold aspect, vis.,
freedom to believe and freedom to act on one's belief. The first is
absolute as long as the belief is confined within the realm of
thought. The second is subject to regulation where the belief is
translated into external acts that affect the public welfare.

Petitioners stress, however, that while they do not take part in the
compulsory flag ceremony, they do not engage in "external acts" or
behavior that would offend their countrymen who believe in
expressing their love of country through the observance of the flag
ceremony.
The sole justification for a prior restraint or limitation on the
exercise of religious freedom is the existence of a grave and
present danger of a character both grave and imminent, of a
serious evil to public safety, public morals, public health or any
other legitimate public interest, that the State has a right and a
duty to prevent. We are not persuaded that by exempting the
Jehovah's Witnesses from saluting the flag, singing the national
anthem and reciting the patriotic pledge, this religious group which
admittedly comprises a "small portion of the school population" will
shake up our part of the globe and suddenly produce a nation
"untaught and uninculcated in and unimbued with reverence for the
flag, patriotism, love of country and admiration for national heroes.
Expelling or banning the petitioners from Philippine schools will
bring about the very situation that this Court had feared in Gerona.
Forcing a small religious group, through the iron hand of the law, to
participate in a ceremony that violates their religious beliefs, will
hardly be conducive to love of country or respect for dully
constituted authorities.
Furthermore, let it be noted that coerced unity and loyalty even to
the country, assuming that such unity and loyalty can be attained
through coercion, is not a goal that is constitutionally obtainable at
the expense of religious liberty. A desirable end cannot be promoted
by prohibited means. Moreover, the expulsion of members of
Jehovah's Witnesses from the schools where they are enrolled will
violate their right as Philippine citizens, under the 1987
Constitution, to receive free education, for it is the duty of the State
to "protect and promote the right of all citizens to quality
education . . . and to make such education accessible to all. We
hold that a similar exemption may be accorded to the Jehovah's
Witnesses with regard to the observance of the flag ceremony out
of respect for their religious beliefs, however "bizarre" those beliefs
may seem to others. Nevertheless, their right not to participate in
the flag ceremony does not give them a right to disrupt such
patriotic exercises. Paraphrasing the warning cited by this Court in
Non vs. Dames II, while the highest regard must be afforded their
right to the free exercise of their religion, "this should not be taken
to mean that school authorities are powerless to discipline them" if
they should commit breaches of the peace by actions that offend
the sensibilities, both religious and patriotic, of other persons. If
they quietly stand at attention during the flag ceremony while their
classmates and teachers salute the flag, sing the national anthem
and recite the patriotic pledge, we do not see how such conduct
may possibly disturb the peace, or pose "a grave and present
danger of a serious evil to public safety, public morals, public health
or any other legitimate public interest that the State has a right
(and duty) to prevent.

Aglipay vs. Ruiz

Gr no. L-45459 March 13

1937
FACTS: The government appropriated P60T for design of new
stamps which commemorated the 33rd Eucharistic Congress in
Manila. The design was the Map of the Philippines. Monsignor
Aglipay petitioned to stop the making and sale of the stamps. He
reasoned that this was a violation of Section 5.

RULING: The court ruled that the purpose of the stamp was to
attract tourist to the Philippines, not for the purpose of promoting
religion. The benefiting by the Catholic Church was only incidental
to the main purpose or the principal effect. It was not intended to
be the primary beneficiary. The stamp emphasized Manila (as
shown by the design), not the event or a particular religion.

expressly disqualified by the election code, are eligible to run for


public office.

AUSTRIA vs. NLRC


312 SCRA 410 (1999)

Pamil vs Teleron
G.R. No. L-34854 November 20, 1978
Facts:
In 1971, Fr. Margarito Gonzaga, a priest, won the election for
mayoralty in Albuquerque, Bohol. He was also proclaimed as a
mayor therein. Pamil, a rival candidate file a quo warranto case
against Gonzaga questioning the eligibility of Gonzaga. He argued
that as provided for in the Revised Administrative Code; in no case
shall there be elected or appointed to a municipal office
ecclesiastics, soldiers in active service, persons receiving salaries or
compensation from provincial or national funds, or contractors for
public works of the municipality. In this case, the elected mayor is
a priest. However, Judge Teleron ruled that the Administrative Code
is repealed by the Election Code of 1971 which allowed the
prohibitions of the revised administrative code.

Issue: Whether or not the Revised Administrative Code is no longer


operative.
Held:
Decision is indecisive, the said law, in the deliberations of the court,
failed to obtain the majority vote of eight (8) which is needed in
order for this law to be binding upon the parties in this case. For
this, the petition must be granted and the decision of the lower
court reversed and set aside. Fr. Gonzaga is hereby ordered to
vacate the mayoralty position. It is also pointed out that how can
one who swore to serve the Churchs interest above all be in duty to
enforce state policies which at times may conflict with church
tenets. This is in violation of the separation of the church and state.
The Revised Administrative Code still stands because there is no
implied repeal.
Dissenting Opinion:
J. Teehankee The Comelec ruled that soldiers in active service and
persons receiving salaries or compensation from provincial or
national funds are obviously now allowed to run for a public
elective office because under Sec. 23 of the Election Code of 1971
every person holding a public appointive office or position,
including active members of the Armed Forces shall ipso facto
cease in their office or position on the date they file their
certificates of candidacy. This implies that they are no longer
disqualified from running for an elective office. The Comelec
further ruled that as to the two remaining categories formerly
banned under the Revised Administrative Code, ecclesiastics and
contractors for public works of the municipality are allowed to run
for municipal elective offices under the maxim, Inclusio unius est
exclusio alterius, they being not included in the enumeration of
persons ineligible under the New Election Code. The rule is that all
persons possessing the necessary qualifications, except those

Facts: Private respondent is a religious corporation duly organized


and existing under Philippine laws.
Petitioner is a pastor of the SDA until 1991, when his services are
terminated. Due to the assistance of Austria in collecting Pastor
Rodrigos debt, the latter harbored ill-feelings against petitioner. He
also had a quarrel with another pastor and was asked to answer for
non-remittance of church collection.
Petitioner received a letter of dismissal. NLRC ordered his
reinstatement but was vacated after appeal.
Issue: WON NLRC has jurisdiction to try and decide the complaint.
Ruling: The principle of separation of church and state finds no
application in this case. This case does not concern an ecclesiastical
or purely religious affair as to bar the state from taking cognizance
of the same. An ecclesiastical affair is one that concerns doctrine,
creed, or form of worship of the church, or the adoption and
enforcement within religious association of needful laws and
regulations for the government of the membership, and the power
of excluding from such association those deemed unworthy of
membership. What is involved us the relationship of the church as
an employer and the minister as an employee. The SDA cannot hide
behind the mantle of protection of the doctrine of separation of
church and state. Petitioner was terminated from service without
just or lawful cause. Having been illegally dismissed, petitioner is
entitled to reinstatement to his former position.

Estrada vs. Escritor

AM No. P-02-1651

June 22 2006 and August 4 2003


ESTRADA vs. ESCRITOR
408 SCRA 1 (2003)
Facts: Alejandro Estrada wrote to Judge Caoibes requesting for an
investigation of rumors that respondent Soledad Escritor, court
interpreter in said court, is living with a man not her husband. They
allegedly have a child of eighteen to twenty years old. He filed the
charge against Escritor as he believes that she is committing an
immoral act that tarnishes the image of the court, thus she should
not be allowed to remain employed therein as it might appear that
the court condones her act.
Judge Caoibes referred the letter to Escritor who stated
that there is no truth as to the veracity of the allegation and
challenged Estrada to appear in the open and prove his allegation
in the proper forum. Judge Caoibes set a preliminary conference.
Escritor moved for the inhibition of Judge Caoibes from hearing her
case to avoid suspicion and bias as she previously filed an
administrative complaint against him and said case was still
pending in the Office of the Court Administrator (OCA). Escritors
motion was denied. The preliminary conference proceeded with
both Estrada and Escritor in attendance. Respondent Escritor
testified that when she entered the judiciary in 1999, she was
already a widow, her husband having died in 1998. She admitted
that she has been living with Luciano without the benefit of
marriage for twenty years and that they have a son. But as a

member of the religious sect known as the Jehovahs Witnesses and


the Watch Tower and Bible Tract Society, their conjugal
arrangement is in conformity with their religious beliefs. In fact,
after ten years of living together, she executed a Declaration of
Pledging Faithfulness.
Escritors partner, Luciano, executed a similar pledge. Both
pledges were executed and signed by three witnesses. At the time
Escritor executed her pledge, her husband was still alive but living
with another woman. Luciano was likewise married at that time,
but had been separated in fact from his wife. During her testimony,
Escritor volunteered to present members of her congregation to
confirm the truthfulness of their Declarations of Pledging
Faithfulness, but Judge Caoibes deemed it unnecessary and
considered her identification of her signature and the signature of
Quilapio sufficient authentication of the documents.
Issue: WON respondents right to religious freedom was abridged
when she was held administratively liable for having illicit relations.
Ruling: The Declaration of Pledging of Faithfulness executed by the
respondent and her mate greatly affect the administrative liability
of respondent.
Jehovahs Witnesses admit and recognize the
supremacy of the proper public authorities in the marriage
arrangement. Freedom of choice guarantees the liberty of the
religious conscience and prohibits any degree of compulsion or
burden, whether direct or indirect, in the practice of ones religion.
The Free Exercise Clause principally guarantees voluntarism,
although the Establishment Clause also assures voluntarism by
placing the burden of the advancement of religious groups on their
intrinsic merits and not on the support of the state. In interpreting
the Free Exercise Clause, the realm of belief poses no difficulty. The
realm of belief and creed is infinite and limitless bounded only by
ones imagination and thought.
So is the freedom of belief,
including religious belief, limitless and without bounds.
Court has ruled that government employees engaged in
illicit relations are guilty of disgraceful and immoral conduct for
which he/she may be held administratively liable. Respondent
Escritor does not claim that there is error in the settled
jurisprudence that an illicit relation constitutes disgraceful and
immoral conduct for which a government employee is held liable.
Nor is there an allegation that the norms of morality with respect to
illicit relations have shifted towards leniency from the time these
precedent cases were decided. The Court finds that there is no
such error or shift, thus the SC find no reason to deviate from these
rulings that such illicit relationship constitutes disgraceful and
immoral conduct punishable under the Civil Service Law.
Respondent having admitted the alleged immoral conduct, she, like
the respondents in the above-cited cases, could be held
administratively liable. However, there is a distinguishing factor
that sets the case at bar apart from the cited precedents, i.e., as a
defense, respondent invokes religious freedom since her religion,
the Jehovahs Witnesses, has, after thorough investigation, allowed
her conjugal arrangement with Luciano based on the churchs
religious beliefs and practices.
There is no doubt that choosing between keeping her
employment and abandoning her religious belief and practice and
family on the one hand, and giving up her employment and keeping
her religious practice and family on the other hand, puts a burden
on her free exercise of religion. The burden on respondent in the
case at bar is even greater as the price she has to pay for her
employment is not only her religious precept but also her family
which, by the Declaration Pledging Faithfulness, stands honorable
before God and men.However, even if the Court deems sufficient
respondents evidence on the sincerity of her religious belief and its
centrality in her faith, the case at bar cannot still be decided using
the compelling state interest test. The case at bar is one of first
impression, thus the parties were not aware of the burdens of
proof they should discharge in the Courts use of the compelling

state interest test. , it is inappropriate for the complainant, a


private person, to present evidence on the compelling interest of
the state. The burden of evidence should be discharged by the
proper agency of the government which is the Office of the Solicitor
General. To properly settle the issue in the case at bar, the
government should be given the opportunity to demonstrate the
compelling state interest it seeks to uphold in opposing the
respondents stance that her conjugal arrangement is not immoral
and punishable as it comes within the scope of free exercise
protection. Should the Court prohibit and punish her conduct where
it is protected by the Free Exercise Clause, the Courts action would
be an unconstitutional encroachment of her right to religious
freedom.
The SC cannot therefore simply take a passing look at
respondents claim of religious freedom, but must instead apply the
compelling state interest test. The government must be heard on
the issue as it has not been given an opportunity to discharge its
burden of demonstrating the states compelling interest which can
override respondents religious belief and practice. To repeat, this is
a case of first impression where we are applying the compelling
state interest test in a case involving purely religious conduct. The
careful application of the test is indispensable as how we will decide
the case will make a decisive difference in the life of the respondent
who stands not only before the Court but before her Jehovah God.
Dispositive portion: The case is REMANDED to the Office of the
Court Administrator. The Solicitor General is ordered to intervene in
the case where it will be given the opportunity (a) to examine the
sincerity and centrality of respondents claimed religious belief and
practice; (b) to present evidence on the states compelling
interest to override respondents religious belief and practice; and
(c) to show that the means the state adopts in pursuing its interest
is the least restrictive to respondents religious freedom.
ESTRADA vs. ESCRITOR
492 SCRA 1 (2006)
Facts: Respondent Soledad Escritor once again stands before the
Court invoking her religious freedom and her Jehovah God in a bid
to save her family united without the benefit of legal marriage and livelihood. The State, on the other hand, seeks to wield its
power to regulate her behavior and protect its interest in marriage
and family and the integrity of the courts where respondent is an
employee.
Issue: WON the compelling state interest test must be applied.
Ruling: The Constitution adheres to the benevolent neutrality
approach that gives room for accommodation of religious exercises
as required by the Free Exercise Clause. Thus, in arguing that
respondent should be held administratively liable as the
arrangement she had was illegal per se because, by universally
recognized standards, it is inherently or by its very nature bad,
improper, immoral and contrary to good conscience, the Solicitor
General failed to appreciate that benevolent neutrality could allow
for accommodation of morality based on religion, provided it does
not offend compelling state interests. Finally, even assuming that
the OSG has proved a compelling state interest, it has to further
demonstrate that the state has used the least intrusive means
possible so that the free exercise is not infringed any more than
necessary to achieve the legitimate goal of the state, i.e., it has
chosen a way to achieve its legitimate state end that imposes as
little as possible on religious liberties. Again, the Solicitor General
utterly failed to prove this element of the test. Other than the two
documents offered as cited above which established the sincerity of
respondents religious belief and the fact that the agreement was
an internal arrangement within respondents congregation, no iota

of evidence was offered. In fact, the records are bereft of even a


feeble attempt to procure any such evidence to show that the
means the state adopted in pursuing this compelling interest is the
least restrictive to respondents religious freedom.
Thus, the SC held that in this particular case and under
these distinct circumstances, respondent Escritors conjugal
arrangement cannot be penalized as she has made out a case for
exemption from the law based on her fundamental right to freedom
of religion. The Court recognizes that state interests must be
upheld in order that freedoms - including religious freedom - may
be enjoyed. In the area of religious exercise as a preferred
freedom, however, man stands accountable to an authority higher
than the state, and so the state interest sought to be upheld must
be so compelling that its violation will erode the very fabric of the
state that will also protect the freedom. In the absence of a
showing that such state interest exists, man must be allowed to
subscribe to the Infinite.

Victoriano vs Elizalde
FACTS: Benjamin victoriano a member of iglesia ni cristo had been
in the employ of the Elizalde Rope factory Inc since 1958. Her was a
member of elizalde rope workers union which had with the
company a CBA containing a closed shop provision which reads as
follow Membership union shall be required as a condition of
employment for all permanent employees worker covered by this
agreement. RA 3350 was enacted introducing an amendment to
paragraph (4) subsection (a) of section 4 of RA 875 as follows but
such agreement shall not cover members of any religious sect
which prohibit affiliation of their member in any such 0labor
organization Benjamin victoriano presents his resignation to
appellant union thereupon the union wrote a formal letter to
separate the appellee from the service in view of the fact that he
was resigning from the union as member of the company notified
the apellee and his counsel that unless the appellee could achieve a
satisfactory arrangement with the union the company would be
constrained to dismiss him from the service . this prompted
appellee to file an action for injunction to enjoin the company and
the union from dismissing apallee.
ISSUE: WON RA 3350 is unconstitutional
HELD:
the constitution provision only prohibits legislation for the support
of any religious tenets or the modes of worship of any sect, thus
forestalling compulsion by law of the acceptance of any creed or
the chosen form of religion within limits of utmost amplitude. RA
3350 does not require as a qualification on condition in joining any
lawful association membership in any particular religion on in any
religious sect neither does the act requires affiliation with a
religious sect that prohibits its member from joining a labor union
as a condition on qualification for withdrawing from labor union RA
3350 only exempts member with such religious affililiation from the
required to do a positive act to exercise the right to join or to
resign from the union. He is exempted from form the coverage of
any closed shop agreement that a labor union may have entered
into. Therefore RA 3350 is never an illegal evasion of constitutional
provision or prohibition to accomplish a desired result which is
lawful in itself by vering or following a legal way to do it.

KAPATIRAN SA MEAT AND


CANNING DIVISION (TUPAS
Local Chapter No. 1027),
petitioner, VS. THE
HONORABLE BLR DIRECTOR
PURA FERRER CALLEJA, MEAT AND
CANNING DIVISION UNIVERSAL ROBINA
CORPORATION and MEAT AND CANNING DIVISION NEW EMPLOYEES
AND WORKERS UNITED LABOR ORGANIZATION Gr no 82914 june
20 1998 Ruling:
The public respondent did not err in dismissing the petitioner's
appeal in BLR Case No. A-12-389-87. This Court's decision in
Victoriano vs. Elizalde Rope Workers' Union, 59 SCRA 54, upholding
the right of members of the IGLESIA NI KRISTO sect not to join a
labor union for being contrary to their religious beliefs, does not bar
the members of that sect from forming their own union. The public
respondent correctly observed that the "recognition of the tenets of
the sect ... should not infringe on the basic right of self-organization
granted by the constitution to workers, regardless of religious
affiliation."
The fact that TUPAS was able to negotiate a new CBA with ROBINA
within the 60-day freedom period of the existing CBA, does not
foreclose the right of the rival union, NEW ULO, to challenge TUPAS'
claim to majority status, by filing a timely petition for certification
election on October 13, 1987 before TUPAS' old CBA expired on
November 15, 1987 and before it signed a new CBA with the
company on December
3, 1987. As pointed out by Med-Arbiter Abdullah, a "certification
election is the best forum in ascertaining the majority status of the
contending unions wherein the workers themselves can freely
choose their bargaining representative thru secret ballot." Since it
has not been shown that this order is tainted with unfairness, this
Court will not thwart the holding of a certification election
(Associated Trade Unions [ATU] vs. Noriel, 88 SCRA 96).
WHEREFORE, the petition for certiorari is denied, with costs against
the petitioner.
SO ORDERED.

Free exercise clause:

Islamic Dawah Council of the


Philippines, Inc. vs. Executive
Secretary

G.R. No. 153888. July 9, 2003.

Facts:
Petitioner is a non-governmental organization that extends
voluntary services to the Filipino people, especially to Muslim
Communities. Petitioner began to issue, for a fee, halal
certifications to qualified products and food manufacturers on
account of the actual need to certify food products as halal and also
due to halal food producers' request. Subsequently, Executive Order
(EO) 46 was issued creating the Philippine Halal Certification
Scheme and designating respondent Office of Muslim Affairs (OMA)
to oversee its implementation. In this petition for prohibition,
petitioner alleged, among others, that the subject EO violates the
constitutional provision on the separation of Church and State.
In granting the petition, the Supreme Court ruled that freedom of
religion was accorded preferred status by the framers of the
fundamental law and it has consistently affirmed this preferred
status. Without doubt, classifying a food product as halal is a
religious function because the standards used are drawn from the
Qur'an and Islamic beliefs. By giving the OMA the exclusive power
to classify food products as halal, EO 46 encroached on the religious
freedom of Muslim organizations like herein petitioner to interpret
for Filipino Muslims what food products are fit for Muslim
consumption. Also, by arrogating to itself the task of issuing halal
certifications, the State has in effect forced Muslims to accept its
own interpretation of the Qur'an and Sunnah on halal food.
The Court further ruled that only the prevention of an immediate
and grave danger to the security and welfare of the community can
justify the infringement of religious freedom. In the case at bar, the
Court found no compelling justification for the government to
deprive Muslim organizations, like herein petitioner, of their
religious right to classify a product as halal, even on the premise
that the health of Muslim Filipinos can be effectively protected by
assigning to OMA the exclusive power to issue halal certificates.
Issue:
Whether or not Eexecutive Order 46 violates the
constitutional provision on the separation of Church and State.
Held:
No. In granting the petition, the Supreme Court ruled that
freedom of religion was accorded preferred status by the framers of
the fundamental law and it has consistently affirmed this preferred
status. Without doubt, classifying a food product as halal is a
religious function because the standards used are drawn from the
Qur'an and Islamic beliefs. By giving the OMA the exclusive power
to classify food products as halal, Executive Order 46 encroached
on the religious freedom of Muslim organizations like herein
petitioner to interpret for Filipino Muslims what food products are fit
for Muslim consumption. Also, by arrogating to itself the task of
issuing halal certifications, the State has in effect forced Muslims to
accept its own interpretation of the Qur'an and Sunnah on halal
food.
The Court further ruled that only the prevention of an immediate
and grave danger to the security and welfare of the community can
justify the infringement of religious freedom. In the case at bar, the
Court found no compelling justification for the government to
deprive Muslim organizations, like herein petitioner, of their
religious right to classify a product as halal, even on the premise
that the health of Muslim Filipinos can be effectively protected by
assigning to OMA the exclusive power to issue halal certificates.
Only the prevention of an immediate and grave danger to
the security and welfare of the community can justify the
infringement of religious freedom. If the government fails to show
the seriousness and immediacy of the threat, State intrusion is
constitutionally unacceptable. In a society with a democratic
framework like ours, the State must minimize its interference with

the affairs of its citizens and instead allow them to exercise


reasonable freedom of personal and religious activity. In the case at
bar, we find no compelling justification for the government to
deprive Muslim organizations, like herein petitioner, of their
religious right to classify a product as halal, even on the premise
that the health of Muslim Filipinos can be effectively protected by
assigning to OMA the exclusive power to issue halal certifications.
The protection and promotion of the Muslim Filipinos' right to health
are already provided for in existing laws and ministered to by
government agencies charged with ensuring that food products
released in the market are fit for human consumption, properly
labeled and safe. Unlike EO 46, these laws do not encroach on the
religious freedom of Muslims.

RE:
REQUEST OF MUSLIM
EMPLOYEES IN THE
DIFFERENT COURTS IN ILIGAN
CITY (RE: OFFICE HOURS)
477 SCRA 648
Facts: Several Muslim employees in the different courts in the said
city requested to Judge Salazar that they be allowed to enjoy
certain privileges during the Holy Month of Ramadhan. Judge
Salazar expressed his conformity with the first request, i.e., allowing
them to hold office from 7:30 a.m. to 3:30 p.m. without any break
during the month of Ramadan. However, he expressed some
misgivings about the second request, i.e., excusing them from work
from 10:00 a.m. to 2:00 p.m. every Friday during the entire
calendar year. In support of their requests, the Muslim employees
invoke PD 291 as amended by P.D. No. 322. The avowed purpose of
P.D. No. 291 was to reinforce national unity by recognizing Muslim
holidays and making them part of our national holidays.
Issue: WON the freedom to exercise ones religion was abridged in
this case.
Ruling: The Court recognizes that the observance of Ramadan and
the Friday Muslim Prayer Day is integral to the Islamic faith.
However, while the observance of Ramadan and allowing the
Muslim employees in the Judiciary to hold flexible office hours from
7:30 a.m. to 3:30 p.m. without any break during the month of
Ramadan finds support in Section 3 (a) of P.D. No. 291, as amended
by P.D. No. 322, there is no such basis to excuse them from work
from 10:00 a.m. to 2:00 p.m. every Friday, the Muslim Prayer Day,
during the entire calendar year.
On the other hand, the need of the State to prescribe
government office hours as well as to enforce them uniformly to all
civil servants, Christians and Muslims alike, cannot be disregarded.
Underlying Section 5, Rule XVII of the Omnibus Rules Implementing
Book V of E.O. No. 292 is the interest of the general public to be
assured of continuous government service during office hours every
Monday through Friday. The said rule enjoins all civil servants, of
whatever religious denomination, to render public service of no less
than eight hours a day or forty (40) hours a week.
To allow the Muslim employees in the Judiciary to be
excused from work from 10:00 a.m. to 2:00 p.m. every Friday
(Muslim Prayer Day) during the entire calendar year would mean a
diminution of the prescribed government working hours. For then,
they would be rendering service twelve (12) hours less than that
required by the civil service rules for each month.
Further, this
would encourage other religious denominations to request for
similar treatment. The performance of religious practices, whether

by the Muslim employees or those belonging to other religious


denominations, should not prejudice the courts and the public.
Indeed, the exercise of religious freedom does not exempt anyone
from compliance with reasonable requirements of the law, including
civil service laws.
Hence, the SC GRANTED the request to allow the Muslim
employees in the Judiciary to hold office hours from 7:30 a.m. to
3:30 p.m. without break during the month of Ramadan pursuant to
Section 3 (a) of Presidential Decree No. 291, as amended by
Presidential Decree No. 322.

Ang Ladlad vs COMELEC

Gr

no

190582 April 2010

thereafter were shipped to Mindanao specifically in Davao where


they were signed as laborers. Said women are inmates of the
houses of prostitution situated in Gardenia Street, in the district of
Sampaloc.
That when the petitioner filed for habeas corpus, the respondent
moved to dismiss the case saying that those women were already
out of their jurisdiction and that , it should be filed in the city of
Davao instead.
The court ruled in favor of the petitioner with the instructions;
For the respondents to have fulfilled the court's order, three

Facts:

optional courses were open: (1) They could have produced the

Comelec refused to recognize Ang Ladlad LGBT Party, an


organization composed of men and women who identify themselves
as lesbians, gays, bisexuals, or trans-gendered individuals
(LGBTs),as a party list based on moral grounds. In the elevation of
the case to the Supreme Court, Comelec alleged that petitioner
made misrepresentation in their application.
Issue: Whether or not Ang Ladlad LGBT Party qualifies for
registration as party-list.

bodies of the persons according to the command of the writ; or (2)


they could have shown by affidavit that on account of sickness or
infirmity those persons could not safely be brought before the
court; or (3) they could have presented affidavits to show that the
parties in question or their attorney waived the right to be present.
Issue:

The writ of Habeas Corpus was filed by the petitioner, with

the prayer that the respondent produce around 170 women whom
Justo Lukban et, al deported to Davao. Liberty of abode was also

Ruling:

raised versus the power of the executive of the Municipality in

Ang Ladlad LGBT Partys application for registration should be


granted.

deporting the women without their knowledge in his capacity as


Mayor.

Comelecs citation of the Bible and the Koran in denying petitioners


application was a violation of the non-establishment clause laid
down in Article 3 section 5 of the Constitution. The proscription by
law relative to acts against morality must be for a secular purpose
(that is, the conduct prohibited or sought to be repressed is
detrimental or dangerous to those conditions upon which depend
the existence and progress of human society"), rather than out of
religious conformity. The Comelec failed to substantiate their
allegation that allowing registration to Ladlad would be detrimental
to society.

Held:

The LGBT community is not exempted from the exercise of its


constitutionally vested rights on the basis of their sexual
orientation. Laws of general application should apply with equal
force to LGBTs, and they deserve to participate in the party-list
system on the same basis as other marginalized and
underrepresented sectors. Discrimination based on sexual
orientation is not tolerated ---not by our own laws nor by any
international laws to which we adhere.

under no restraint and that he, the official, had no jurisdiction over

The court concluded the case by granting the parties aggrieved the
sum of 400 pesos each, plus 100 pesos for nominal damage due to
contempt of court. Reasoning further that if the chief executive of
any municipality in the Philippines could forcibly and illegally take a
private citizen and place him beyond the boundaries of the
municipality, and then, when called upon to defend his official
action, could calmly fold his hands and claim that the person was
this other municipality.
We believe the true principle should be that, if the respondent is
within the jurisdiction of the court and has it in his power to obey
the order of the court and thus to undo the wrong that he has
inflicted, he should be compelled to do so. Even if the party to
whom the writ is addressed has illegally parted with the custody of
a person before the application for the writ is no reason why the
writ should not issue. If the mayor and the chief of police, acting
under no authority of law, could deport these women from the city

Section 6

of Manila to Davao, the same officials must necessarily have the


same

means

to

return

them

from

Davao

to

Manila.

The

respondents, within the reach of process, may not be permitted to

Villavicencio vs. Lukban


Facts:

restrain a fellow citizen of her liberty by forcing her to change her


domicile and to avow the act with impunity in the courts, while the
person who has lost her birthright of liberty has no effective
recourse. The great writ of liberty may not thus be easily evaded.

Justo Lukban as Manila City's Mayor together with Anton Hohmann,


the city's Chief of Police, took custody of about 170 women at the
night of October 25 beyond the latters consent and knowledge and

MARCOS vs. MANGLAPUS

177 SCRA 668 (1989)


Facts: Former President Marcos sought to compel the Secretary of
Foreign Affairs to issue him travel documents so that he could
return to the Philippines from his exile in Hawaii. He argued that the
decision of then Pres. Cory Aquino to bar his return violates his right
to travel as guaranteed by the Bill of Rights.
Note: There was no court order, nor was there a law limiting his
right to travel. TAU MU
Ruling 1: The Court held that the right to travel means only the
right to leave. It does not include the right to return to the country.
These are two distinct rights. The right to return is not protected by
the Bill of Rights but is protected by the Philippine Law by virtue of
the Principle of Incorporation. The principle of incorporation states
that generally accepted principles of international law is part of the
law of the land.
This right to return is part of those generally accepted principles.

If the right to return is protected by the Constitution, it can


only be restricted by law. In this case, there was no law.
If the right to return is not protected by the Bill of Rights
but is protected by the Bill of Rights but is protected by the
Philippine Law, who can restrict it?

Ruling 2: The Court ruled that there is no more need of a law. the
executive can limit it as part of the Presidents implied powers
(Doctrine of Residual Powers). This was how the SC justified the
power of Pres. Aquino in barring the return of former Pres. Marcos.
Note: Inherent power of the executive: (1) The power to do
anything not prohibited by the Constitution; and (2) The power to
do anything not prohibited by law.

Section 7

Coastal Road and Reclamation Project containing a total area of


1,915,894 square meters. Subsequently, the Register of Deeds of
the Municipality of Paraaque issued Transfer Certificates of Title in
the name of PEA, covering the three reclaimed islands known as the
Freedom Islands. In 1995, PEA entered into a Joint Venture
Agreement with AMARI, a private corporation, to develop the
Freedom Islands. The JVA also required the reclamation of an
additional 250 hectares of submerged areas surrounding these
islands to complete the configuration in the Master Development
Plan of the Southern Reclamation Project-MCCRRP. PEA and AMARI
entered into the JVA through negotiation without public bidding. The
Board of Directors of PEA, in its Resolution No. 1245, confirmed the
JVA. Thereafter, JVA was approved.
In 1996, then Senate President Maceda delivered a
privilege speech in the Senate and denounced the JVA as the
grandmother of all scams. The Senate Committees reported the
results of their investigation in Senate Committee Report No. 560
and among the conclusions of their report are: (1) the reclaimed
lands PEA seeks to transfer to AMARI under the JVA are lands of the
public domain which the government has not classified as alienable
lands and therefore PEA cannot alienate these lands; (2) the
certificates of title covering the Freedom Islands are thus void, and
(3) the JVA itself is illegal.
Hence, petitioner, as a taxpayer, filed the instant Petition
for Mandamus with Prayer for the Issuance of a Writ of Preliminary
Injunction and Temporary Restraining Order. Petitioner contends
the government stands to lose billions of pesos in the sale by PEA of
the reclaimed lands to AMARI.
Petitioner prays that PEA publicly disclose the terms of any
renegotiation of the JVA, invoking Section 28, Article II, and Section
7, Article III, of the 1987 Constitution on the right of the people to
information on matters of public concern. Petitioner assails the sale
to AMARI of lands of the public domain as a blatant violation of
Section 3, Article XII of the 1987 Constitution prohibiting the sale of
alienable lands of the public domain to private corporations. Finally,
petitioner asserts that he seeks to enjoin the loss of billions of
pesos in properties of the State that are of public dominion.
Issue:

CHAVEZ vs. PEA Gr no 133258 july 3 2002

WON the constitutional right to information includes official


information on on-going negotiations before a final agreement.

384 SCRA 152


Facts:
The government, through the Commissioner of Public Highways,
signed a contract with the Construction and Development
Corporation of the Philippines to reclaim certain foreshore and
offshore areas of Manila Bay. The contract also included the
construction of Phases I and II of the Manila-Cavite Coastal Road.
CDCP obligated itself to carry out all the works in consideration of
fifty percent of the total reclaimed land. In 1977, then President
Marcos issued Presidential Decree No. 1084 creating PEA. PD No.
1084 tasked PEA to reclaim land, including foreshore and
submerged areas, and to develop, improve, acquire, x x x lease
and sell any and all kinds of lands. On the same date, then
President Marcos issued Presidential Decree No. 1085 transferring
to PEA the lands reclaimed in the foreshore and offshore of the
Manila Bay under the Manila-Cavite Coastal Road and Reclamation
Project.
In 1981, then President Marcos issued a memorandum
directing PEA to amend its contract with CDCP. In 1988, President
Aquino issued Special Patent No. 3517, granting and transferring to
PEA the parcels of land so reclaimed under the Manila-Cavite

Ruling:
Before the consummation of the contract, PEA must, on its own and
without demand from anyone, disclose to the public matters
relating to the disposition of its property. These include the size,
location, technical description and nature of the property being
disposed of, the terms and conditions of the disposition, the parties
qualified to bid, the minimum price and similar information. PEA
must prepare all these data and disclose them to the public at the
start of the disposition process, long before the consummation of
the contract, because the Government Auditing Code requires
public bidding. If PEA fails to make this disclosure, any citizen can
demand from PEA this information at any time during the bidding
process.
Information, however, on on-going evaluation or review of
bids or proposals being undertaken by the bidding or review
committee is not immediately accessible under the right to
information. While the evaluation or review is still on-going, there
are no official acts, transactions, or decisions on the bids or

proposals.
However, once the committee makes its official
recommendation, there arises a definite proposition on the part of
the government.
From this moment, the publics right to
information attaches, and any citizen can access all the nonproprietary information leading to such definite proposition.

agencies or officials. The third category refers to research data,


whether raw, collated or processed, owned by the government and
used in formulating government policies.

A consummated contract is not a requirement for the


exercise of the right to information. Otherwise, the people can
never exercise the right if no contract is consummated, and if one is
consummated, it may be too late for the public to expose its
defects. Requiring a consummated contract will keep the public in
the dark until the contract, which may be grossly disadvantageous
to the government or even illegal, becomes a fait accompli. This
negates the State policy of full transparency on matters of public
concern, a situation which the framers of the Constitution could not
have intended. Such a requirement will prevent the citizenry from
participating in the public discussion of any proposed contract,
effectively truncating a basic right enshrined in the Bill of Rights.
The Court can allow neither an emasculation of a constitutional
right, nor a retreat by the State of its avowed policy of full
disclosure of all its transactions involving public interest.
The information that petitioner may access on the
renegotiation
of
the
JVA
includes
evaluation
reports,
recommendations, legal and expert opinions, minutes of meetings,
terms of reference and other documents attached to such reports or
minutes, all relating to the JVA. However, the right to information
does not compel PEA to prepare lists, abstracts, summaries and the
like relating to the renegotiation of the JVA. The right only affords
access to records, documents and papers, which means the
opportunity to inspect and copy them. One who exercises the right
must copy the records, documents and papers at his expense. The
exercise of the right is also subject to reasonable regulations to
protect the integrity of the public records and to minimize
disruption to government operations, like rules specifying when and
how to conduct the inspection and copying.

The State policy of full transparency in all transactions


involving public interest reinforces the peoples right to information
on matters of public concern.
This State policy is expressed in
Section 28, Article II of the Constitution, thus: Subject to reasonable
conditions prescribed by law, the State adopts and implements a
policy of full public disclosure of all its transactions involving public
interest.
These twin provisions of the Constitution seek to promote
transparency in policy-making and in the operations of the
government, as well as provide the people sufficient information to
exercise effectively other constitutional rights.
These twin
provisions are essential to the exercise of freedom of expression. If
the government does not disclose its official acts, transactions and
decisions to citizens, whatever citizens say, even if expressed
without any restraint, will be speculative and amount to nothing.
These twin provisions are also essential to hold public officials at
all times accountable to the people, for unless citizens have the
proper information, they cannot hold public officials accountable for
anything.
Armed with the right information, citizens can
participate in public discussions leading to the formulation of
government policies and their effective implementation.
An
informed citizenry is essential to the existence and proper
functioning of any democracy.

RA 6713 IRR Sec. 3 Rule 4

There is no claim by PEA that the information demanded


by petitioner is privileged information rooted in the separation of
powers. The information does not cover Presidential conversations,
correspondences, or discussions during closed-door Cabinet
meetings which, like internal deliberations of the Supreme Court
and other collegiate courts, or executive sessions of either house of
Congress, are recognized as confidential. This kind of information
(a)
cannot be pried open by a co-equal branch of government.

Rule IV
Transparency of Transaction
and Access to Information
Section 3. Every department, office or agency shall provide official
information, records or documents to any requesting public,
except if:
such information, record or document must be kept secret in the
interest of national defense or security or the conduct of foreign
affairs;

Hence, the SC ruled that the constitutional right to


information includes official information on on-going negotiations
before a final contract. The information, however, must constitute
(b) such disclosure would put the life and safety of an individual in
definite propositions by the government and should not cover
imminent danger;
recognized exceptions like privileged information, military and
diplomatic secrets and similar matters affecting national security
and public order.
(c) the information, record or document sought falls within the
concepts of established privilege or recognized exceptions as may
be provided by law or settled policy or jurisprudence;
Note:
The right covers three categories of information which are matters
of public concern, namely: (1) official records; (2) documents and (d) such information, record or document comprises drafts
decisions, orders, rulings, policy decisions, memoranda, etc.;
papers pertaining to official acts, transactions and decisions; and
(3) government research data used in formulating policies.
The first category refers to any document that is part of the public
records in the custody of government agencies or officials. The
second category refers to documents and papers recording,
evidencing, establishing, confirming, supporting, justifying or
explaining official acts, transactions or decisions of government

of

(e) it would disclose information of a personal nature where disclosure


would constitute a clearly unwarranted invasion of personal
privacy;

(f) it would disclose investigatory records compiled for law


enforcement purposes or information which if written would be
contained in such records, but only to the extent that the
production of such records or information would (i) interfere with
enforcement proceedings, (ii) deprive a person of a right to a fair
trial or an impartial adjudication, (iii) disclose the identity of a
confidential source and in the case of a record compiled by a
criminal law enforcement authority in the course of a criminal
investigation, or by an agency conducting a lawful national
security intelligence investigation, confidential information
furnished only by the confidential source, or (iv) unjustifiably
disclose investigative techniques and procedures; or

(g) it would disclose information the premature disclosure of which


would (i) in the case of a department, office or agency which
agency regulates currencies, securities, commodities, or financial
institutions, be likely to lead to significant financial speculation in
currencies, securities, or commodities, or significantly endanger
the stability of any financial institution; or (ii) in the case of any
department, office or agency be likely or significantly to frustrate
implementation of a proposed official action, except that
subparagraph (f) (ii) shall not apply in any instance where the
department, office or agency has already disclosed to the public
the content or nature of its proposed action, or where the
department, office or agency is required by law to make such
disclosure on its own initiative prior to taking final official action
on such proposal.

Section 8
P.O. 180. Organizations Must be reg. with CSC and DOLE
III. Registration of Employees Organization
Section 7. Government employees organizations shall register
with the Civil Service Commission and the Department of Labor
and Employment. The application shall be filed with the Bureau of
Labor Relations of the Department which shall process the same in
accordance with the provisions of the Labor Code of the
Philippines, as amended. Applications may also be filed with the
Regional Offices of the Department of Labor and Employment
which shall immediately transmit the said applications to the
Bureau of Labor Relations within three (3) days from receipt
thereof.

Section 8. Upon approval of the application, a registration


certificate shall be issued to the organization recognizing it as a
legitimate employees organization with the right to represent its
members and undertaken activities to further and defend its
interests. The corresponding certificates of registration shall be
jointly approved by the Chairman of the Civil Service Commission
and Secretary of Labor and Employment.

Section 9
SUMULONG vs. GUERRERRO
154 SCRA 461 (1987)

Facts: The NHA wanted to use Sumulongs property for socialized


housing for the lower and middle class. The owner contended that
socialized housing is not public use because not everyone can
benefit from this, only the handful of people who to be given the
houses.

Held: The court held that the socialized housing is within the
context of public use. Public use has acquired a more
comprehensive meaning. That is whatever would result to indirect
public benefit or welfare is also public use. It also ruled that it will
benefit everyone in the sense that it will affect the safety, health
and environment. Providing housing to these people will help in
lessening the incidence of violence and problems concerning
health. In the end, it will benefit everybody in a way. In short,
socialize housing falls within the meaning of public use.

Association
of
Landowners vs DAR

Small

FACTS:
These are consolidated cases involving common legal questions
including serious challenges to the constitutionality of R.A. No. 6657
also known as the "Comprehensive Agrarian Reform Law of 1988"
In G.R. No. 79777, the petitioners are questioning the P.D No. 27
and E.O Nos. 228 and 229 on the grounds inter alia of separation of
powers, due process, equal protection and the constitutional
limitation that no private property shall be taken for public use
without just compensation.
In G.R. No. 79310, the petitioners in this case claim that the power
to provide for a Comprehensive
Agrarian Reform Program as decreed by the Constitution belongs to
the Congress and not to the President, the also allege that
Proclamation No. 131 and E.O No. 229 should be annulled for
violation of the constitutional provisions on just compensation, due
process and equal protection. They contended that the taking must
be simultaneous with payment of just compensation which such
payment is not contemplated in Section 5 of the E.O No. 229.
In G.R. No. 79744, the petitioner argues that E.O Nos. 228 and 229
were invalidly issued by the President and that the said executive
orders violate the constitutional provision that no private property
shall be taken without due process or just compensation which was
denied to the petitioners.
In G.R. No 78742 the petitioners claim that they cannot eject their
tenants and so are unable to enjoy their right of retention because
the Department of Agrarian Reform has so far not issued the
implementing rules of the decree. They therefore ask the Honorable
Court for a writ of mandamus to compel the respondents to issue
the said rules.
ISSUE:
Whether or not the laws being challenged is a valid exercise of
Police power or Power of Eminent Domain.
RULING:
Police Power through the Power of Eminent Domain, though there
are traditional distinction between the police power and the power
of eminent domain, property condemned under police power is
noxious or intended for noxious purpose, the compensation for the
taking of such property is not subject to compensation, unlike the
taking of the property in Eminent Domain or the power of

expropriation which requires the payment of just compensation to


the owner of the property expropriated.

CITY
GOVERNMENT
OF
QUEZON CITY vs. HON. JUDGE
VICENTE G. ERICTA
Police Power Not Validly Exercised
FACTS:
Quezon City enacted an ordinance
entitled ORDINANCE
REGULATING THE ESTABLISHMENT,
MAINTENANCE AND OPERATION OF PRIVATE MEMORIAL TYPE
CEMETERY OR BURIAL GROUND WITHIN THE JURISDICTION OF
QUEZON CITY AND PROVIDING PENALTIES FOR THE VIOLATION
THEREOF. The law basically provides that at least six (6) percent
of the total area of the memorial park cemetery shall be set aside
for charity burial of deceased persons who are paupers and have
been residents of Quezon City for at least 5 years prior to their
death, to be determined by competent City Authorities. QC justified
the law by invoking police power.
ISSUE: Whether or not the ordinance is valid.
HELD:
The SC held the law as an invalid exercise of police power. There is
no reasonable relation between the setting aside of at least six (6)
percent of the total area of all private cemeteries for charity burial
grounds of deceased paupers and the promotion of health, morals,
good order, safety, or the general welfare of the people. The
ordinance is actually a taking without compensation of a certain
area from a private cemetery to benefit paupers who are charges of
the municipal corporation. Instead of building or maintaining a
public cemetery for this purpose, the city passes the burden to
private cemeteries.

Vda de. Ouano vs. City of


Cebu Gr no 168770 Feb 9 2011
Expropriation; abandonment of public purpose.
In this case, the Mactan Cebu International Airport Authority
(MCIAA) and/or its predecessor agency had not actually used the
lots subject of the final decree of expropriation in Civil Case No. R1881 for the purpose they were originally taken by the government,
i.e., for the expansion and development of Lahug Airport. In fact,
the Lahug Airport had been closed and abandoned.
Also, in this case, it was preponderantly established by evidence
that the National Airport Corporation, MCIAAs predecessor, through
its team of negotiators, had given assurance to the affected
landowners that they would be entitled to repurchase their
respective lots in the event they are no longer used for airport
purposes.

The SC held that the government acquires only such rights in


expropriated parcels of land as may be allowed by the character of
its title over the properties. This means that in the event the
particular public use for which a parcel of land is expropriated is
abandoned, the owner shall not be entitled to recover or repurchase
it as a matter of right, unless such recovery or repurchase is
expressed in or irresistibly deducible from the condemnation
judgment.
The SC held that the decision in Civil Case No. R-1881 enjoined
MCIAA, as a condition of approving expropriation, to allow recovery
or repurchase upon abandonment of the Lahug airport project. In
effect, the government merely held the properties condemned in
trust until the proposed public use or purpose for which the lots
were condemned was actually consummated by the government.
Since the government failed to perform the obligation that is the
basis of the transfer of the property, then the lot owners can
demand the reconveyance of their old properties after the payment
of the condemnation price. A condemnor should commit to use the
property pursuant to the purpose stated in the petition for
expropriation, failing which it should file another petition for the
new purpose. If not, then it behooves the condemnor to return the
said property to its private owner, if the latter so desires. The
government cannot plausibly keep the property it expropriated in
any manner it pleases and, in the process, dishonor the judgment
of expropriation. Anunciacion Vda. De Ouano, et al. v.
Republic of the Philippines, et al./Mactan-Cebu International
Airport [MCIAA] v. Ricardo L. Inocian, in his personal
capacity and as Attorney-in-Fact of Olympia E. Esteves, et
al. and Aletha Suico Magat in her personal capacity and as
Attorney-in-Fact of Philip M. Suico, et al. G.R. Nos. 168770 &
168812, February 9, 2011.
Expropriation; reconveyance of expropriated property. In
accordance with Art. 1187 of the Civil Code on mutual
compensation, MCIAA may keep whatever income or fruits it may
have obtained from the parcels of land expropriated. In turn, the
landowners need not require the accounting of interests earned by
the amounts they received as just compensation.
Following Art. 1189 of the Civil Code providing that if the thing is
improved by its nature, or by time, the improvement shall inure to
the benefit of the creditor, the landowners do not have to settle the
appreciation of the values of their respective lots as part of the
reconveyance process, since the value increase is merely the
natural effect of nature and time. Anunciacion Vda. De Ouano,
et al. v. Republic of the Philippines, et al./Mactan-Cebu
International Airport [MCIAA] v. Ricardo L. Inocian, in his
personal capacity and as Attorney-in-Fact of Olympia E.
Esteves, et al. and Aletha Suico Magat in her personal
capacity and as Attorney-in-Fact of Philip M. Suico, et al.
G.R. Nos. 168770 & 168812, February 9, 2011.

Section 10
Clements vs Nolting Gr no L17959 Jan 24 1922
Ruling:
We submit that the mere statement of the results which must flow
from the recognition of the principle contended for by the

respondent, and involved in a denial of the plaintiff's claim, is


sufficient to refute every argument which may be advanced to
support it. Plaintiff, and the hundreds of teachers and other
employees of the Insular Government affected by the depreciation
of the Philippine paper peso, are merely asking for fair treatment,
for an honest compliance on the part of the Government with its
part of the agreement. We do not doubt that, as a matter of fact,
the defendant herein and every responsible official of the Philippine
Government recognizes the justice of the plaintiff's contention, and
that the necessity for this rule has arisen from an apprehension lest
their natural tendency to do what they know to be right and fair
may constitute a technical violation of the law.
The contention on the part of the respondent that the Philippine
paper peso is a legal tender for the payment of a contract debt,
when some other specie has not been provided, is not tenable for
the reason that it violates the terms of the express contracts
A contract to pay a certain sum in money, without any stipulation as
to the kind of money in which it shall be paid, may always be
satisfied by payment of that sum in any currency which is lawful
money at the place and time at which payment is to be made. That
is the general rule, under both the common and the civil law. But
when the contract stipulates the specie or kind or character of
money for the performance of the contract, it must be satisfied in
the medium of payment mentioned in the contract.
That doctrine is established and affirmed by the law in force in the
Philippine Islands. The Civil Code, still in force in the Philippine
Islands, by article 1170, provides expressly that "payments of debts
of money shall be made in the specie stipulated and, should it not
be possible to deliver such specie, in silver or gold coin legally
current in Spain." Article 1754 of the Civil Code provides that the
obligations of persons who borrow money shall be governed by the
provisions of said article 1170 of the same Code. (Serrales vs. Esbri,
200 U. S., 103; City of San Juan vs. St. John's Gas Co., 195 U. S.,
510.)
Contracts are made for things, not names or sounds, and the
obligation of the contract arises from its terms and the means
which the law affords for its enforcement. Under the Civil Code the
contract constitutes the law of the parties unless it violates some
provision of law or public policy. The parties themselves make the
law by which they shall be governed, and it is the business of the
courts to see that the parties to a legal contract comply with its
terms. A law which changes the terms of a legal contract between
parties, either in the time or mode of performance, or imposes new
conditions, or dispenses with those expressed, or authorizes for its
satisfaction something different from that provided in its terms, is
law which impairs the obligation of a contract and is therefore null
and void. An interference with the terms of a legal contract by
legislation is unwarranted and illegal. A contract is not fulfilled by
the delivery of one thing which is different from the thing the
contract provides for. Words in contracts are to be given the
meaning which they were understood to have by the parties at the
time of the making of the contract. There cannot exist in this
jurisdiction one law for debtors and another law for creditors. The
genius, the nature, and the spirit of our Government amount to a
prohibition of such acts of legislation, and the general principles of
law and reason forbid them.
The Legislature may enjoin, permit, forbid, and punish; it may
declare new crimes and establish rules of conduct for all its citizens
in future cases; it may command what is right and forbid what is
wrong, but it cannot change innocence into guilt and punish
innocence as a crime, or violate the rights of an antecedent lawful
private contract or the right of private property. (Calder vs. Bul, 3
Dallas, 388.)

The fundamental maxims of a free government seem to require that


the rights of personal liberty and private property should be held
sacred, and that includes contractual rights. (Wilkinson vs. Leland, 2
Peters, 657.)
It would be ruinous to the commercial interests of the Philippine
Islands to declare that the payment of debts of money could be
made in other specie than that stipulated in the contract.
For all of the foregoing facts and the law, we are fully persuaded
that the remedy prayed for should be, and is hereby, granted. And
it is hereby ordered and decreed that the writ of mandamus be
issued to the defendant herein, commanding him to countersign, or
cause to the countersigned the original of the warrant set forth in
paragraph 9 of the complaint, and to deliver the same to the
plaintiff so that he may present it to the Treasurer of the Philippine
Islands and receive payment of said sum of P73.33 due him as
averred in the complaint; and without any finding as to costs. So
ordered.

Oposa vs factoran Sec 10 applies on TLA


Facts:
Principal petitioners, are all minors duly represented and joined by
their respective parents. Impleaded as an additional plaintiff is the
Philippine Ecological Network, Inc. (PENI), a domestic, non-stock
and nonprofit corporation organized for the purpose of, inter alia,
engaging in concerted action geared for the protection of our
environment and natural resources. The original defendant was the
Honorable Fulgencio S. Factoran, Jr., then Secretary of the
Department of Environment and Natural Resources (DENR). His
substitution in this petition by the new Secretary, the Honorable
Angel C. Alcala, was subsequently ordered upon proper motion by
the petitioners. The complaint was instituted as a taxpayers'
classsuit and alleges that the plaintiffs "are all citizens of the
Republic of the Philippines, taxpayers, and entitled to the full
benefit, use and enjoyment of the natural resource treasure that is
the country's virgin tropical forests." The same was filed for
themselves and others who are equally concerned about the
preservation of said resource but are "so numerous that it is
impracticable to bring them all before the Court."
On 22 June 1990, the original defendant, Secretary Factoran, Jr.,
filed a Motion to Dismiss the complaint based on two grounds,
namely: the plaintiffs have no cause of action against him and, the
issue raised by the plaintiffs is a political question which properly
pertains to the legislative or executive branches of Government. In
their 12 July 1990 Opposition to the Motion, the petitioners maintain
that, the complaint shows a clear and unmistakable cause of action,
the motion is dilatory and the action presents a justiciable question
as it involves the defendant's abuse of discretion.
On 18 July 1991, respondent Judge issued an order granting the
aforementioned motion to dismiss. In the said order, not only was
thedefendant's claim that the complaint states no cause of action
against him and that it raises a political question sustained, the
respondent Judge further ruled that the granting
of the relief prayed for would result in the impairment of contracts
which is prohibited by the fundamental law of the land.
Plaintiffs thus filed the instant special civil action for certiorari under
Rule 65 of the Revised Rules of

Court and ask this Court to rescind and set aside the dismissal order
on the ground that the respondent Judge gravely abused his
discretion in dismissing the action. Again, the parents of the
plaintiffs-minors not only represent their children, but have also
joined the latter in this case.
Petitioners contend that the complaint clearly and unmistakably
states a cause of action as it contains sufficient allegations
concerning their right to a sound environment based on Articles 19,
20 and 21 of the Civil Code (Human Relations), Section 4 of
Executive Order (E.O.) No. 192 creating the DENR, Section 3 of
Presidential Decree (P.D.) No. 1151 (Philippine Environmental
Policy), Section 16, Article II of the 1987 Constitution recognizing
the right of the people to a balanced and healthful ecology, the
concept of generational genocide in Criminal Law and the concept
of man's inalienable right to self-preservation and self-perpetuation
embodied in natural law. Petitioners likewise rely on the
respondent's correlative obligation per Section 4 of E.O. No. 192, to
safeguard the people's right to a healthful environment.
It is further claimed that the issue of the respondent Secretary's
alleged grave abuse of discretion in granting Timber License
Agreements (TLAs) to cover more areas for logging than what is
available involves a judicial question.
Anent the invocation by the respondent Judge of the Constitution's
non-impairment clause, petitioners maintain that the same does not
apply in this case because TLAs are not contracts. They likewise
submit that even if TLAs may be considered protected by the said
clause, it is well settled that they may still be revoked by the State
when the public interest so requires.
Issues:
(1) Whether or not the petitioners have locus standi.
(2) Whether or not the petiton is in a form of a class suit.
(3) Whether or not the TLAs can be out rightly cancelled.
(4) Whether or not the petition should be dismissed.

Held:
As to the matter of the cancellation of the TLAs, respondents
submit that the same cannot be done by the State without due
process of law. Once issued, a TLA remains effective for a certain
period of time usually for twenty-five (25) years. During its
effectivity, the same can neither be revised nor cancelled unless
the holder has been found, after due notice and hearing, to have
violated the terms of the agreement or other forestry laws and
regulations. Petitioners' proposition to have all the TLAs
indiscriminately cancelled without the requisite hearing would be
violative of the requirements of due process.
The subject matter of the complaint is of common and general
interest not just to several, but to all citizens of the Philippines.
Consequently, since the parties are so numerous, it, becomes
impracticable, if not totally impossible, to bring all of them before
the court. The plaintiffs therein are numerous and representative
enough to ensure the full protection of all concerned interests.
Hence, all the requisites for the filing of a valid class suit under
Section 12, Rule 3 of the Revised Rules of Court are present both in
the said civil case and in the instant petition, the latter being but an
incident to the former.

Petitioners minors assert that they represent their generation as


well as generations yet unborn. Their personality to sue in behalf of
the succeeding generations can only be based on the concept of
intergenerational responsibility insofar as the right to a balanced
and healthful ecology is concerned. Nature means the created
world in its entirety. Every generation has a responsibility to the
next to preserve that rhythm and harmony for the full enjoyment of
a balanced and healthful ecology. The minors' assertion of their
right to a sound environment constitutes, at the same time, the
performance of their obligation to ensure the protection of that
right for the generations to come.
The complaint focuses on one specific fundamental legal right the
right to a balanced and healthful ecology which, for the first time in
our nation's constitutional history, is solemnly incorporated in the
fundamental law. Section 16, Article II of the 1987 Constitution.
While the right to a balanced and healthful ecology is to be found
under the Declaration of Principles and State Policies and not under
the Bill of Rights, it does not follow that it is less important than any
of the civil and political rights enumerated in the latter. Such a right
belongs to a different category of rights altogether for it concerns
nothing less than self-preservation and self-perpetuation aptly
and fittingly stressed by the petitioners the advancement of which
may even be said to predate all governments and constitutions. As
a matter of fact, these basic rights need not even be written in the
Constitution for they are assumed to exist from the inception of
humankind. If they are now explicitly mentioned in the fundamental
charter, it is because of the well-founded fear of its framers that
unless the rights to a balanced and healthful ecology and to health
are mandated as state policies by the Constitution itself, thereby
highlighting their continuing importance and imposing upon the
state a solemn obligation to preserve the first and protect and
advance the second, the day would not be too far when all else
would be lost not only for the present generation, but also for those
to come generations which stand to inherit nothing but parched
earth incapable of sustaining life.
Conformably with the enunciated right to a balanced and healthful
ecology and the right to health, as well as the other related
provisions of the Constitution concerning the conservation,
development and utilization of the country's natural resources, then
President Corazon C. Aquino promulgated on 10 June
1987 E.O. No. 192, Section 4 of which expressly mandates that the
Department of Environment and Natural Resources "shall be the
primary government agency responsible for the conservation,
management, development and proper use of the country's
environment and natural resources, specifically forest and grazing
lands, mineral, resources, including those in reservation and
watershed areas, and lands of the public domain, as well as the
licensing and regulation of all natural resources as may be provided
for by law in order to ensure equitable sharing of the benefits
derived therefrom for the welfare of the present and future
generations of Filipinos." Section 3 thereof makes the following
statement of policy:

The above provision stresses "the necessity of maintaining a sound


ecological balance and protecting and enhancing the quality of the
environment." Section 2 of the same Title, on the other hand,
specifically speaks of the mandate of the DENR; however, it makes
particular reference to the fact of the agency's being subject to law
and higher authority.
It may, however, be recalled that even before the ratification of the
1987 Constitution, specific statutes already paid special attention to
the "environmental right" of the present and future generations. On
6 June 1977, P.D. No. 1151 and P.D. No. 1152 were issued. Thus, the

right of the petitioners to a balanced and healthful ecology is as


clear as the DENR's duty under its mandate and by virtue of its
powers and functions under E.O. No. 192 and the Administrative
Code of 1987 to protect and advance the said right.
A denial or violation of that right by the other who has the
correlative duty or obligation to respect or protect the same gives
rise to a cause of action. Petitioners maintain that the granting of
the TLAs, which they claim was done with grave abuse of discretion,
violated their right to a balanced and healthful ecology; hence, the
full protection thereof requires that no further TLAs should be
renewed or granted.
It is settled in this jurisdiction that in a motion to dismiss based on(1)
the ground that the complaint fails to state a cause of action; the
question submitted to the court for resolution involves the
sufficiency of the facts alleged in the complaint itself. No other
matter should be considered; furthermore, the truth of falsity of the
said allegations is beside the point for the truth thereof is deemed
hypothetically admitted. Policy formulation or determination by the
executive or legislative branches of Government is not squarely put
in issue. What is principally involved is the enforcement of a right
vis-a-vis policies already formulated and expressed in legislation. It(2)
must, nonetheless, be emphasized that the political question
doctrine is no longer, the insurmountable obstacle to the exercise of
judicial power or the impenetrable shield that protects executive
and legislative actions from judicial inquiry or review.

(NO CASES)

Section 12
Miranda vs Arizona (Miranda rights)
RIGHTS OF PERSON UNDER CUSTODIAL INVESTIGATION:
Right to remain silent

Only an accused has the absolute right to remain silent.


A person who is not an accused may assume the stance of
silence only when asked an incriminating question.
A person under custodial investigation has the right to
refuse to answer any question. Moreover, his silence may NOT be
used against him [PEOPLE vs. FRAGO,232 SCRA 653].
Right to have competent and independent counsel,
preferably of his own choice
The counsel made available to the person under custodial
investigation must be competent and independent and preferably to
the detainees own choice.

In the second place, even if it is to be assumed that the same are


contracts, the instant case does not involve a law or even an
Q: What is the privilege given to that person under investigation?
A: If a person cannot afford a counsel, he must be provided with
executive issuance declaring the cancellation or modification of
existing timber licenses. Hence, the non-impairment clause cannot
one. So, that is a right and a privilege. It is different from the right
to counsel because the right to counsel, everybody has the right to
as yet be invoked. Nevertheless, granting further that a law has
actually been passed mandating cancellations or modifications, the
counsel, but the person who cannot afford counsel has the privilege
and is given one.
same cannot still be stigmatized as a violation of the nonimpairment clause. This is because by its very nature and purpose,
such as law could have only been passed in the exercise of the(3) Right to be provided with the services of counsel if he cannot
police power of the state for the purpose of advancing the right of
afford the services of one.
the people to a balanced and healthful ecology, promoting their
health and enhancing the general welfare.
Finally, it is difficult to imagine, as the trial court did, how the non-(4) Right to be informed of these rights.
impairment clause could apply with respect to the prayer to enjoin
the respondent Secretary from receiving, accepting, processing,
CRIMINAL PROCESS [MIRANDA Case]:
renewing or approving new timber licenses for, save in cases of
renewal, no contract would have as of yet existed in the other
1. Investigation prior to the filing of the charges;
instances. Moreover, with respect to renewal, the holder is not
2. Preliminary Examination and Investigation after charges
entitled to it as a matter of right.
are filed; and
3. Trial Period.
Petition is hereby GRANTED, and the challenged Order of
respondent Judge of 18 July 1991 dismissing Civil Case No. 90-777
is hereby set aside. The petitioners may therefore amend their
Q: When do the rights under Sec. 12 (1) end?
complaint to implead as defendants the holders or grantees of the
questioned timber license agreements.

Pangasinan vs. PSC


SC :Although there is no doubt that it is private property, it is
affected with a public interest and must be submitted to the control
of the government for the common good (Pangasinan
Transportation Co. v. PSC, 70 Phil 221).

Section 11

A: The rights under this section end when the investigation prior to
the filing of the charges also end.

GENERAL RULE: The rights can be waived. BUT the waiver must
be done in writing and in the presence of counsel. The provisions
say that these rights can be waived, if it is in writing and in the
presence of counsel.
Q: What cannot be waived from the above-mentioned rights?
A: The right to be informed cannot be waived, it is an absolute
right.

PURPOSES OF THE RIGHTS IN SEC. 12:


(1) To prohibit incommunicado investigation in a police dominated
atmosphere resulting in selfincriminating statements without
the Miranda Warning.
(2) To prevent coerced confessions given in a police dominated
atmosphere.

NOTE: In Galman case, there was a law compelling the accused to


testify under the pain of contempt while in Ayson casem there is
none.
GENERAL RULE: In administrative investigations, the rights

He was arrested, he was on board a police vehicle


already, he was not in jail. But he was already
deprived of his freedom of action in a significant way
because he was already in the vehicle of the police
and it was the police who questioned him. [PEOPLE vs.
BOLANOS]

3. The investigation must be made by a law enforcer.


[Note: ALL elements must concur.]

People vs Ador june 14 2004

under Sec. 12 do not attach. EXCEPTION: Galman case


because there was a law or EO compelling the accused to
testify
WHEN RIGHTS ARE AVAILABLE:
(1) AFTER a person has been taken into custody or
(2) When a person is otherwise deprived of his freedom of action
in any significant way.
(3) When the investigation is being conducted by the government
(police, DOJ, NBI) with respect to a criminal offense.
(4) Signing of arrest reports and booking sheets.

REQUISITES WHEN RIGHT ATTACHES:


1.
2.
3.

He must be under the custody of law enforcers or is in jail or


deprived of freedom of action in a significant way.
He must be under investigation subjected to questioning or
interrogation by a law enforcer.
Such questioning must be on relation to a crime.

WHEN THE RIGHT ATTACHES OR WHEN CAN A PERSON


INVOKE THESE RIGHTS:
1. Such person must be under investigation;

The right attaches at the start of the investigation


when the investigating officers start to ask and
question immediate information. Now, the question
has to be specific, meaning you are being singled out,
targeted as a perpetrator. If only asked a general
question, that is not considered under investigation
because it says here that you must be interrogated or
questioned about matters relating to the crime, so it
must be a specific kind of investigation.

When a suspect is being invited to the police station


and asked questions about the crime, even if he is not
arrested. But if all the requisites are present, then the
right attaches. [PEOPLE vs. TAN]

When the questioning is not a general inquiry for any


possible lead to the perpetration of the crime under
investigation. [PEOPLE vs. BRAVO, 318 SCRA 812]

The accused was arrested and confession was made


while walking along the highway with the policemen.
The admission is inadmissible. [PEOPLE vs. BARIQUIT,
341 SCRA 600]
2. The person being investigated must be:
a. In the custody of a law enforcer, or
b. Must be in jail, or
c. Deprived of freedom of action in a significant way.

The first circumstance which the prosecution sought to prove is that


the accused were supposedly seen fleeing from the locus criminis,
armed with their respective weapons. Thus, the trial court, gleaning
from the evidence presented, found that "[w]hen about to stand,
Calsis saw Godofredo B. Ador, Diosdado B. Ador, Jr. and Diosdado B.
Ador III, and a person going to the direction of the house of the
Adors which is about 500 meters away." 61 In fact, prosecution
witness Calsis allegedly even saw Diosdado Jr. carrying "a long
firearm but x x x could not determine what kind of gun it was." 62
However, the trial court acquitted Diosdado Jr. But only rightly so.
For, Calsis had difficulty in identifying the Adors notwithstanding his
assertion that he knew and saw them personally. We defer to his
direct examination
The testimony of Calsis, if at all, could hardly be used against
Diosdado III whom he miserably failed to positively identify during
trial. In fact, the acquittal of Diosdado Jr. by the trial court renders
the entire testimony of Calsis in serious doubt. Calsis was presented
to positively identify the assailants who were supposedly personally
known to him and were just ten (10) meters away from him. It
puzzles us no end why he cannot even identify the Adors in open
court.
Thus, despite Calsis assertion that Diosdado Jr. was one of the
assailants, the trial court doubted him and gave credence to the
alibi of Diosdado Jr. that the latter was in Nangka, Marikina, when
the killings took place. The trial court favored the unbiased
testimony of Aspe who said that Diosdado Jr. worked as a
timekeeper and warehouseman with him at the Consuelo
Construction at Nangka, Marikina, from February 15, 1997, until
March 22, 1997, and went home to Pacol only on May 27, 1997.
This ruling is strengthened by the fact that on the morning following
the killings, all the male members of the Ador family were brought
to the police headquarters for paraffin examination and Diosdado Jr.
was not among them.64 We thus respect the finding of the trial court
that indeed Diosdado Jr. was not at the scene of the crime absent
any indication that the lower court overlooked some facts or
circumstances which if considered would alter the outcome of the
case.65
While it is true that the courts are not bound to accept or reject an
entire testimony, and may believe one part and disbelieve
another,66 our Constitution and the law mandate that all doubts
must be resolved in favor of the accused. Calsis committed an
obvious blunder in identifying the supposed assailants which this
Court cannot simply let go. On the contrary, it creates reasonable
doubt in our minds if Calcis really saw the persons he allegedly saw
or if he was even where he said he was that evening. For, it is
elementary that the positive identification of the accused is crucial
in establishing his guilt beyond reasonable doubt. That is wanting in
the instant case.

What is more, Calsis asseverations, at the outset, could no longer


be used against Godofredo since both the prosecution and the
defense have already rested and the case against Godofredo was
already submitted for decision when Calsis was presented. 67 Neither
can they still be used against Diosdado Jr. who was already
acquitted by the trial court.
Both Diosdado III and Godofredo denied the charges hurled against
them. But, while it is true that alibi and denial are the weakest of
the defenses as they can easily be fabricated, 68 absent such clear
and positive identification, the doctrine that the defense of denial
cannot prevail over positive identification of the accused must yield
to the constitutional presumption of innocence. 69 Hence, while
denial is concededly fragile and unstable, the conviction of the
accused cannot be based thereon. 70 The rule in criminal law is
firmly entrenched that verdicts of conviction must be predicated on
the strength of the evidence for the prosecution and not on the
weakness of the evidence for the defense.71

People Vs. Judge Ayson


175 SCRA 216

answer incriminating questions and not a prohibition of inquiry." the


right can be claimed only when the specific question, incriminatory
in character, is actually put to the witness. It cannot be claimed at
any other time. It does not give a witness the right to disregard a
subpoena, to decline to appear before the court at the time
appointed, or to refuse to testify altogether. It is a right that a
witness knows or should know. He must claim it and could be
waived.
Rights in custodial interrogation as laid down in miranda v. Arizona:
the rights of the accused include:
1)

he shall have the right to remain silent and to counsel, and to


be informed of such right.

2)

nor force, violence, threat, intimidation, or any other means


which vitiates the free will shall be used against him.

3)

any confession obtained in violation of these rights shall be


inadmissible in evidence.

The individual may knowingly and intelligently waive these rights


and agree to answer or make a statement. But unless and until
such rights and waivers are demonstrated by the prosecution at the
trial, no evidence obtained as a result of interrogation can be used
against him.

G.R. No. 85215 July 7, 1989


Facts:
Felipe Ramos was a ticket freight clerk of the Philippine Airlines,
assigned at its Baguio City station. It was alleged that he was
involved in irregularities in the sales of plane tickets, the PAL
management notified him of an investigation to be conducted. That
investigation was scheduled in accordance with PAL's Code of
Conduct and Discipline, and the Collective Bargaining Agreement
signed by it with the
Philippine Airlines Employees' Association (PALEA) to which Ramos
pertained. A letter was sent by Ramos stating his willingness to
settle the amount of P76,000. The findings of the Audit team were
given to him, and he refuted that he misused proceeds of tickets
also stating that he was prevented from settling said amounts. He
proffered a compromise however this did not ensue. Two months
after a crime of estafa was charged against Ramos. Ramos pleaded
not guilty. Evidence by the prosecution contained Ramos written
admission and statement, to which defendants argued that the
confession was taken without the accused being represented by a
lawyer. Respondent Judge did not admit those stating that accused
was not reminded of his constitutional rights to remain silent and to
have counsel. A motion for reconsideration filed by the prosecutors
was denied. Hence this appeal.
Issue:
Whether or Not the respondent Judge correct in making
inadmissible as evidence the admission and statement of accused.
Held:
No. Section 20 of the 1987 constitution provides that the right
against self-incrimination (only to witnesses other than accused,
unless what is asked is relating to a different crime charged- not
present in case at bar).
This is accorded to every person who gives evidence, whether
voluntarily or under compulsion of subpoena, in any civil, criminal,
or administrative proceeding. The right is not to "be compelled to
be a witness against himself. It prescribes an "option of refusal to

People vs Bonola

gr no 116394 june 19

1997
The crime at bar was committed in 1979. In force at the time
appellant gave his extrajudicial confession was Section 20 of Article
IV of the 1973 Constitution, viz:
Sec. 20. No person shall be compelled to be a witness against
himself. Any person under investigation for the commission of
an offense shall have the right to remain silent and to counsel,
and to be informed of such right. No force, violence, threat
intimidation, or any other means which vitiates the free will
shall be used against him. Any confession obtained in violation
of this section shall be inadmissible in evidence. (emphasis
ours)
In light of this constitutional proscription, we hold that appellant's
extrajudicial confession is inadmissible in evidence. The doctrine on
waiver of the rights of an accused to remain silent and to counsel
has evolved over the years, particularly upon the effectivity of the
1973
Constitution. 32 In People vs. Morales 33 and, later, in People vs.Galit,
34
this Court laid down the procedure to be followed when a suspect
is taken into custody for investigation, to wit:
7. At the time a person is arrested, it shall be the
duty of the arresting officer to inform him of the
reason for the arrest and he must be shown the
warrant of arrest, if any. He shall be informed of
his constitutional rights to remain silent and to
counsel, and that any statement he might make
could be used against him. The person arrested
shall have the right to communicate with his
lawyer, a relative, or anyone he chooses by the
most expedient means by telephone if possible
or by letter or messenger. It shall be the
responsibility of the arresting officer to see to it
that this
is
accomplished. No custodial

investigation shall be conducted unless it be in


the presence of counsel engaged by the person
arrested, by any person on his behalf, or
appointed by the court upon petition either of the
detainee himself or by anyone on his behalf. The
right to counsel may be waived but the waiver
shall not be valid unless made with the
assistance of counsel. Any statement obtained in
violation of the procedure laid down, whether
exculpatory or inculpatory, in whole or in part,
shall be inadmissible in evidence. (emphasis
supplied)
In line with Morales and Galit, we held in People vs. Decierdo 35 that
uncounselled confessions are inadmissible in evidence. We
elucidated that when the accused is not assisted by counsel, his
statement, in contemplation of the law, becomes "involuntary"
even if it were otherwise voluntary, in a technical sense. In People
vs. Dacoycoy 36 and People vs.Pecardal, 37 we categorically ruled
that a waiver of the constitutional right to counsel shall not be valid
when the waiver is made without the presence and assistance of
counsel.

We hold that the totality of the circumstantial evidence


against appellant cannot support his conviction.
The statements made by appellant pertaining to the stolen pieces
of jewelry are inadmissible in evidence against him because these
were taken in violation of his rights to counsel and to remain silent.
Nor can the recovered pieces of jewelry be used as evidence
against the appellant. They are fruits of the poisonous tree. 43 The
other circumstantial evidence cited by the Solicitor General cannot
support appellant's conviction. We note that appellant was
implicated in the crime because of the uncorroborated testimony of
prosecution witness Berania that appellant was near the scene of
the crime in the late afternoon of November 12, 1979. Berania
claimed he knew appellant by face because the latter used to work
as a palay weeder for Berania's uncle, Macario Joaquin, whose
house is situated opposite the house of the victims. 44 Yet, when
Berania was interviewed by Pfc. De Armas on the day the crime was
discovered, Berania did not reveal that appellant was one of the
three persons passing near the house of the victims. The
description of the probable suspects, through the information
relayed by
Berania to Pfc. de Armas, is as follows: 45

In the case at bar, appellant was not represented or assisted by


counsel at the time he waived his custodial rights as a suspect in a
crime. The pertinent part of appellant's "Salaysay", states:
The evidence shows that the arresting policemen did not
comply with the procedure laid down in Morales andGalit.
Appellant was immediately placed under custodial investigation
upon his arrest. Allegedly, he verbally admitted his complicity
in the crime when interrogated by the police. His incriminating
statements, however, were made without the presence and
assistance of counsel. Pfc.
Dela Cruz testified as follows: 38
It is not material that appellant's confession came in verbal form.
Section 20, Article IV of the 1973 Constitution does not distinguish
between verbal and non-verbal confessions. So long as they are
uncounselled, they are inadmissible in evidence. 41 What is sought
to be avoided is "the evil of extorting from the very mouth of the
person undergoing interrogation for the commission of an offense,
the very evidence with which to prosecute and thereafter convict
him." 42
In the case at bar, the need for counsel is more pronounced.
Appellant was nineteen (19) years old when arrested. He was
unschooled, barely literate. He was a stranger to the niceties of the
law, ignorant of the rituals of police investigation. It is difficult to
believe he made an intelligent waiver of his right to counsel.
Be that as it may, the Solicitor General argues that there are
enough circumstantial evidence linking appellant to the robberyslaying incident, viz:

7. Jose Berania, the person who discovered the incident


alleged that he saw these three persons passing near the
house of the victims looking at said house suspiciously and
described them as follows: That one of them stands 5'8" in
height, slim built, black complexion, army type haircut
wearing a colored red checkered shirt and the other stands
5'3" in height, regular built with curly hair. This allegation
was confirmed by one Pacifico Manalansang, another witness
whom he saw drinking at the house of the victims.
Vital as it is, the omission was not explained by the prosecution
during the trial. It was not also shown that the description of
Berania tallied with the physical features of appellant. Even
prosecution witness Pfc. Dela Cruz revealed that they were looking
for another suspect before the name of appellant finally surfaced.
Pfc. dela Cruz testified: 46
Assuming, arguendo, that appellant was near the scene of
the crime several hours before the killing, his presence
cannot lead to a reasonable inference that he was one of
the assailants of the victims.
IN VIEW WHEREOF, the decision of the Regional Trial Court of
Malolos, Bulacan (Branch 43), in Criminal
Case No. 3653-M, is REVERSED and SET ASIDE. Appellant TEODORO
BONOLA y DELA CRUZ is hereby ACQUITTED of the crime charged
due to reasonable doubt. His immediate release from confinement
is hereby ordered, unless there is any other lawful cause for his
continued detention. Costs de oficio.
SO ORDERED.

Under Section 4 of Rule 133, circumstantial evidence is sufficient for


conviction if:

Section 13

(a) There is more than one circumstance;


(b) The facts from which the inferences are
derived are proven; and
(c) The combination of all the circumstances is
such as to produce a conviction beyond a
reasonable doubt.

Leviste vs. CA Gr no 189122 March 17 2010


Facts:

Jose Antonio Leviste was charged with the crime of murder but was
convicted by the RTC for the lesser crime of homicide. He appealed
the RTC's decision to the CA then he field an application for
admission to bail pending appeal, due to his advanced age and
health condition, and claiming the absence of any risk or possibility
of flight on his part.
The CA denied his application on the ground that the discretion to
extend bail during the course of appeal should be exercised with
grave caution and only for strong reasons. That bail is not a sick
pass for an ailing or aged detainee or a prisoner needing medical
care outside the prison facility.
On this matter, Levisete questioned the ruling of the CA and
averred that the CA committed grave abuse of discretion in the
denial of his application for bail considering that none of the
conditions justifying denial of bail under the Sec. 5 (3) Rule 114 of
the Rules of Court was present. That when the penalty imposed by
the trial court is more than six years but not more than 20 years
and the circumstances in the above-mentioned provision are
absent, bail must be granted to an appellant pending appeal.
Issue:
Whether or not the CA committed grave abuse of discretion in
denying the application for bail of Leviste.
Ruling:
No, under Sec 5 of Rule 114 bail is discretionary, upon conviction by
the RTC of an offense not punishable by death, reclusion perpetua,
or life imprisonment. Under par. 3 of the same rule if the penalty
impose is more than 6 years the accused shall be denied bail, or his
bail be cancelled upon a showing by the prosecution, with notice to
the accused, of the following or other circumstances:

Bail pending appeal is governed by Sec. 5 of Rule 114, Revised


Rules
of
Criminal
Procedure,
which
provides:chanroblesvirtuallawlibrary
Sec. 5. Bail, when discretionary. Upon conviction by the Regional
Trial Court of an offense not punishable by death, reclusion
perpetua, or life imprisonment, admission to bail is discretionary.
The application for bail may be filed and acted upon by the trial
court despite the filing of a notice of appeal, provided it has not
transmitted the original record to the appellate court. However, if
the decision of the trial court convicting the accused changed the
nature of the offense from non-bailable to bailable, the application
for bail can only be filed with and resolved by the appellate court.

Should the court grant the application, the accused may be allowed
to continue on provisional liberty during the pendency of the appeal
under the same bail subject to the consent of the bondsman.
If the penalty imposed by the trial court is imprisonment exceeding
six (6) years, the accused shall be denied bail, or his bail shall be
cancelled upon a showing by the prosecution, with notice to the
accused, of the following or other similar circumstances:
(a) That he is a recidivist, quasi-recidivist, or habitual delinquent, or
has committed the crime aggravated by the circumstance of
reiteration;
(b) That he has previously escaped from legal confinement, evaded
sentence, or violated the conditions of his bail without a valid
justification;
(c) That he committed the offense while under probation, parole, or
conditional pardon;

1.

that he is a recidivist, quasi-recidivist, or habitual


delinquent, or has committed the crime aggravated by the
circumstance of reiteration;

(d) That the circumstances of his case indicate the probability of


flight if released on bail; or

2.

that he has previously escaped from legal confinement,


evaded sentence, or violated the conditions of his bail
without a valid justification;

(e) That there is undue risk that he may commit another crime
during the pendency of the appeal.

3.

that he committed the offense while under probation,


parole, or conditional pardon;

4.

that the circumstances of his case indicate the probability


of flight if released on bail; or

The appellate court may, motu proprio or on motion of any party,


review the resolution of the Regional
Trial Court after notice to the adverse party in either case.
(Emphasis supplied.)

5.

that there is undue risk that he may commit another crime


during the pendency of the appeal.

That bail is expressly declared to be discretionary pending appeal


and it cannot be said that CA committed grave abuse of discretion.
After conviction by the trial court, the presumption of innocence
terminates and, accordingly, the constitutional right to bail ends,
from then on the grant of bail is subject to judicial discretion.

Cyril Qui vs People Gr no 196161 Sep 26


2012
Ruling:

Under the present rule, the grant of bail is a matter of discretion


upon conviction by the RTC of an offense not punishable by death,
reclusion perpetua or life imprisonment, as here. The Court
held:chanroblesvirtuallawlibrary
Indeed, pursuant to the "tough on bail pending appeal" policy, the
presence of bail-negating conditions mandates the denial or
revocation of bail pending appeal such that those circumstances
are deemed to be as grave as conviction by the trial court for an
offense punishable by death, reclusion perpetua or life
imprisonment where bail is prohibited.5rll
In the exercise of that discretion, the proper courts are to be guided
by the fundamental principle that the allowance of bail pending
appeal should be exercised not with laxity but with grave caution
and only for strong reasons, considering that the accused has been
in fact convicted by the trial court.6rll

The petition is bereft of merit.


The CA denied petitioners application for bail pending appeal on the
ground that she is a flight risk, a bail-negating factor under Sec.
5(d) of Rule 114 quoted above. The appellate court anchored its

denial on several circumstances, pointed out by the OSG, which


showed petitioners propensity to evade the law, as when she failed
to attend the hearings before the RTC, which compelled said court
to issue three warrants for petitioners arrest. There is no dispute,
and petitioner does not deny the fact, that on various dates,
specifically on August 24, 2005, February 20, 2006 and March 8,
2010, the RTC issued warrants for her arrest. The March 8, 2010
RTC Order also directed the forfeiture of her bail bond.
Petitioners plea for bail pending appeal is bereft of merit.
The CA properly exercised its discretion in denying petitioners
application for bail pending appeal. The CAs determination as to
petitioner being a high risk for flight is not without factual mooring.
Indeed, the undisputed fact that petitioner did not attend the
hearings before the RTC, which compelled the trial court to issue
warrants for her arrest, is undeniably indicative of petitioners
propensity to trifle with court processes. This fact alone should
weigh heavily against a grant of bail pending appeal.

been convicted of a crime where the penalty imposed by the trial


court is imprisonment exceeding six (6) years.
In all, the Court finds the CA to have exercised its discretion soundly
when it denied petitioner's application for bail pending appeal.
WHEREFORE, the instant petition is DENIED for lack of merit.
Accordingly, the assailed December 17, 2010 and March 17,
2011 Resolutions of the Court of Appeals in CA-G.R. CR No.
33494 are AFFIRMED. No costs.rllbrr
SO ORDERED.

Hadja Dipatuan vs Mangutara


AM no. RTJ 09-2190 April 2010
Ruling:

Petitioners penchant to disobey court processes may also be


deduced from the fact that she lied in order to wiggle out of, and
justify her non-appearance on the March 8, 2010 hearing before the
RTC. Petitioner gave the convenient but false excuse that her
father, Cirilo Calpito, was hospitalized on said hearing day (i.e.,
March 8, 2010) and that Cirilo died on March 24, 2010. The lies
foisted on the court were exposed by: (1) the Death Certificate of
Cirilo Calpito clearly showing that he died on March 24, 2009 or a
year before the aforesaid March 2010 RTC hearing; and (2) the
Certification issued by Dr. Aniana Javier stating that Cirilo went to
her clinic on March 9, 2009.
Lest it be overlooked, the RTC notice sent to petitioners bonding
company was returned with the notation "moved out," while the
notice sent to petitioners given address was returned unclaimed
with the notation "RTS no such person according to Hesita Family"
who were the actual occupants in petitioners given address. The
fact of transferring residences without informing her bondsman and
the trial court can only be viewed as petitioners inclination to evade
court appearance, as indicative of flight, and an attempt to place
herself beyond the pale of the law.
Petitioners argument that she has the constitutional right to bail
and that the evidence of guilt against her is not strong is spurious.
Certainly, after one is convicted by the trial court, the presumption
of innocence, and with it, the constitutional right to bail,
ends.7rll As to the strength of evidence of guilt against her,
suffice it to say that what is before the Court is not the appeal of
her conviction, let alone the matter of evaluating the weight of the
evidence adduced against her.
Consequently, the Court agrees with the appellate courts finding of
the presence of the fourth circumstance enumerated in the abovequoted Sec. 5 of Rule 114, Revised Rules of Criminal Procedure, and
holds that the appellate court neither erred nor gravely abused its
discretion in denying petitioners application for bail pending appeal.
The appellate court appeared to have been guided by the
circumstances provided under the Rules. As the Court categorically
held in People v. Fitzgerald, "As for an accused already convicted
and sentenced to an imprisonment term exceeding six years, bail
may be denied or revoked based on prosecution evidence as to the
existence of any of the. circumstances under
Sec. 5, paragraphs (a) to (e) x x x."8rll Evidently, the
circumstances succinctly provided in Sec. 5 of Rule 114, Revised
Rules of Criminal Procedure have been placed as a guide for the
exercise of the appellate court's discretion in granting or denying
the application for bail, pending the appeal of an accused who has

Judges; gross ignorance of the law. While a judge may not be


held liable for gross ignorance of the law for every erroneous order
that he renders, it is also axiomatic that when the legal principle
involved is sufficiently basic, lack of conversance with it constitutes
gross ignorance of the law. Indeed, even though a judge may not
always be subjected to disciplinary action for every erroneous order
or decision he renders, that relative immunity is not a license to be
negligent or abusive and arbitrary in performing his adjudicatory
prerogatives. It does not mean that a judge need not observe
propriety, discreetness and due care in the performance of his
official functions. This is because if judges wantonly misuse the
powers vested on them by the law, there will not only be confusion
in the administration of justice but also oppressive disregard of the
basic requirements of due process.
The rule is very explicit as to when admission to bail is discretionary
on the part of the respondent Judge. In offenses punishable by
reclusion perpetua or death, the accused has no right to bail when
the evidence of guilt is strong. Thus, as the accused in Criminal
Case No. 3620-01 had been sentenced to reclusion perpetua, the
bail should have been cancelled, instead of increasing it as
respondent Judge did.
Clearly, in the instant case, the act of Mangotara in increasing the
bail bond of the accused instead of canceling it is not a mere
deficiency in prudence, discretion and judgment on the part of
respondent Judge, but a patent disregard of well-known rules.
When an error is so gross and patent, such error produces an
inference of bad faith, making the judge liable for gross ignorance
of the law. It is a pressing responsibility of judges to keep abreast
with the law and changes therein, as well as with the latest
decisions of the Supreme Court. One cannot seek refuge in a mere
cursory acquaintance with the statute and procedural rules.
Ignorance of the law, which everyone is bound to know, excuses no
one not even judges.

Bernardo vs Customs

Gr no L-11363

March 1916
The record shows that the petitioner is nineteen years of age. It
would seem that a young man of that age should have no difficulty
in knowing his real name. The fact that he did not, at least casts
some suspicion upon his identity.

While perhaps many of the statements made during the


examination of Mygatt were not technically admissible, yet when
we take into consideration the fact that they were presented in the
presence of the petitioner and his lawyer, and that no objection was
made by either, and that said declarations were presented primarily
for the purpose of showing the percentage of the petitioner and the
general reputation of that fact in the community in which he
claimed he was born and reared, we are of the opinion and so hold
that for that purpose they were sufficient proof to justify the
conclusion of the department of customs "that the evidence now
presented that the detained is not the Bernardo Molden who was
born in Mogpog, but that he is a Chinese person who entered the
Philippine Islands, October 25, 1914, unlawfully and in violation of
the Act of Congress of February 20, 1907, as stated in the warrant
of arrest."

judgments of conviction or acquittal. Moreover, the right to bail


flows from the presumption of innocence in favor of accused who is
entitled to acquittal unless his guilt is proved beyond reasonable
doubt. It follows, therefore, that the provision on bail will not apply
to extradition he is sought to be extradited are bailable in the
United States is no argument.
Extradition proceedings are separate and distinct from the trial for
the offenses for which he is charged. He should apply for bail
before the US court, not before the extradition court.

In reaching this conclusion concerning the existence and sufficiency


of such proof, we have not taken into consideration the affidavits
presented nor the telegrams and letters of Gregorio Nieva. Said
affidavits and letters should not be considered as proof for the
reason that the petitioner did not have an opportunity to see or
hear said person as a witness. (Chun Tung vs. Collector of Customs,
R. G. No. 9860, decided October 14, 1914, not published; Loo Sing
vs. Collector of Customs, 27 Phil. Rep., 491.) The petitioner and his
attorney, however, had an opportunity to cross-examined Mygatt
and had a right to call witnesses to disprove his statements and his
declarations. The petitioner called no witnesses and made no effort
to disprove the declarations of Mygatt. He preferred to rely upon a
mere technicality rather than to attempt to show, by undisputable
proof, that he was the son of a Filipino woman and was born in the
municipality of Mogpog. If he was the person he claimed to be, it
would have been easy for him to have called his parents and to
have proved his allegations. The burden was upon him to show that
he was the person he claimed to be.

521 SCRA 470 (2007)

Finding as we do that the department of customs had some proof


before it, which justified its conclusions in fact and in law, we are
without jurisdiction, in the absence of proof of abuse of authority, to
change or modify those conclusions. Therefore we are of the
opinion and so declare that the judgment of the Collector of
Customs that the petitioner should be deported is hereby affirmed,
with costs, and it is hereby ordered and decreed that the judgment
of the court a quo be modified and that the record be returned to
the court whence it came, with direction that a judgment be
entered in accordance herewith. So ordered.

GOVERNMENT OF HONG KONG


vs. OLALIA AND MUNOZ

Facts: The Republic of the Philippines and the then British Crown
Colony of Hong Kong signed an Agreement for the Surrender of
Accused and Convicted Persons. Two years after the agreement was
signed, Hong Kong reverted back to the Peoples Republic of China
and became the Hong Kong Special Administrative Region. Muoz
was charged before the Hong Kong Court with 3 counts of the
offense of accepting an advantage as agent.
He also faces 7
counts of the offense of conspiracy to defraud, penalized by the
common law of Hong Kong. Thereafter, warrants of arrest were
issued against him. If convicted, he faces a jail term of 7 to 14
years for each charge.
The DOJ received from the Hong Kong DOJ a request for the
provisional arrest of Muoz. The DOJ then forwarded the request to
the NBI which, in turn, filed an application for the provisional arrest
of private respondent. Hence, an order of arrest was issued against
Muoz. Petitioner filed a petition for the extradition of Muoz. For
his part, Muoz filed, in the same case, a petition for bail which was
opposed by petitioner. The application for bail was denied holding
that there is no Philippine law granting bail in extradition cases and
that private respondent is a high flight risk. However, when the
case was raffled to PR Judge Olalia since the previous judge
inhibited himself, he granted Muozs application for bail.
Issue: WON a prospective extraditee may be granted bail.
Ruling:

GOVERNMENT
PURGANAN (VIP)

OF

USA

vs.

September 24, 2002


Facts: The Government of the United States of America, through
the Department of Justice, filed with the Regional Trial Court a
petition for the extradition of Mark Jimenez who was facing various
criminal charges in Florida. Petitioners prayed for his immediate
arrest, and the judge set it for hearing. Thereafter, the judge issued
an order for his arrest and fixed bail for his temporary liberty at P1
million. In extradition proceedings, is the prospective extraditee
entitled to post bail while the extradition proceedings are pending?
Ruling: No. Extradition proceedings are not criminal in nature. By
using the term before conviction, it is apparent that the right to
bail in Sec. 13, Art III of the Constitution does not apply to
extradition proceedings because extradition courts do not render

Philippine jurisprudence has not limited the exercise of the right to


bail to criminal proceedings only. This Court has admitted to bail
persons who are not involved in criminal proceedings. In fact, bail
has been allowed in this jurisdiction to persons in detention during
the pendency of administrative proceedings, taking into cognizance
the obligation of the Philippines under international conventions to
uphold human rights.
The right of a prospective extraditee to apply for bail in this
jurisdiction must be viewed in the light of the various treaty
obligations of the Philippines concerning respect for the promotion
and protection of human rights.
Under these treaties, the
presumption lies in favor of human liberty. Thus, the Philippines
should see to it that the right to liberty of every individual is not
impaired.
Muoz was arrested on September 23, 1999, and remained
incarcerated until December 20,
2001, when the trial court ordered his admission to bail. In other
words, he had been detained for over two (2) years without having

been convicted of any crime. By any standard, such an extended


period of detention is a serious deprivation of his fundamental right
to liberty. In fact, it was this prolonged deprivation of liberty which
prompted the extradition court to grant him bail. While our
extradition law does not provide for the grant of bail to an
extraditee, however, there is no provision prohibiting him or her
from filing a motion for bail, a right to due process under the
Constitution.

accused is apprised of his right to have and to be assisted by


counsel. So ordered.

Thus, in this case, there is no showing that Muoz


presented evidence to show that he is not a flight risk.
Consequently, this case should be remanded to the trial court to
determine whether private respondent may be granted bail on the
basis of clear and convincing evidence.

Garcia vs Enrile April 26 1983 Gr no 61016

Section 14
People vs Holgado Gr no L-2809 march 22
1950
Ruling:
One of the great principles of justice guaranteed by our Constitution
is that "no person shall be held to answer for a criminal offense
without due process of law", and that all accused "shall enjoy the
right to be heard by himself and counsel." In criminal cases there
can be no fair hearing unless the accused be given the opportunity
to be heard by counsel. The right to be heard would be of little avail
if it does not include the right to be heard by counsel. Even the
most intelligent or educated man may have no skill in the science
of the law, particularly in the rules of procedure, and, without
counsel, he may be convicted not because he is guilty but because
he does not know how to establish his innocence. And this can
happen more easily to persons who are ignorant or uneducated. It
is for this reason that the right to be assisted by counsel is deemed
so important that it has become a constitutional right and it is so
implemented that under our rules of procedure it is not enough for
the Court to apprise an accused of his right to have an attorney, it
is not enough to ask him whether he desires the aid of an attorney,
but it is essential that the court should assign one de oficio if he so
desires and he is poor grant him a reasonable time to procure an
attorney of his own.
It must be added, in the instant case, that the accused who was
unaided by counsel pleaded guilty but with the following
qualification: "but I was instructed by one Mr. Ocampo." The trial
court failed to inquire as to the true import of this qualification. the
record does not show whether the supposed instructions was real
and whether it had reference to the commission of the offense or to
the making of the plea guilty. No investigation was opened by the
court on this matter in the presence of the accused and there is
now no way of determining whether the supposed instruction is a
good defense or may vitiate the voluntariness of the confession.
Apparently the court became satisfied with the fiscal's information
that he had investigated Mr. Ocampo and found that the same had
nothing to do with this case. Such attitude of the court was wrong
for the simple reason that a mere statement of the fiscal was not
sufficient to overcome a qualified plea of the accused. But above
all, the court should have seen to it that the accused be assisted by
counsel specially because of the qualified plea given by him and
the seriousness of the offense found to be capital by the court.
The judgment appealed from is reversed and the case is remanded
to the Court below for a new arraignment and a new trial after the

Section 15
Reversal of the Lansang
Montenegro Doctrine

Doctrine

&

Reinstatement

of

the

In July 1982, Sabino Padilla, together w/ 8 others who were having a


conference in a house in
Bayombong, NV, were arrested by members of the PC. The raid of
the house was authorized by a search warrant issued by Judge
Sayo. Josefina, mother of Sabino, opposed the arrest averring that
no warrant of arrest was issued but rather it was just a warrant of
arrest hence the arrest of her son and the others was w/o just
cause. Sabino and companions together with 4 others were later
transferred to a facility only the PCs know. Josefina petitioned the
court for the issuance of the writ of habeas corpus.
ISSUE: Whether or not the arrests done against Sabino et al is valid.
HELD:
In a complete about face, the SC decision in the Lansang Case was
reversed and the ruling in the Barcelon Case & the Montenegro
Case was again reinstated. The questioned power of the president
to suspend the privilege of the WoHC was once again held as
discretionary in the president. The SC again reiterated that the
suspension of the writ was a political question to be resolved solely
by the president. It was also noted that the suspension of the
privilege of the writ of habeas corpus must, indeed, carry with it the
suspension of the right to bail, if the governments campaign to
suppress the rebellion is to be enhanced and rendered effective. If
the right to bail may be demanded during the continuance of the
rebellion, and those arrested, captured and detained in the course
thereof will be released, they would, without the least doubt, rejoin
their comrades in the field thereby jeopardizing the success of
government efforts to bring to an end the invasion, rebellion or
insurrection.
NOTE: This ruling was abrogated by Sec 18, Art 7 of the
1987 Constitution which expressly constitutionalized the
Lansang Doctrine. Note as well that under Art 3 (Sec 13) of
the Constitution it is stated that the right to bail shall not
be impaired even if the privilege of the writ of habeas
corpus is suspended.
Section 16
(NO CASES)

Section 17
People vs Gamboa February 25 1991
ISSUE: Whether or not THE TRIAL COURT ERRED IN NOT REJECTING
THE PARAFFIN TEST RESULTS AS INADMISSIBLE EVIDENCE.
Ruling:

As to the paraffin test to which the appellant was subjected to he


raises the question, under the sixth assigned error, that it was not
conducted in the presence of his lawyer. This right is afforded to
any person under investigation for the commission of an offense
whose confession or admission may not be taken unless he is
informed of his right to remain silent and to have competent and
independent counsel of his own choice. 8 His right against selfincrimination is not violated by the taking of the paraffin test of his
hands. This constitutional right extends only to testimonial
compulsion and not when the body of the accused is proposed to be
examined as in this case. 9 Indeed, the paraffin test proved
positively that he just recently fired a gun. Again, this kind of
evidence buttresses the case of the prosecution

People vs Otadora Gr no L-2154 April 26


1950

woman can also invoke the right to privacy and it will most probably
be ordered in favor of the woman)

BELTRAN vs. SAMSON

(VIP)

Gr

no.

32025 Sep 23 1929


Facts: Accused was charged with falsification and the fiscal ordered
him to take a specimen of his writing and afterwards will compare
his handwriting with the writing of the falsified documents. The
accused refused invoking the right against self-incrimination.
Ruling: The court held this as a violation of the right to selfincrimination. Asking the accused to write is a positive estimated
act. Writing is more than moving the hand, the body or the fingers.
It is more than just a mechanical act because it requires the use of
intelligence and attention.

Ruling:
Further corroboration of appellant's criminal connection with the
bloody affair is the undisputed possession by Otadora of the pants
of Francisco Galos (Exhibit C) and his hat Exhibit D. It appears that
when Francisco Galos denied ownership of the pants he was
ordered to put it on; and the judge found that it fitted him perfectly.
This incident gave the defense opportunity for extended argument
that the constitutional protection against self-incrimination had
been erroneously disregarded. But we discover in the record no
timely objection upon that specific ground. And it is to be doubted
whether the accused could benefit from the error, if any.
Furthermore, and this is conclusive, "measuring or photographing
the party is not within the privilege" (against self-incrimination).
"Nor is the removal or replacement of his garments or shoes. Nor is
the requirement that the party move his body to enable the
foregoing things to be done." (Wigmore on Evidence, Vol. 4, p. 878,
quoted in Beltran vs. Samson and Jose, 53 Phil., 570, 576).
In conclusion, we are fully satisfied from a reading of the whole
expediente that the appellant induced Antonio Otadora to commit
the double murder, and furnished him with the deadly firearm. She
is just as guilty as if she herself had perpetrated the murderous
assaults. The slaying is qualified by the circumstance of treachery. It
is aggravated by evident premeditation; but for lack of sufficient
votes the appellant is sentenced to suffer life imprisonment for
each murder, (not exceeding 40 years, art. 70, Rev. Penal Code),
and to indemnify the heirs of the Castros in the sum of P4,000. The
appealed judgment will be thus modified.

VILLAFLOR vs. SUMMERS

Sep. 8

1920

Facts: The accused was charged for adultery and upon motion of
the prosecutor, the judge ordered her examined to find out if she is
pregnant. She refused and was detained for contempt. She went to
the SC.
Ruling: The court ruled that the right extends to testimonial self
incrimination only. Order of the Court was only for an ocular
inspection of the body of the accused. So as not to embarrass the
witness, the court ordered the physicians not to do any other
means more than necessary. (NOTE: If this case happened now, the

DISTINCTION
CASES:

BETWEEN

THE

BELTRAN

AND

VILLAFLOR

14.
Classify the act
In the Villaflor case, the accused was not asked to perform a
positive act. She was only asked to perform a negative act, or to
present an examination of herself, whereas, in the Beltran case, the
accused was asked to perform a positive act.

15.
Evidence
Villaflor was only asked to explain something already in existence,
whereas in Beltran, accused was asked to produce evidence not yet
in existence.

SJS vs. DANGEROUS DRUGS


BOARD (G.R. No. 157870
NOVEMBER 3, 2008

Facts: Section 36 of RA 9165 (Comprehensive Dangerous Drugs Act


of 2002) provides that Authorized drug testing shall be done by any
government forensic laboratories or by any of the drug testing
laboratories accredited and monitored by the DOH to safeguard the
quality of the test results. The drug testing shall employ, among
others, two (2) testing methods, the screening test which will
determine the positive result as well as the type of drug used and
the confirmatory test which will confirm a positive screening test.
The following shall be subjected to undergo drug testing: (c)
Students of secondary and tertiary schools; (d) Officers and
employees of public and private office; (f) All persons charged
before the prosecutor's office with a criminal offense having an
imposable penalty of imprisonment of not less than six years and
one day shall undergo a mandatory drug test; and (g) All
candidates for public office whether appointed or elected both in
the national or local government shall undergo a mandatory drug
test.

Petitioner Laserna, Jr., as citizen and taxpayer, seeks in his


Petition for Certiorari and Prohibition that the aforementioned
provision of RA9165 be struck down as unconstitutional for
infringing on the constitutional right to privacy, the right against
unreasonable search and seizure, and the right against self incrimination, and for being contrary to the due process and equal
protection guarantees.
On the other hand, Social Justice Society, a registered political party,
seeks to prohibit the
Dangerous Drugs Board and the Philippine Drug Enforcement
Agency from enforcing paragraphs (c), (d), (f), and (g) of Sec. 36 of
RA 9165 on the ground that they are constitutionally infirm. For
one, the provisions constitute undue delegation of legislative power
when they give unbridled discretion to schools and employers to
determine the manner of drug testing. For another, the provisions
trench in the equal protection clause inasmuch as they can be used
to harass a student or an employee deemed undesirable. And for a
third, a person's constitutional right against unreasonable searches
is also breached by said provisions.
Issue: WON the aforesaid provision violated the constitutional right
against self-incrimination.
Ruling:
Unlike the situation covered by Sec. 36(c) and (d) of RA 9165, the
Court finds no valid justification for mandatory drug testing for
persons accused of crimes. In the case of students, the
constitutional viability of the mandatory, random, and suspicionless
drug testing for students emanates primarily from the waiver by the
students of their right to privacy when they seek entry to the
school, and from their voluntarily submitting their persons to the
parental authority of school authorities. In the case of private and
public employees, the constitutional soundness of the mandatory,
random, and suspicionless drug testing proceeds from the
reasonableness of the drug test policy and requirement. However,
the SC find the situation entirely different in the case of persons
charged before the public prosecutor's office with criminal offenses
punishable with six (6) years and one (1) day imprisonment. The
operative concepts in the mandatory drug testing are
"randomness" and "suspicionless." In the case of persons charged
with a crime before the prosecutor's office, a mandatory drug
testing can never be random or suspicionless. The ideas of
randomness and being suspicionless are antithetical to their being
made defendants in a criminal complaint. They are not randomly
picked; neither are they beyond suspicion. When persons suspected
of committing a crime are charged, they are singled out and are
impleaded against their will. The persons thus charged, by the bare
fact of being haled before the prosecutor's office and peaceably
submitting themselves to drug testing, if that be the case, do not
necessarily consent to the procedure, let alone waive their right to
privacy. To impose mandatory drug testing on the accused is a
blatant attempt to harness a medical test as a tool for criminal
prosecution, contrary to the stated objectives of RA 9165. Drug
testing in this case would violate a persons' right to privacy. Worse
still, the accused persons are veritably forced to incriminate
themselves. Thus, in this case, Section 36 (f) of RA 9165 was
declared unconstitutional and Section and (d) were declared
constitutional..

Law on Public Officers


(Article X)
. Datu Michael Kida
Senate Gr no. 196271 October 18 2011

vs.

Case Digest: Abas Kida v. Senate


G.R. No. 196271, : October 18, 2011
DATU MICHAEL ABAS KIDA, in his personal capacity, and in
representation of MAGUINDANAO
FEDERATION OF AUTONOMOUS IRRIGATORS ASSOCIATION, INC., et
al., Petitioners, v. SENATE OF THE PHILIPPINES, represented by its
President JUAN PONCE ENRILE, HOUSE OF REPRESENTATIVES, et al.,
Respondents.
FACTS:
On August 1, 1989 or two years after the effectivity of the 1987
Constitution, Congress acted through
Republic Act (RA) No. 6734 entitled "An Act Providing for an Organic
Act for the Autonomous Region in Muslim Mindanao."The initially
assenting provinces were Lanao del Sur,Maguindanao, Sulu and
Tawitawi.RA No. 6734 scheduled the first regular elections for the
regional officials of the ARMM on a date not earlier than 60 days nor
later than 90 days after its ratification.
Thereafter, R.A. No. 9054 was passed to further enhance the
structure of ARMM under R.A. 6734. Along with it is the reset of the
regular elections for the ARMM regional officials to the second
Monday of September 2001.
RA No. 9333was subsequently passed by Congress to reset the
ARMM regional elections to the 2ndMonday of August 2005, and on
the same date every 3 years thereafter. Unlike RA No. 6734 and RA
No. 9054, RA No. 9333 was not ratified in a plebiscite.
Pursuant to RA No. 9333, the next ARMM regional elections should
have been held onAugust 8, 2011. COMELEC had begun preparations
for these elections and had accepted certificates of candidacies for
the various regional offices to be elected.But onJune 30, 2011, RA
No. 10153 was enacted, resetting the ARMM elections to May 2013,
to coincide with the regular national and local elections of the
country.With the enactment into law of RA No. 10153, the COMELEC
stopped its preparations for the ARMM elections.
Several cases for certiorari, prohibition and madamus originating
from different parties arose as a consequence of the passage of R.A.
No. 9333 and R.A. No. 10153 questioning the validity of said laws.
OnSeptember 13, 2011, the Court issued a temporary restraining
order enjoining the implementation of RA No. 10153 and ordering the
incumbent elective officials of ARMM to continue to perform their
functions should these cases not be decided by the end of their term
onSeptember 30, 2011.
The petitioners assailing RA No. 9140, RA No. 9333 and RA No. 10153
assert that these laws amend RA No. 9054 and thus, have to comply
with the supermajority vote and plebiscite requirements prescribed

under Sections 1 and 3, Article XVII of RA No. 9094 in order to


become effective.
The petitions assailing RA No. 10153 further maintain that it is
unconstitutional for its failure to comply with the three-reading
requirement of Section 26(2), Article VI of the Constitution.Also cited
as grounds are the alleged violations of the right of suffrage of the
people of ARMM, as well as the failure to adhere to the "elective and
representative" character of the executive and legislative
departments of the ARMM. Lastly, the petitioners challenged the
grant to the President of the power to appoint OICs to undertake the
functions of the elective ARMM officials until the officials elected
under the May 2013 regular elections shall have assumed office.
Corrolarily, they also argue that the power of appointment also gave
the President the power of control over the ARMM, in complete
violation of Section 16, Article X of the Constitution.

Sec. 2, Art. XVIII) will have to be synchronized with the election for
President and Vice President (under
Sec. 5, Art. XVIII) is likewise evident from the x x xrecords of the
proceedings in the Constitutional Commission. [Emphasis supplied.]
Although called regional elections, the ARMM elections should be
included among the elections to be synchronized as it is a "local"
election based on the wording and structure of the Constitution.
Regional elections in the ARMM for the positions of governor, vicegovernor and regional assembly representatives fall within the
classification of "local" elections, since they pertain to the elected
officials who will serve within the limited region of ARMM. From the
perspective of the Constitution, autonomous regions are considered
one of the forms of local governments, as evident from Article Xof
the Constitution entitled "Local Government."Autonomous regions
are established and discussed under Sections 15 to 21 of this Article
the article wholly devoted to Local Government.

ISSUE:
Whether or not the 1987 Constitution mandates the synchronization
of elections
Whether or not the passage of RA No. 10153 violates the provisions
of the 1987 Constitution
HELD:
Court dismissed the petition and affirmed the constitutionality of R.A.
10153 in toto. The Court agreed with respondent Office of the
Solicitor General (OSG) on its position that the Constitution
mandates synchronization, citing Sections 1, 2 and 5, Article XVIII
(Transitory Provisions) of the 1987 Constitution. While the
Constitution does not expressly state that Congress has to
synchronize national and local elections, the clear intent towards this
objective can be gleaned from the Transitory Provisions (Article XVIII)
of the Constitution,which show the extent to which the Constitutional
Commission, by deliberately making adjustments to the terms of the
incumbent officials, sought to attain synchronization of elections.
The objective behind setting a common termination date for all
elective officials, done among others through the shortening the
terms of the twelve winning senators with the least number of votes,
is to synchronize the holding of all future elections whether national
or local to once every three years.This intention finds full support in
the discussions during the Constitutional Commission deliberations.
Furthermore, to achieve synchronization, Congressnecessarilyhas to
reconcile the schedule of the ARMMs regular elections (which should
have been held in August 2011 based on RA No. 9333) with the fixed
schedule of the national and local elections (fixed by RA No. 7166 to
be held in May 2013).
In Osme v. Commission on Elections, the court thus explained:
It is clear from the aforequoted provisions of the 1987 Constitution
that the terms of office of Senators, Members of the House of
Representatives, the local officials, the President and the VicePresident have been synchronized to end on the same hour, date
and year noon of June 30, 1992.
It is likewise evident from the wording of the above-mentioned
Sections that the term ofsynchronizationis used synonymously as the
phraseholding simultaneouslysince this is the precise intent in
terminating their Office Tenure on the sameday or occasion.This
common termination date will synchronize future elections to once
every three years (Bernas, the Constitution of the Republic of the
Philippines, Vol. II, p. 605).
That the election for Senators, Members
Representatives and the local officials (under

of

the

House

of

Second issue: Congress, in passing RA No. 10153, acted strictly


within its constitutional mandate. Given an array of choices, it acted
within due constitutional bounds and with marked reasonableness in
light of the necessary adjustments that synchronization demands.
Congress, therefore, cannot be accused of any evasion of a positive
duty or of a refusal to perform its duty nor is there reason to accord
merit to the petitioners claims of grave abuse of discretion.
In relation with synchronization, both autonomy and the
synchronization of national and local elections are recognized and
established constitutional mandates, with one being as compelling
as the other.If their compelling force differs at all, the difference is in
their coverage; synchronization operates on and affects the whole
country, while regional autonomy as the term suggests directly
carries a narrower regional effect although its national effect cannot
be discounted.
In all these, the need for interim measures is dictated by necessity;
out-of-the-way arrangements and approaches were adopted or used
in order to adjust to the goal or objective in sight in a manner that
does not do violence to the Constitution and to reasonably accepted
norms.Under these limitations, the choice of measures was a
question of wisdom left to congressional discretion.
However, the holdover contained in R.A. No. 10153, for those who
were elected in executive and legislative positions in the ARMM
during the 2008-2011 term as an option that Congress could have
chosen because a holdover violates Section 8, Article X of the
Constitution. In the case of the terms of local officials, their term has
been fixed clearly and unequivocally, allowing no room for any
implementing legislation with respect to the fixed term itself and no
vagueness that would allow an interpretation from this Court. Thus,
the term of three years for local officials should stay at three (3)
years as fixed by the Constitution and cannot be extended by
holdover by Congress.

RA No. 10153, does not in any way amend what the organic law of
the ARMM(RA No. 9054) sets outs in terms of structure of
governance.What RA No. 10153 in fact only does is to"appoint
officers-in-charge for the Office of the Regional Governor, Regional
Vice Governor and Members of the Regional
Legislative Assembly who shall perform the functions pertaining to
the said offices until the officials duly elected in the May 2013
elections shall have qualified and assumed office."This power is far
different from appointing elective ARMM officials for the abbreviated
term ending on the assumption to office of the officials elected in the
May 2013 elections. It must be therefore emphasized that the law
must be interpreted as an interim measure to synchronize elections
and must not be interpreted otherwise.

legal requirements and ask for new recommendees who have the
necessary eligibilities and qualifications.

San Juan vs. CSC

GR No. 92299, 19 April 1991

Facts: The Provincial Budget Officer of Rizal (PBO) was left vacant;
thereafter Rizal Governor San Juan, peititioner, nominated Dalisay
Santos for the position and the latter quickly assumed position.
However, Director Abella of Region IV Department of Budget and
Management (DBM) did not endorse the nominee, and
recommended private respondent Cecilia Almajose as PBO on the
ground that she was the most qualified. This appointment was
subsequently approved by the DBM. Petitioner protested the
appointment of Almajose before the DBM and the Civil Service
Commission who both dismissed his complaints. His arguments rest
on his contention that he has the sole right and privilege to
recommend the nominees to the position of PBO and that the
appointee should come only from his nominees. In support thereof,
he invokes Section 1 of Executive Order No. 112.

Issue: Whether or not DBM is empowered to appoint a PBO who was


not expressly nominated by the provincial governor.

Held: Under the cited Sec 1 of EO 112, the petitioner's power to


recommend is subject to the qualifications prescribed by existing
laws for the position of PBO. Consequently, in the event that the
recommendations made by the petitioner fall short of the required
standards, the appointing authority, public respondent DBM is
expected to reject the same. In the event that the Governor
recommends an unqualified person, is the Department Head free to
appoint anyone he fancies?
Petitioner states that the phrase of said law: "upon recommendation
of the local chief executive concerned" must be given mandatory
application in consonance with the state policy of local autonomy as
guaranteed by the 1987 Constitution under Art. II, Sec. 25 and Art. X,
Sec. 2 thereof. He further argues that his power to recommend
cannot validly be defeated by a mere administrative issuance of
public respondent DBM reserving to itself the right to fill-up any
existing vacancy in case the petitioner's nominees do not meet the
qualification requirements as embodied in public respondent DBM's
Local Budget Circular No. 31 dated February 9, 1988.
This case involves the application of a most important constitutional
policy and principle, that of local autonomy. We have to obey the
clear mandate on local autonomy. Where a law is capable of two
interpretations, one in favor of centralized power in Malacaang and
the other beneficial to local autonomy, the scales must be weighed
in favor of autonomy.
The 1935 Constitution clearly limited the executive power over local
governments to "general supervision . . . as may be provided by
law." The President controls the executive departments. He has no
such power over local governments. He has only supervision and
that supervision is both general and circumscribed by statute. The
exercise of greater local autonomy is even more marked in the
present Constitution. Article II, Section 25 provides: "The State shall
ensure the autonomy of local governments"
Thereby, DBM Circular is ultra vires and is, accordingly, set aside.
The DBM may appoint only from the list of qualified recommendees
nominated by the Governor. If none is qualified, he must return the
list of nominees to the Governor explaining why no one meets the

Aguirre vs. Pimentel gr no. 132988 july 19 2000


FACTS: This is a petition for certiorari and prohibition seeking to
annul Section 1 of Administrative Order No. 372, issued by the
President, insofar as it requires local government units to reduce
their expenditures by 25% of their authorized regular appropriations
for non-personal services and to enjoin respondents from
implementing Section 4 of the Order, which withholds a portion of
their internal revenue allotments.
HELD: Section 1 of the AO does not violate local fiscal autonomy.
Local fiscal autonomy does not rule out any manner of national
government intervention by way of supervision, in order to ensure
that local programs, fiscal and otherwise, are consistent with
national goals. AO 372 is merely directory and has been issued by
the President consistent with his powers of supervision over local
governments. A directory order cannot be characterized as an
exercise of the power of control. The AO is intended only to advise
all government agencies and instrumentalities to undertake costreduction measures that will help maintain economic stability in the
country. It does not contain any sanction in case of noncompliance.
The Local Government Code also allows the President to interfere in
local fiscal matters, provided that certain requisites are met: (1) an
unmanaged public sector deficit of the national government; (2)
consultations with the presiding officers of the Senate and the House
of Representatives and the presidents of the various local leagues;
(3) the corresponding recommendation of the secretaries of the
Department of Finance, Interior and Local Government, and Budget
and Management; and (4) any adjustment in the allotment shall in
no case be less than 30% of the collection of national internal
revenue taxes of the third fiscal year preceding the current one.
Section 4 of AO 372 cannot be upheld. A basic feature of local fiscal
autonomy is the automatic release of the shares of LGUs in the
national internal revenue. This is mandated by the Constitution and
the Local Government Code. Section 4 which orders the withholding
of 10% of the LGUs IRA clearly contravenes the Constitution and the
law.

Public Corporations
1.

Mercado vs. Manzano

Facts: Under Section 40(d) of the Local Government Code, those


holding dual citizenship are disqualified from running for any elective
local position.
Petitioner Ernesto S. Mercado and private respondent Eduardo B.
Manzano were candidates for vice mayor of the City of Makati.
The proclamation of private respondent was suspended in view of a
pending petition for disqualification. The Second Division of the
COMELEC issued a resolution, dated May 7, 1998, granting the
petition and ordered the cancellation of the COC of private
respondent on the ground that he is a dual citizen.

Private respondent filed a motion for reconsideration. The motion


remained pending even until after the election held on May 11, 1998.
Pursuant to Omnibus Resolution No. 3044, of the COMELEC, the
board of canvassers tabulated the votes cast for vice mayor of
Makati City but suspended the proclamation of the winner.
Subsequently, petitioner sought to intervene in the case for
disqualification. Private respondent opposed contending that at the
time of the Elections, the resolution of the Second Division adopted
on 7 May 1998 was not yet final so that, effectively, petitioner may
not be declared the winner even assuming that Manzano is
disqualified to run for and hold the elective office of Vice-Mayor of
the City of Makati.
Issue: WON petitioner who intervened prior proclamation will hold
the elective office of the Vice-Mayor when respondent is disqualified.
Held: Yes. Private respondent argues that petitioner has neither
legal interest in the matter in litigation nor an interest to protect
because he is a defeated candidate for the vice-mayoralty post of
Makati City who cannot be proclaimed as the Vice-Mayor of Makati
City even if the private respondent be ultimately disqualified by final
and executory judgment.
The flaw in this argument is it assumes that, at the time petitioner
sought to intervene in the proceedings before the COMELEC, there
had already been a proclamation of the results of the election for the
vice mayoralty contest for Makati City, on the basis of which
petitioner came out only second to private respondent. The fact,
however, is that there had been no proclamation at that time.
Certainly, petitioner had, and still has, an interest in ousting private
respondent from the race at the time he sought to intervene. The
rule in Labo v. COMELEC, reiterated in several cases, only applies to
cases in which the election of the respondent is contested, and the
question is whether one who placed second to the disqualified
candidate may be declared the winner. In the present case, at the
time petitioner filed a Motion for Leave to File Intervention on May
20, 1998, there had been no proclamation of the winner, and
petitioners purpose was precisely to have private respondent
disqualified from running for an elective local position under par
40(d) of R.A. No. 7160.

New Sun Valley Homeowners


Assoc. vs. City of Paranaque Gr
no.156686 July 27 2011
The issues before us are:
1.

Whether or not petitioner has a right to the protection of the law


that would entitle it to injunctive relief against the implementation
of BSV Resolution No. 98-096; and

2.

Whether or not petitioner failed to exhaust administrative


remedies.
The Ruling of the Court:

The Court of Appeals passed upon petitioners claims as to the


validity of the dismissal in this wise:
We do not agree. Although the Motion to Dismiss was filed on the
same day, but after, the Amended
Petition was filed, the same cannot be considered as directed merely
against the original petition which Appellant already considers as

non-existing. The records will show that Appellants Amended


Petition contained no material amendments to the original petition.
Both allege the same factual circumstances or events that constitute
the Appellants cause of action anent the Appellees alleged violation
of
Appellants propriety rights over the subdivision roads in question.
Corollarily, the allegations in Appellees Motion to Dismiss, as well as
the grounds therefore predicated on lack of cause of action and
jurisdiction, could very well be considered as likewise addressed to
Appellants Amended Petition.
It bears stressing that due process simply means giving every
contending party the opportunity to be heard and the court to
consider every piece of evidence presented in their favor (Batangas
Laguna
Tayabas Bus Company versus Benjamin Bitanga, G.R. Nos. 137934 &
137936[)]. In the instant case,
Appellant cannot be said to have been denied of due process. As
borne by the records, while Appellees Motion to Dismiss did not set
the time for the hearing of the motion, the day set therefore was the
same date set for the hearing of Appellants prayer for the issuance
of a writ of preliminary injunction that is, November 20, 1998, with
the precise purpose of presenting evidence in support of the motion
to dismiss on the same said scheduled hearing date and time when
Appellant and its counsel would be present. Moreover, Appellants
predication of lack of due hearing is belied by the fact that the
hearing held on November 20, 1999 took up not only the matter of
whether or not to grant the injunction, but also tackled the
jurisdictional issue raised in Appellees Motion to Dismiss, which
issues were intertwined in both incidents. 67
We see no reason to depart from these findings by the Court of
Appeals. Petitioners recourse in questioning BSV Resolution No. 98096 should have been with the Mayor of Paraaque City, as clearly
stated in Section 32 of the Local Government Code, which provides:
Section 32. City and Municipal Supervision over Their Respective
Barangays. - The city or municipality, through the city or municipal
mayor concerned, shall exercise general supervision over component
barangays to ensure that said barangays act within the scope of
their prescribed powers and functions.
We do not see how petitioners act could qualify as an exception to
the doctrine of exhaustion of administrative remedies. We have
emphasized the importance of applying this doctrine in a recent
case, wherein we held:
The doctrine of exhaustion of administrative remedies is a
cornerstone of our judicial system. The thrust of the rule is that
courts must allow administrative agencies to carry out their
functions and discharge their responsibilities within the specialized
areas of their respective competence. The rationale for this doctrine
is obvious. It entails lesser expenses and provides for the speedier
resolution of controversies. Comity and convenience also impel
courts of justice to shy away from a dispute until the system of
administrative redress has been completed.68
It is the Mayor who can best review the Sangguniang Barangays
actions to see if it acted within the scope of its prescribed powers
and functions. Indeed, this is a local problem to be resolved within
the local government. Thus, the Court of Appeals correctly found that
the trial court committed no reversible error in dismissing the case
for petitioners failure to exhaust administrative remedies, as the
requirement under the Local Government Code that the closure and
opening of roads be made pursuant to an ordinance, instead of a
resolution, is not applicable in this case because the subject roads
belong to the City Government of Paraaque.

Moreover, being the party asking for injunctive relief, the burden of
proof was on petitioner to show ownership over the subject roads.
This, petitioner failed to do.

Rules of Court provide which matters constitute judicial notice, to


wit:
Rule 129

In civil cases, it is a basic rule that the party making allegations has
the burden of proving them by a preponderance of evidence. Parties
must rely on the strength of their own evidence and not upon the
weakness of the defense offered by their opponent.69
Petitioner dared to question the barangays ownership over the
subject roads when it should have been the one to adduce evidence
to support its broad claims of exclusivity and privacy. Petitioner did
not submit an iota of proof to support its acts of ownership, which, as
pointed out by respondents, consisted of closing the subject roads
that belonged to the then Municipality of Paraaque and were
already being used by the public, limiting their use exclusively to the
subdivisions homeowners, and collecting fees from delivery vans
that would pass through the gates that they themselves had built. It
is petitioners authority to put up the road blocks in the first place
that becomes highly questionable absent any proof of ownership.
On the other hand, the local government units power to close and
open roads within its jurisdiction is clear under the Local Government
Code, Section 21 of which provides:
Section 21. Closure and Opening of Roads. (a) A local government
unit may, pursuant to an ordinance, permanently or temporarily
close or open any local road, alley, park, or square falling within its
jurisdiction: Provided, however, That in case of permanent closure,
such ordinance must be approved by at least two-thirds (2/3) of all
the members of the sanggunian, and when necessary, an adequate
substitute for the public facility that is subject to closure is provided.

WHAT NEED NOT BE PROVED


SECTION 1. Judicial notice, when mandatory.A court shall take
judicial notice, without the introduction of evidence, of the existence
and territorial extent of states, their political history, forms of
government and symbols of nationality, the law of nations, the
admiralty and maritime courts of the world and their seals, the
political constitution and history of the Philippines, the official acts of
the legislative, executive and judicial departments of the Philippines,
the laws of nature, the measure of time, and the geographical
divisions.(1a)1avvphi1
The activities claimed by petitioner to be part of judicial knowledge
are not found in the rule quoted above and do not support its
petition for injunctive relief in any way.
As petitioner has failed to establish that it has any right entitled to
the protection of the law, and it also failed to exhaust administrative
remedies by applying for injunctive relief instead of going to the
Mayor as provided by the Local Government Code, the petition must
be denied.
WHEREFORE, premises considered, the petition is hereby DENIED.
The Court of Appeals DECISION dated October 16, 2002 and its
RESOLUTION dated January 17, 2003 in CA-G.R. CV No. 65559 are
both AFFIRMED.
SO ORDERED.

We quote with approval the ruling of the Court of Appeals in this


regard, as follows:
Contrary, however, to Appellants position, the above-quoted
provision, which requires the passage of an ordinance by a local
government unit to effect the opening of a local road, can have no
applicability to the instant case since the subdivision road lots
Gr no. 191761 Nov. 14, 2012
sought to be opened to decongest traffic in the area - namely
Rosemallow and Aster Streets have already been donated by the
The Issues
Sun Valley Subdivision to, and the titles thereto already issued in the
name of, the City Government of Paraaque since the year 1964
CEPALCO enumerated the following reasons for warranting review:
(Annexes "2" to "7" of Appellees Brief). This fact has not even been
denied by the Appellant in the proceedings below nor in the present
1. In spite of its patent illegality, a City Ordinance passed in violation
recourse. Having been already donated or turned over to the City
or in excess of the citys delegated power to tax was upheld;
Government of Paraaque, the road lots in question have since then
taken the nature of public roads which are withdrawn from the 2. In a case involving pure questions of law, the Court of Appeals still
commerce of man, and hence placed beyond the private rights or
insisted on a useless administrative remedy before resort to the
claims of herein Appellant. Accordingly, the Appellant was not in the
court may be made; and
lawful exercise of its predicated rights when it built obstructing
structures closing the road lots in question to vehicular traffic for the 3. Recent legislation affirming CEPALCOs tax exemptions was
use of the general Public. Consequently, Appellees act of passing
disregarded.16
the disputed barangay resolution, the implementation of which is
sought to be restrained by Appellant, had for its purpose not the
In a Resolution dated 6 July 2011,17 this Court required both parties
opening of a private road but may be considered merely as a
to discuss whether the amount of tax imposed by Section 2 of
directive or reminder to the Appellant to cause the opening of a
Ordinance No. 9503-2005 complies with or violates, as the case may
public road which should rightfully be open for use to the general
be, the limitation set by Section 151, in relation to Sections 137 and
public.70
143(h), of the Local Government Code.

Cagayan Elec. Power Co. vs. City


of CDO

Petitioner wants this Court to recognize the rights and interests of


the residents of Sun Valley
Subdivision but it miserably failed to establish the legal basis, such
as its ownership of the subject roads, which entitles petitioner to the
remedy prayed for. It even wants this Court to take "judicial
knowledge that criminal activities such as robbery and kidnappings
are becoming daily fares in Philippine society."71 This is absurd. The

The Courts Ruling:


Failure to Exhaust Administrative Remedies

Ordinance No. 9503-2005 is a local revenue measure. As such, the


Local Government Code applies.
SEC. 187. Procedure for Approval and Effectivity of Tax Ordinances
and Revenue Measures; Mandatory Public Hearings. The procedure
for approval of local tax ordinances and revenue measures shall be
in accordance with the provisions of this Code: Provided, That public
hearings shall be conducted for the purpose prior to the enactment
thereof: Provided, further, That any question on the constitutionality
or legality of tax ordinances or revenue measures may be raised on
appeal within thirty (30) days from the effectivity thereof to the
Secretary of Justice who shall render a decision within sixty (60) days
from the date of receipt of the appeal: Provided, however, That such
appeal shall not have the effect of suspending the effectivity of the
ordinance and the accrual and payment of the tax, fee, or charge
levied therein: Provided, finally, That within thirty (30) days after
receipt of the decision or the lapse of the sixty-day period without
the Secretary of Justice acting upon the appeal, the aggrieved party
may file appropriate proceedings with a court of competent
jurisdiction.
SEC. 188. Publication of Tax Ordinances and Revenue Measures.
Within ten (10) days after their approval, certified true copies of all
provincial, city, and municipal tax ordinances or revenue measures
shall be published in full for three (3) consecutive days in a
newspaper of local circulation: Provided, however, That in provinces,
cities and municipalities where there are no newspapers of local
circulation, the same may be posted in at least two (2) conspicuous
and publicly accessible places.
The Sangguniang Panlungsod of Cagayan de Oro approved
Ordinance No. 9503-2005 on 10 January 2005. Section 5 of said
ordinance provided that the "Ordinance shall take effect after 15
days following its publication in a local newspaper of general
circulation for at least three (3) consecutive issues." Gold Star Daily
published Ordinance No. 9503-2005 on 1 to 3 February 2005.
Ordinance No. 9503-2005 thus took effect on 19 February 2005.
CEPALCO filed its petition for declaratory relief before the Regional
Trial Court on 30 September 2005, clearly beyond the 30-day period
provided in Section 187. CEPALCO did not file anything before the
Secretary of Justice. CEPALCO ignored our ruling in Reyes v. Court of
Appeals18 on the mandatory nature of the statutory periods:
Clearly, the law requires that the dissatisfied taxpayer who questions
the validity or legality of a tax ordinance must file his appeal to the
Secretary of Justice, within 30 days from effectivity thereof. In case
the Secretary decides the appeal, a period also of 30 days is allowed
for an aggrieved party to go to court. But if the Secretary does not
act thereon, after the lapse of 60 days, a party could already
proceed to seek relief in court. These three separate periods are
clearly given for compliance as a prerequisite before seeking redress
in a competent court. Such statutory periods are set to prevent
delays as well as enhance the orderly and speedy discharge of
judicial functions. For this reason the courts construe these
provisions of statutes as mandatory.
A municipal tax ordinance empowers a local government unit to
impose taxes. The power to tax is the most effective instrument to
raise needed revenues to finance and support the myriad activities
of local government units for the delivery of basic services essential
to the promotion of the general welfare and enhancement of peace,
progress, and prosperity of the people. Consequently, any delay in
implementing tax measures would be to the detriment of the public.
It is for this reason that protests over tax ordinances are required to
be done within certain time frames. In the instant case, it is our view
that the failure of petitioners to appeal to the Secretary of Justice
within 30 days as required by Sec. 187 of R.A. 7160 is fatal to their
cause.

As in Reyes, CEPALCOs failure to appeal to the Secretary of Justice


within the statutory period of 30 days from the effectivity of the
ordinance should have been fatal to its cause. However, we relax the
application of the rules in view of the more substantive matters.
City of Cagayan de Oros Power to Create Sources of Revenue vis-avis CEPALCOs Claim of Exemption
Section 5, Article X of the 1987 Constitution provides that "each local
government unit shall have the power to create its own sources of
revenues and to levy taxes, fees, and charges subject to such
guidelines and limitations as the Congress may provide, consistent
with the basic policy of local autonomy. Such taxes, fees, and
charges shall accrue exclusively to the local government." The Local
Government Code supplements the Constitution with Sections 151
and 186:
SEC. 151. Scope of Taxing Powers. Except as otherwise provided in
this Code, the city may levy the taxes, fees and charges which the
province or municipality may impose: Provided, however, That the
taxes, fees and charges levied and collected by highly urbanized and
independent component cities shall accrue to them and distributed
in accordance with the provisions of this Code.
The rates of taxes that the city may levy may exceed the maximum
rates allowed for the province or municipality by not more than fifty
percent (50%) except the rates of professional and amusement
taxes.
SEC. 186. Power to Levy Other Taxes, Fees or Charges. Local
government units may exercise the power to levy taxes, fees or
charges on any base or subject not otherwise specifically
enumerated herein or taxed under the provisions of the National
Internal Revenue Code, as amended, or other applicable laws:
Provided, That the taxes, fees, or charges shall not be unjust,
excessive, oppressive, confiscatory or contrary to declared national
policy: Provided, further, That the ordinance levying such taxes, fees,
or charges shall not be enacted without any prior public hearing
conducted for the purpose.
Although CEPALCO does not question the authority of the
Sangguniang Panlungsod of Cagayan de Oro to impose a tax or to
enact a revenue measure, CEPALCO insists that Ordinance No. 95032005 is an imposition of an income tax which is prohibited by Section
133(a)19 of the Local Government Code. Unfortunately for CEPALCO,
we agree with the ruling of the trial and appellate courts that
Ordinance No. 9503-2005 is a tax on business. CEPALCOs act of
leasing for a consideration the use of its posts, poles or towers to
other pole users falls under the Local Government Codes definition
of business. Business is defined by Section 131(d) of the Local
Government Code as "trade or commercial activity regularly
engaged in as a means of livelihood or with a view to profit." In
relation to Section 131(d),20 Section 143(h)21 of the Local
Government Code provides that the city may impose taxes, fees, and
charges on any business which is not specified in Section 143(a) to
(g)22 and which the sanggunian concerned may deem proper to tax.
In contrast to the express statutory provisions on the City of Cagayan
de Oros power to tax, CEPALCOs claim of tax exemption of the
income from its poles relies on a strained interpretation.23 Section 1
of R.A. No. 9284 added Section 9 to R.A. No. 3247, CEPALCOs
franchise:
SEC. 9. Tax Provisions. The grantee, its successors or assigns, shall
be subject to the payment of all taxes, duties, fees or charges and
other impositions applicable to private electric utilities under the
National Internal Revenue Code (NIRC) of 1997, as amended, the
Local Government Code and other applicable laws: Provided, That
nothing herein shall be construed as repealing any specific tax

exemptions, incentives, or privileges granted under any relevant law:5.


Provided, further, That all rights, privileges, benefits and exemptions
accorded to existing and future private electric utilities by their
respective franchises shall likewise be extended to the grantee.
The grantee shall file the return with the city or province where its
facility is located and pay the taxes due thereon to the
Commissioner of Internal Revenue or his duly authorized6.
representative in accordance with the NIRC and the return shall be
subject to audit by the Bureau of Internal Revenue.
The Local Government Code withdrew tax exemption privileges7.
previously given to natural or juridical persons, and granted local
government units the power to impose franchise tax,24 thus:
SEC. 137. Franchise Tax. Notwithstanding any exemption granted
by any law or other special law, the province may impose a tax on
businesses enjoying a franchise, at a rate not exceeding fifty percent8.
(50%) of one percent (1%) of the gross annual receipts for the
preceding calendar year based on the incoming receipt, or realized,
within its territorial jurisdiction.
SEC. 193. Withdrawal of Tax Exemption Privileges. Unless otherwise
provided in this Code, tax exemptions or incentives granted to, or
presently enjoyed by all persons, whether natural or juridical,
including government-owned or controlled corporations, except local
water districts, cooperatives duly registered under R.A. No. 6938,
non-stock and non-profit hospitals and educational institutions, are
hereby withdrawn upon the effectivity of this Code.
SEC. 534. Repealing Clause. x x x.
(f) All general and special laws, acts, city charters, decrees,
executive orders, proclamations and administrative regulations, or
part or parts thereof which are inconsistent with any of the
provisions of this Code are hereby repealed or modified accordingly. 9.
It is hornbook doctrine that tax exemptions are strictly construed
against the claimant. For this reason, tax exemptions must be based
on clear legal provisions. The separate opinion in PLDT v. City of
Davao25 is applicable to the present case, thus:
Tax exemptions must be clear and unequivocal. A taxpayer claiming
a tax exemption must point to a specific provision of law conferring
on the taxpayer, in clear and plain terms, exemption from a common
burden. Any doubt whether a tax exemption exists is resolved
against the taxpayer. Tax exemptions cannot arise by mere
implication, much less by an implied re-enactment of a repealed tax
exemption clause.
CEPALCOs claim of exemption under the "in lieu of all taxes" clause
must fail in light of Section 193 of the Local Government Code as
well as Section 9 of its own franchise.

Ordinance No. 9503-2005s Compliance with the Local Government


Code
In our Resolution dated 6 July 2011,26 we asked both parties to
discuss whether the amount of tax imposed by Section 2 of
Ordinance No. 9503-2005 complies with or violates, as the case may
be, the limitation set by Section 151, in relation to Sections 137 and
6.
143(h), of the Local Government Code.
CEPALCO argues that Ordinance No. 9503-2005 should be invalidated
because the City of Cagayan de Oro exceeded its authority in
enacting it. CEPALCO argued thus:

Thus, the taxes imposable under either Section 137 or Section


143(h) are not unbridled but are restricted as to the amount which
may be imposed. This is the first limitation. Furthermore, if it is a city
which imposes the same, it can impose only up to one-half of what
the province or municipality may impose. This is the second
limitation.
Let us now examine Ordinance No. 9503-2005 of the respondent City
of Cagayan de Oro in the light of the twin limitations mentioned
above.
Ordinance No. 9503-2005 of the respondent City of Cagayan de Oro
imposes a tax on the lease or rental of electric and/or
telecommunication posts, poles or towers by pole owners to other
pole users "at the rate of ten (10) percent of the annual rental
income derived therefrom."
With respect to Section 137, considering that the tax allowed
provinces "shall not exceed fifty percent (50%) of one percent (1%)
of the gross annual receipts for the preceding calendar year based
on the incoming receipt, or realized, within its territorial jurisdiction,"
the tax imposed by Ordinance No. 95032005 "at the rate of ten (10)
percent of the annual rental income derived therefrom" is too much.
There is a whale of a difference between the allowable 50% of 1%
and the 10% tax imposed by the respondent. To illustrate: assuming
that the gross annual receipt is Php100, the maximum tax that a
province may impose under Section 137 (50% of 1%) shall be Php0.5
or only fifty centavos. Therefore, the maximum tax that the City may
impose shall only be one-half of this, which is Php0.25 or only
twenty-five centavos. But the questioned Ordinance imposes a tax
amounting to 10% of the gross annual receipt of Php100, which is
Php10, or Ten Pesos. This a whooping [sic] 40 times more than that
allowed for the province! The violation made by respondent city of
its delegated taxing authority is all too patent.
With respect to Section 143(h), the rate of tax which the municipality
may impose "shall not exceed two percent (2%) of gross sales or
receipts of the preceding calendar year." On the other hand, the tax
imposed by Ordinance No. 9503-2005 is "at the rate of ten (10)
percent of the annual rental income derived therefrom." Again, it is
obvious that the respondent Citys questioned tax ordinance is way
too much. Using the same tax base of Php100 to illustrate, let us
compute:
Under Section 143(h), the maximum tax that a municipality may
impose is 2% of Php100, which is Php2 or Two Pesos. Therefore, the
maximum tax that the City may impose shall be one-half of this,
which is Php1 or One Peso. But the tax under Ordinance No. 95032005 is Php10, or Ten Pesos. This is a whooping [sic] 10 times more
than that allowed for the municipality! As in the earlier instance
discussed above, the violation made by the respondent city of its
delegated taxing authority is all too patent.27 (Boldfacing and
underscoring in the original)
The interpretation of the City of Cagayan de Oro is diametrically
opposed to that of CEPALCO. The City of Cagayan de Oro points out
that under Section 151 of the Local Government Code, cities not only
have the power to levy taxes, fees and charges which the provinces
or municipalities may impose, but the maximum rate of taxes
imposable by cities may exceed the maximum rate of taxes
imposable by provinces or municipalities by as much as 50%. The
City of Cagayan de Oro goes on to state:
Thus, Section 30 of City of Cagayan de Oros Ordinance No. 88472003, otherwise known as the Revenue Code of Cagayan de Oro,
imposes a franchise tax on the gross receipts realized from the
preceding year by a business enjoying a franchise, at the rate of
75% of 1%. The increase of 25% over that which is prescribed under
Section 137 of the LGC is in accordance with Section 151 thereof

prescribing the allowable increase on the rate of tax on the


businesses duly identified and enumerated under Section 143 of the
LGC or those defined and categorized in the preceding sections
thereof;
7.

8.

9.

Section 143 of the LGC prescribes the rate of taxes on the identified
categories of business enumerated therein which were determined
to be existing at the time of its enactment. On the other hand,
Section 151 of the LGC prescribes the allowable rate of increase over
the rate of taxes imposed on businesses identified under Section 143
and the preceding sections thereof. It is [City of Cagayan de Oros
humble opinion that the allowable rate of increase provided under
Section 151 of the LGC applies only to those businesses identified
and enumerated under Section 143 thereof. Thus, it is respectfully
submitted by City of Cagayan de Oro that the 2% limitation
prescribed under Section 143(h) applies only to the tax rates on the
businesses identified thereunder and does not apply to those that
may thereafter be deemed taxable under Section 186 of the LGC,
such as the herein assailed Ordinance No. 9503-2005. On the same
vein, it is the respectful submission of City of Cagayan de Oro that
the limitation under Section 151 of the LGC likewise does not apply
in our particular instance, otherwise it will run counter to the intent
and purpose of Section 186 of the LGC;
Be it strongly emphasized here that CEPALCO is differently situated
vis--vis the rest of the businesses identified under Section 143 of
the LGC. The imposition of a tax "xxx on the lease or rental of
electric and/or telecommunications posts, poles or towers by pole
owners to other pole users at the rate of ten (10%) of the annual
rental income derived therefrom" as provided under Section 2 of the
questioned Ordinance No. 9503-2005 is based on a reasonable
classification, to wit: (a) It is based on substantial distinctions which
make a real difference; (b) these are germane to the purpose of the
law; (c) the classification applies not only to the present conditions
but also to future conditions which are substantially identical to
those of the present; and (d) the classification applies only to those
belonging to the same class;
Furthermore, Section 186 of the LGC allow [sic] local government
units to exercise their taxing power to levy taxes, fees or charges on
any base or subject not otherwise specifically enumerated in the
preceding sections, more particularly Section 143 thereof, or under
the provisions of the National Internal Revenue Code, as long as they
are not unjust, excessive, oppressive, confiscatory or contrary to
declared national policy. Moreover, a public hearing is required
before the Ordinance levying such taxes, fees or charges can be
enacted;

10. It is respectfully submitted by City of Cagayan de Oro that the tax


rate imposed under Section 2 of the herein assailed Ordinance is not
unjust, excessive, oppressive, confiscatory or contrary to a declared
national policy;
11. A reading of Section 143 of the LGC reveals that it has neither
identified the operation of a business engaged in leasing nor
prescribed its tax rate. Moreover, a Lessor, in any manner, is not
included among those defined as Contractor under Section 131(h) of
the LGC. However, a Lessor, in its intended general application in
City of Cagayan de Oro (one who rents out real estate properties),
was identified, categorized and included as one of the existing
businesses operating in the city, and thus falling under the
provisions of Ordinance No. 8847-2003 (the Revenue Code of
Cagayan de Oro) and, therefore, imposed only a tax rate of 2% on
their gross annual receipts;
12. While the herein assailed Ordinance similarly identifies that the base
of the tax imposed therein are receipts and/or revenue derived from
rentals of poles and posts, CEPALCO cannot be considered under the
definition of Lessor under the spirit, essence and intent of Section

58(h) of the Revenue Code of Cagayan de Oro, because the same


refers only to "Real Estate Lessors, Real Estate Dealers and Real
Estate Developers." Thus, CEPALCO should be, as it has been,
categorized as a (Distinct) Lessor where it enjoys not only a
tremendous and substantial edge but also an absolute advantage in
the rental of poles, posts and/or towers to other telecommunication
and cable TV companies and the like over and above all others in
view of its apparent monopoly by allowing the use of their poles,
posts and/or towers by, leasing them out to, telecommunication and
cable TV companies operating within the city and suburbs.
Furthermore, CEPALCO has neither competition in this field nor does
it expect one since there are no other persons or entities who are
engaged in this particular business activity;
CEPALCO is mistaken when it states that a city can impose a tax up
to only one-half of what the province or city may impose. A more
circumspect reading of the Local Government Code could have
prevented this error. Section 151 of the Local Government Code
states that, subject to certain exceptions, a city may exceed by "not
more than 50%" the tax rates allowed to provinces and
municipalities.29 A province may impose a franchise tax at a rate
"not exceeding 50% of 1% of the gross annual receipts."30 Following
Section 151, a city may impose a franchise tax of up to 0.0075 (or
0.75%) of a business gross annual receipts for the preceding
calendar year based on the incoming receipt, or realized, within its
territorial jurisdiction. A municipality may impose a business tax at a
rate not exceeding "two percent of gross sales or receipts."31
Following Section 151, a city may impose a business tax of up to
0.03 (or 3%) of a business gross sales or receipts of the preceding
calendar year.
CEPALCO also erred when it equates Section 137s "gross annual
receipts" with Ordinance No. 95032005s "annual rental income."
Section 2 of Ordinance No. 9503-2005 imposes "a tax on the lease or
rental of electric and/or telecommunication posts, poles or towers by
pole owners to other pole users at the rate of ten (10) percent of the
annual rental income derived therefrom," and not on CEPALCOs
gross annual receipts. Thus, although the tax rate of 10% is
definitely higher than that imposable by cities as franchise or
business tax, the tax base of annual rental income of "electric and/or
telecommunication posts, poles or towers by pole owners to other
pole users" is definitely smaller than that used by cities in the
computation of franchise or business tax. In effect, Ordinance No.
9503-2005 wants a slice of a smaller pie.
However, we disagree with the City of Cagayan de Oros submission
that Ordinance No. 9503-2005 is not subject to the limits imposed by
Sections 143 and 151 of the Local Government Code. On the
contrary, Ordinance No. 9503-2005 is subject to the limitation set by
Section 143(h). Section 143 recognizes separate lines of business
and imposes different tax rates for different lines of business. Let us
suppose that one is a brewer of liquor and, at the same time, a
distributor of articles of commerce. The brewery business is subject
to the rates established in Section 143(a) while the distribution
business is subject to the rates established in Section 143(b). The
City of Cagayan de Oros imposition of a tax on the lease of poles
falls under Section 143(h), as the lease of poles is CEPALCOs
separate line of business which is not covered by paragraphs (a) to
(g) of Section 143. The treatment of the lease of poles as a separate
line of business is evident in Section 4(a) of Ordinance No. 95032005. The City of Cagayan de Oro required CEPALCO to apply for a
separate business permit.1wphi1
More importantly, because "any person, who in the course of trade or
business x x x leases goods or properties x x x shall be subject to the
value-added tax,"32 the imposable tax rate should not exceed two
percent of gross receipts of the lease of poles of the preceding
calendar year. Section 143(h) states that "on any business subject to
x x x value-added x x x tax under the National Internal Revenue

Code, as amended, the rate of tax shall not exceed two percent (2%)
of gross sales or receipts of the preceding calendar year" from the
lease of goods or properties. Hence, the 10% tax rate imposed by
Ordinance No. 9503-2005 clearly violates Section 143(h) of the Local
Government Code.
Finally, in view of the lack of a separability clause, we declare void
the entirety of Ordinance No. 95032005. Any payment made by
reason of the tax imposed by Ordinance No. 9503-2005 should,
therefore, be refunded to CEPALCO. Our ruling, however, is made
without prejudice to the enactment by the City of Cagayan de Oro of
a tax ordinance that complies with the limits set by the Local
Government Code.
WHEREFORE, we GRANT the petition. The Decision of the Court of
Appeals in CA-G.R. CV No. 01105-Min promulgated on 28 May 2009
and the Resolution promulgated on 24 March 2010 are REVERSED
and SET ASIDE Ordinance No. 9503-2005 is declared void.
SO ORDERED.

4.

Datu Michael Kida vs. Senate Gr no.

196271 October 18 2011


Same with case number 1 (ARTICLE X)

5. Legaspi vs Cebu City


ISSUE: Whether or not Ordinance No. 1664 is valid and constitutional.
HELD: The Court of Appeals decision is sustained.
CONSTITUTIONAL LAW - Tests for a valid ordinance
In City of Manila v. Laguio, Jr., G.R. No. 118127, April 12, 2005the
Court restates the tests of a valid ordinance thusly:

individuals not possessing legislative power. The National


Legislature, however, may delegate this power to the President and
administrative boards as well as the lawmaking bodies of municipal
corporations or local government units. Once delegated, the agents
can exercise only such legislative powers as are conferred on them
by the national lawmaking body. (emphasis supplied)
In the present case, delegated police power was exercised by the
LGU of the City of Cebu.
The CA opined, and correctly so, that vesting cities like the City of
Cebu with the legislative power to enact traffic rules and regulations
was expressly done through Section 458 of the LGC, and also
generally by virtue of the General Welfare Clause embodied in
Section 16 of the LGC.
The police power granted to local government units must always be
exercised with utmost observance of the rights of the people to due
process and equal protection of the law. Such power cannot be
exercised whimsically, arbitrarily or despotically as its exercise is
subject to a qualification, limitation or restriction demanded by the
respect and regard due to the prescription of the fundamental law,
particularly those forming part of the Bill of Rights. Individual rights,
it bears emphasis, may be adversely affected only to the extent that
may fairly be required by the legitimate demands of public interest
or public welfare. Due process requires the intrinsic validity of the
law in interfering with the rights of the person to his life, liberty and
property.
Judged according to the foregoing enunciation of the guaranty of due
process of law, the contentions of the petitioners cannot be
sustained. Even under strict scrutiny review, Ordinance No. 1664 met
the substantive tests of validity and constitutionality by its
conformity with the limitations under the Constitution and the
statutes, as well as with the requirements of fairness and reason, and
its consistency with public policy.
The subject of Ordinance No. 1664 is to ensure "a smooth flow of
vehicular traffic in all the streets in the City of Cebu at all times".
To reiterate, the clamping of the illegally parked vehicles was a fair
and reasonable way to enforce the ordinance against its
transgressors; otherwise, the transgressors would evade liability by
simply driving away.

The tests of a valid ordinance are well established. A long line of


decisions has held that for an ordinance to be valid, it must not only
be within the corporate powers of the local government unit to enact
and must be passed according to the procedure prescribed by law, it
must also conform to the following substantive requirements: (1)
must not contravene the Constitution or any statute; (2) must not be
unfair or oppressive;(3) must not be partial or discriminatory; (4)
must not prohibit but may regulate trade; (5) must be general and
consistent with public policy; and (6) must not be unreasonable.

DENIED.

As jurisprudence indicates, the tests are divided into the formal (i.e.,
whether the ordinance was enacted within the corporate powers of
the LGU, and whether it was passed in accordance with the
procedure prescribed by law), and the substantive (i.e., involving
inherent merit, like the conformity of the ordinance with the
limitations under the Constitution and the statutes, as well as with
the requirements of fairness and reason, and its consistency with
public policy).

2. Whether or not the issuance of the writ of execution was proper?

In Metropolitan Manila Development Authorityv. Bel-Air Village


Association,Inc., G.R. No. 135962, March 27, 2000the Court cogently
observed that police power is lodged primarily in the National
Legislature. It cannot be exercised by any group or body of

Province of aklan vs jody king


ISSUES:1. Whether or not the doctrine of primary jurisdiction is
applicable to this case?

HELD: Court of Appeals decision is reversed.


POLITICAL LAW: doctrine of primary jurisdiction
COA has primary jurisdiction over private respondents money claims.
Petitioner is not estopped from raising the issue of jurisdiction

The doctrine of primary jurisdiction holds that if a case is such that


its determination requires the expertise, specialized training and

knowledge of the proper administrative bodies, relief must first be


obtained in an administrative proceeding before a remedy is supplied
by the courts even if the matter may well be within their proper
jurisdiction.It applies where a claim is originally cognizable in the
courts, and comes into play whenever enforcement of the claim
requires the resolution of issues which, under a regulatory scheme,
have been placed within the special competence of an administrative
agency. In such a case, the court in which the claim is sought to be
enforced may suspend the judicial process pending referral of such
issues to the administrative body for its view or, if the parties would
not be unfairly disadvantaged, dismiss the case without prejudice.
The objective of the doctrine of primary jurisdiction is to guide the
court in determining whether it should refrain from exercising its
jurisdiction until after an administrative agency has determined
some question or some aspect of some question arising in the
proceeding before the court.
As can be gleaned, respondent seeks to enforce a claim for sums of
money allegedly owed by petitioner, a local government unit.
Under Commonwealth Act No. 327,as amended by Section 26 of
Presidential Decree No. 1445,it is the COA which has primary
jurisdiction over money claims against government agencies and
instrumentalities.
Section 26. General jurisdiction. The authority and powers of the
Commission shall extend to and comprehend all matters relating to
auditing procedures, systems and controls, the keeping of the
general accounts of the Government, the preservation of vouchers
pertaining thereto for a period of ten years, the examination and
inspection of the books, records, and papers relating to those
accounts; and the audit and settlement of the accounts of all persons
respecting funds or property received or held by them in an
accountable capacity, as well as the examination, audit, and
settlement of all debts and claims of any sort due from or owing to
the Government or any of its subdivisions, agencies and
instrumentalities. The said jurisdiction extends to all governmentowned or controlled corporations, including their subsidiaries, and
other self-governing boards, commissions, or agencies of the
Government, and as herein prescribed, including non-governmental
entities subsidized by the government, those funded by donations
through the government, those required to pay levies or government
share, and those for which the government has put up a counterpart
fund or those partly funded by the government.
Pursuant to its rule-making authority conferred by the 1987
Constitution and existing laws, the COA promulgated the 2009
Revised Rules of Procedure of the Commission on Audit. Rule II,
Section 1 specifically enumerated those matters falling under COAs
exclusive jurisdiction, which include "money claims due from or
owing to any government agency." Rule VIII, Section 1 further
provides: Section 1. Original Jurisdiction - The Commission Proper
shall have original jurisdiction over:

POLITICAL LAW: money claims against local government units are


under the jurisdiction of the COA
In Euro-Med Laboratories Phil., Inc. v. Province of Batangas, we ruled
that it is the COA and not the RTC which has primary jurisdiction to
pass upon petitioners money claim against respondent local
government unit. Such jurisdiction may not be waived by the parties
failure to argue the issue nor active participation in the proceedings.
Thus:
This case is one over which the doctrine of primary jurisdiction
clearly held sway for although petitioners collection suit
forP487,662.80 was within the jurisdiction of the RTC, the
circumstances surrounding petitioners claim brought it clearly within
the ambit of the COAs jurisdiction.
First, petitioner was seeking the enforcement of a claim for a certain
amount of money against a local government unit. This brought the
case within the COAs domain to pass upon money claims against the
government or any subdivision thereof under Section 26 of the
Government Auditing Code of the Philippines:
The authority and powers of the Commission on Audit shall extend to
and comprehend all matters relating to the examination, audit, and
settlement of all debts and claims of any sort due from or owing to
the Government or any of its subdivisions, agencies, and
instrumentalities.
The scope of the COAs authority to take cognizance of claims is
circumscribed, however, by an unbroken line of cases holding
statutes of similar import to mean only liquidated claims, or those
determined or readily determinable from vouchers, invoices, and
such other papers within reach of accounting officers. Petitioners
claim was for a fixed amount and although respondent took issue
with the accuracy of petitioners summation of its accountabilities,
the amount thereof was readily determinable from the receipts,
invoices and other documents. Thus, the claim was well within the
COAs jurisdiction under the Government Auditing Code of the
Philippines.
Second, petitioners money claim was founded on a series of
purchases for the medical supplies of respondents public hospitals.
Both parties agreed that these transactions were governed by the
Local Government Code provisions on supply and property
management and their implementing rules and regulations
promulgated by the COA pursuant to Section 383 of said Code.
Petitioners claim therefore involved compliance with applicable
auditing laws and rules on procurement. Such matters are not within
the usual area of knowledge, experience and expertise of most
judges but within the special competence of COA auditors and
accountants. Thus, it was but proper, out of fidelity to the doctrine of
primary jurisdiction, for the RTC to dismiss petitioners complaint.

a)

money claim against the Government;

b)

request for concurrence in the hiring of legal retainers by


government agency;

Respondents collection suit being directed against a local


government unit, such money claim should have been first brought
to the COA.Hence, the RTC should have suspended the proceedings
and refer the filing of the claim before the COA. Moreover, petitioner
is not estopped from raising the issue of jurisdiction even after the
denial of its notice of appeal and before the CA.

c)

write off of unliquidated cash advances and dormant accounts


receivable
in
amounts
exceeding
one
million
pesos
(P1,000,000.00);

POLITICAL LAW: exceptions to the doctrine of primary jurisdiction

d)

request for relief from accountability for loses due to acts of man,
i.e. theft, robbery, arson, etc, in amounts in excess of Five Million
pesos (P5,000,000.00).

There are established exceptions to the doctrine of primary


jurisdiction, such as:

(a) where there is estoppel on the part of the party invoking the
doctrine;
(b) where the challenged administrative act is patently illegal,
amounting to lack of jurisdiction;
(c) where there is unreasonable delay or official inaction that will
irretrievably prejudice the complainant;
(d) where the amount involved is relatively small so as to make the
rule impractical and oppressive;
(e) where the question involved is purely legal and will ultimately
have to be decided by the courts of justice;
(f)

where judicial intervention is urgent;

(g) when its application may cause great and irreparable damage;
(h) where the controverted acts violate due process;
(i)

when the issue of non-exhaustion of administrative remedies has


been rendered moot;

(j)

when there is no other plain, speedy and adequate remedy; (k)


when strong public interest is involved; and,

Finally, the RTC should have exercised utmost caution, prudence and
judiciousness in issuing the writ of execution and notices of
garnishment against petitioner. The RTC had no authority to direct
the immediate withdrawal of any portion of the garnished funds from
petitioner's depositary banks.Such act violated the express directives
of this Court under Administrative Circular No. 10-2000,which was
issued "precisely in order to prevent the circumvention of
Presidential Decree No. 1445, as well as of the rules and procedures
of the COA."

Province of cagayan vs lara


In order for an entity to legally undertake a quarrying business, he
must first comply with all the requirements imposed not only by the
national government, but also by the local government unit where
his business is situated. Particularly, Section 138(2) of RA 7160 26
requires that such entity must first secure a governors permit prior
to the start of his quarrying operations, viz:
SECTION 138. Tax on Sand, Gravel and Other Quarry Resources.

(l) in quo warranto proceedings.However, none of the foregoing


circumstances is applicable in the present case.

The doctrine of primary jurisdiction does not warrant a court to


arrogate unto itself authority to resolve a controversy the jurisdiction
over which is initially lodged with an administrative body of special
competence.All the proceedings of the court in violation of the
doctrine and all orders and decisions rendered thereby are null and
void.
POLITICAL LAW: writ of execution that violates primary jurisdiction is
void
Writ of Execution issued in violation of COAs primary jurisdiction is
void
Since a judgment rendered by a body or tribunal that has no
jurisdiction over the subject matter of the case is no judgment at all,
it cannot be the source of any right or the creator of any
obligation.All acts pursuant to it and all claims emanating from it
have no legal effect and the void judgment can never be final and
any writ of execution based on it is likewise void.
Clearly, the CA erred in ruling that the RTC committed no grave
abuse of discretion when it ordered the execution of its judgment
against petitioner and garnishment of the latters funds.
In its Supplement to the Motion for Reconsideration, petitioner
argued that it is the COA and not the RTC which has original
jurisdiction over money claim against government agencies and
subdivisions.The
CA,
in
denying
petitioner's
motion
for
reconsideration, simply stated that the issue had become moot by
respondent's filing of the proper petition with the COA. However,
respondent's belated compliance with the formal requirements of
presenting its money claim before the COA did not cure the serious
errors committed by the RTC in implementing its void decision. The
RTC's orders implementing its judgment rendered without jurisdiction
must be set aside because a void judgment can never be validly
executed.

x x x. The permit to extract sand, gravel and other quarry resources


shall be issued exclusively by the provincial governor, pursuant to
the ordinance of the sangguniang panlalawigan. (Emphasis and
underscoring supplied)x x x x

Government of Camarines Norte


vs Gonzales
ISSUE/S: WON Gonzales has security of tenure over her position
as provincial administrator of Camarines Norte.
HELD: NO. Decision of the CA reversed and set aside.
RATIO: A. The Court supported the CAs conclusion that the
position of provincial administrator has been re-classified into a
primarily confidential, non-career position upon the passage of
RA 7160, or the Local Government Code (LGC) which took
effect in January 1992.
In making the position mandatory for
all provinces, the LGC also amended the qualifications for the
position. Further to this, the LGC made the provincial
administrator position co-terminous with its appointing authority,
reclassifying it as a
non-career service position that is
primarily confidential. Upon this, the Court took note of the
argument that Gonzales has acquired a vested legal right over
the position of provincial administrator the moment she
assumed her duties in April 1991, hence the argument that she
cannot be removed from office except for cause and after due
hearing.
According to the SC, the arguments reflect a conceptual
confusion between the nature of the position and an employees
right to hold a position. The nature of a position may change
by law according to the dictates of Congress. The right to hold
a position, on the other hand, is a right that enjoys
constitutional and statutory guarantee, but may itself change
according to the nature of the position. Congress has the power
and prerogative to introduce substantial changes in the
provincial administrator position and to reclassify it as a
primarily confidential, non-career service position. When done in

good faith, these acts would not violate a public officers


security of tenure, even if they result in his removal from
office or the shortening of his term. Modifications in public
office, such as changes in qualifications or shortening of its
tenure, are made in good faith so long as they are aimed at
the office and not at the incumbent.
B.

The Court also pointed out that Gonzales reliance on the case
of Gabriel v. Domingos dissenting opinion (which stated that a
permanent employee remains a permanent employee unless he
is validly terminated) was misplaced. First of all, the factual
differences were pointed out to be dissimilar to the case of
Gonzales, and even granting that they were the same, the
cited case (in Gabriel) of Civil Service Commission v. Javier
actually proposes that corporate secretaries in GOCCs cannot
expect protection for their tenure and appointments upon the
reclassification of their position to a primarily confidential
position. These officers cannot rely on the statutes providing for
their permanent appointments, if and when the Court
determines these to be primarily confidential.
Further to this, said dissenting opinion in Gabriel cited EO 503,
which provided safeguards against termination of government
employees affected by RA 7160s implementation. According to
the dissenting opinion, EO 503 is an obvious indication of the
executive departments intent to protect and uphold both the
national government and the local government employees
security of tenure. However, the Court emphasized that EO
503, however, does not apply to employees of the local
government affected by RA 7160s enactment, as it only applies
to National Government Agencies whose functions are to be
devolved to LGUs.

C.

Finally, the Court noted that both career and non-career service
employees have a right to security of tenure. All permanent
officers and employees in the civil service, regardless of
whether they belong to the career or non-career service
category, are entitled to this guaranty; they cannot be removed
from office except for cause provided by law and after
procedural due process. The concept of security of tenure,
however,
operates
under
a
different rule
for
primarily
confidential employees due to the nature of a primarily
confidential position. Serving at the confidence of the
appointing authority, the primarily confidential employees term
of office expires when the appointing authority loses trust in
the employee. When this happens, the confidential employee is
not removed or dismissed from office. The term merely
expires and the loss of trust and confidence is the just
cause provided by law that results in the termination of
employment. In the case of Gonzales, where the trust and
confidence has been irretrievably eroded, Gov. Pimentel only
exercised his discretion when he decided that he could no
longer entrust his confidence in Gonzales.
Security of tenure in public office simply means that a public
officer or employee shall not be suspended or dismissed except
for cause, as provided by law and after due process. It cannot
be expanded to grant a right to public office despite a change
in the nature of the office held. The CSC might have been
legally correct when it ruled that the petitioner violated
Gonzales right to security of tenure when she was removed
without sufficient just cause from her position, but the situation
had since then been changed. In fact, Gonzales was reinstated
as ordered, but her services were subsequently terminated
under the law prevailing at the time of the termination of her
service. She was then already occupying a position that was
primarily confidential and had to be dismissed because she no

longer enjoyed the trust and confidence of the appointing


authority. Thus, Gonzales termination for lack of confidence
was lawful. She could no longer be reinstated as provincial
administrator of Camarines Norte or to any other comparable
position. This, however, is without prejudice to Gonzales
entitlement to retirement benefits, leave credits, and future
employment in government service.

Rozul vs Sandiganbayan
While the DENR is, indeed, the primary government instrumentality
charged with the mandate of promulgating rules and regulations for
the protection of the environment and conservation of natural
resources, it is not the only government instrumentality clothed with
such authority. While the law has designated DENR as the primary
agency tasked to protect the environment, it was not the intention of
the law to arrogate unto the DENR the exclusive prerogative of
exercising this function. Whether in ordinary or in legal parlance, the
word "primary" can never be taken to be synonymous with "sole" or
"exclusive." In fact, neither the pertinent provisions of PD 705 nor EO
192 suggest that the DENR, or any of its bureaus, shall exercise such
authority to the exclusion of all other government instrumentalities,
i.e., LGUs.
As a final note, We emphasize that the burden of protecting the
environment is placed not on the shoulders of DENR aloneeach and
every one of us, whether in an official or private capacity, has his or
her significant role to play. Indeed, protecting the environment is not
only a responsibility but also a right for which a citizen could and
should freely exercise. Considering the rampant forest denudation,
environmental degradation and plaguing scarcity of natural
resources, each of us is now obligated to contribute and share in the
responsibility of protecting and conserving our treasured natural
resources.
Ruzol chose to exercise this right and to share in this responsibility by
exercising his authority as municipal mayoran act which was
executed with the concurrence and cooperation of nongovernmental
organizations, industry stakeholders, and the concerned citizens of
General Nakar. Admittedly, We consider his acts as invalid but it does
necessarily mean that such mistakes automatically demand Us to
rule a conviction. This is in consonance with the settled principle that
"all reasonable doubt intended to demonstrate error and not crime
should be indulged in for the benefit of the accused."
Under our criminal judicial system, "evil intent must unite with the
unlawful act for a crime to exist," as "there can be no crime when
the criminal mind is wanting." 50 Actus non facit reum, nisi mens sit
rea.
In the present case, the prosecution has failed to prove beyond
reasonable doubt that Ruzol possessed that "criminal mind" when he
issued the subject permits. What is clear from the records is that
Ruzol, as municipal mayor, intended to regulate and monitor
salvaged forest products within General Nakar in order to avert the
occurrence of illegal logging in the area. We find that to hold him
criminally liable for these seemingly noble intentions would be a step
backward and would run contrary to the standing advocacy of
encouraging people to take a pro-active stance in the protection of
the environment and conservation of our natural resources.

PELIZLOY REALTY CORPORATION(2)


v. THE PROVINCE OF BENGUET
Local taxation; amusement tax; excludes resorts, swimming pools,
bath houses, hot springs and tourist spots. Indeed, theaters,
cinemas, concert halls, circuses, and boxing stadia are bound by a
common typifying characteristic in that they are all venues primarily
for the staging of spectacles or the holding of public shows,
exhibitions, performances, and other events meant to be viewed by
an audience. Accordingly, other places of amusement must be
interpreted in light of the typifying characteristic of being venues
where one seeks admission to entertain oneself by seeing or
viewing the show or performances or being venues primarily used
to stage spectacles or hold public shows, exhibitions, performances,
and other events meant to be viewed by an audience.
As dened in The New Oxford American Dictionary, show means a
spectacle or display of something, typically an impressive one;
while performance means an act of staging or presenting a play, a
concert, or other form of entertainment. As such, the ordinary
denitions of the words show and performance denote not only
visual engagement (i.e., the seeing or viewing of things) but also
active doing (e.g., displaying, staging or presenting) such that
actions are manifested to, and (correspondingly) perceived by an
audience.
Considering these, it is clear that resorts, swimming pools, bath
houses, hot springs and tourist spots cannot be considered venues
primarily where one seeks admission to entertain oneself by seeing
or viewing the show or performances. While it is true that they may
be venues where people are visually engaged, they are not primarily
venues for their proprietors or operators to actively display, stage or
present shows and/or performances.
Thus, resorts, swimming pools, bath houses, hot springs and tourist
spots do not belong to the same category or class as theaters,
cinemas, concert halls, circuses, and boxing stadia. It follows that
they cannot be considered as among the other places of
amusement contemplated by Section 140 of the LGC and which may
properly be subject to amusement taxes.

Fernando
college

vs

st.

Scholastica

Issue: Is Marikina Ordinance No. 192, imposing a five-meter


setback, a valid exercise of police power?
Ruling: No. Police power is the plenary power vested in the
legislature to make statutes and ordinances to promote the
health, morals, peace, education, good order or safety and
general welfare of the people. Two tests have been used by
the Court the rational relationship test and the strict scrutiny
test:
Under the rational relationship test, an ordinance must pass
the following requisites:
(1) the interests of the public generally, as distinguished
those of a particular class, require its exercise; and

from

the means employed are reasonably necessary for the


accomplishment of the purpose and not unduly oppressive upon
individuals.
The real intent of the setback requirement was to make the
parking space free for use by the public and not for the
exclusive use of respondents. This would be tantamount to a
taking of private property for public use without just
compensation.
Anent
the
objectives
of
prevention
of
concealment of unlawful acts and un-neighborliness due to the
walls and fences, the parking area is not reasonably necessary
for the accomplishment of these goals. The Court, thus, finds
Section 5 of the Ordinance to be unreasonable and oppressive.
Hence, the exercise of police power is not valid.

16. SOCRATES VS COMELEC


Issue: WON one who has been elected and served for 3 consecutive
full terms is qualified to run for mayor in the recall election.
Held: Yes. The three-term limit rule for elective local officials is found
in Section 8, Article X of the Constitution, which states:
Section 8. The term of office of elective local officials, except
barangay officials, which shall be determined by law, shall be three
years and no such official shall serve for more than three
consecutive terms. Voluntary renunciation of the office for any
length of time shall not be considered as an interruption in the
continuity of his service for the full term for which he was elected.
This three-term limit rule is reiterated in Section 43 (b) of RA No.
7160, otherwise known as the Local Government Code, which
provides:
Section 43.Term of Office. (a) x xx
(b) No local elective official shall serve for more than three (3)
consecutive terms in the same position. Voluntary renunciation of
the office for any length of time shall not be considered as an
interruption in the continuity of service for the full term for which the
elective official was elected.
The first part provides that an elective local official cannot serve
for more than three consecutive terms. The clear intent is that only
consecutive terms count in determining the three-term limit rule.
The second part states that voluntary renunciation of office for any
length of time does not interrupt the continuity of service. The clear
intent is that involuntary severance from office for any length of time
interrupts continuity of service and prevents the service before and
after the interruption from being joined together to form a
continuous service or consecutive terms.
After three consecutive terms, an elective local official cannot seek
immediate re-election for a fourth term. The prohibited election
refers to the next regular election for the same office following the
end of the third consecutive term. Any subsequent election, like a
recall election, is no longer covered by the prohibition for two
reasons. First, a subsequent election like a recall election is no
longer an immediate re-election after three consecutive terms.
Second, the intervening period constitutes an involuntary
interruption in the continuity of service.
Based from the deliberations of a Constitutional Commission, what
the Constitution prohibits is an immediate re-election for a fourth
term following three consecutive terms. The Constitution, however,
does not prohibit a subsequent re-election for a fourth term as long

as the re-election is not immediately after the end of the third


consecutive term. A recall election mid-way in the term following the
third consecutive term is a subsequent election but not an
immediate re-election after the third term.
Neither does the Constitution prohibit one barred from seeking
immediate re-election to run in any other subsequent election
involving the same term of office. What the Constitution prohibits is
a consecutive fourth term.
In the case of Hagedorn, his candidacy in the recall election on
September 24, 2002 is not an immediate re-election after his third
consecutive term which ended on June 30, 2001. The immediate reelection that the Constitution barred Hagedorn from seeking referred
to the regular elections in 2001.

17. LATASA VS COMELEC


whether or not petitioner Latasa is eligible to run as candidate for
the position of mayor of the newlycreated City of Digos immediately
after he served for three consecutive terms as mayor of the
Municipality of Digos?
NO.
As seen in the aforementioned provisions, this Court notes that the
delineation of the metes and bounds of the City of Digos did not
change even by an inch the land area previously covered by the
Municipality of Digos. This Court also notes that the elective officials
of the Municipality of Digos continued to exercise their powers and
functions until elections were held for the new city officials.
True, the new city acquired a new corporate existence separate and
distinct from that of the municipality. This does not mean, however,
that for the purpose of applying the subject Constitutional provision,
the office of the municipal mayor would now be construed as a
different local government post as that of the office of the city
mayor. As stated earlier, the territorial jurisdiction of the City of
Digos is the same as that of the municipality. Consequently, the
inhabitants of the municipality are the same as those in the city.
These inhabitants are the same group of voters who elected
petitioner Latasa to be their municipal mayor for three consecutive
terms. These are also the same inhabitants over whom he held
power and authority as their chief executive for nine years.
This Court reiterates that the framers of the Constitution specifically
included an exception to the peoples freedom to choose those who
will govern them in order to avoid the evil of a single person
accumulating excessive power over a particular territorial jurisdiction
as a result of a prolonged stay in the same office. To allow petitioner
Latasa to vie for the position of city mayor after having served for
three consecutive terms as a municipal mayor would obviously
defeat the very intent of the framers when they wrote this exception.
Should he be allowed another three consecutive terms as mayor of
the City of Digos, petitioner would then be possibly holding office as
chief executive over the same territorial jurisdiction and inhabitants
for a total of eighteen consecutive years. This is the very scenario
sought to be avoided by the Constitution, if not abhorred by it.

LONZANIDA VS COMELEC

Petitioner was elected and served two consecutive terms as mayor


from 1988 to 1995. He then ran again for the same position in the
May 1995 elections, won and discharged his duties as mayor.
However, his opponent contested his proclamation and filed an
election protest before the Regional Trial Court, which ruled that
there was a failure of elections and declared the position of mayor
vacant.
The COMELEC affirmed this ruling and petitioner acceded to the
order to vacate the post. During the May 1998 elections, petitioner
therein again filed his certificate of candidacy for mayor. A petition to
disqualify him was filed on the ground that he had already served
three consecutive terms.
This Court ruled, however, that petitioner therein cannot be
considered as having been duly elected to the post in the May 1995
elections, and that said petitioner did not fully serve the 1995-1998
mayoral term by reason of involuntary relinquishment of office.
2 REQUISITEs AS TO THREE-TERM LIMIT RULE
1.
2.

The official concerned has been elected for three


consecutive terms in the same local government post; and
That he has fully served three consecutive terms.

Note: Voluntary renunciation of the office for any length of time shall
not be considered as an interruption in the continuity of service.

18. BORJA VS COMELEC


Issue: Whether a vice-mayor who became the mayor by
operation of law and who served the remainder of the mayors term
should be considered to have served a term in that office for the
purpose of the three-term limit under the Constitution?
No. When private respondent occupied the post of the
mayor upon the incumbents death and served for the remainder of
the term, he cannot be construed as having served a full term
as contemplated under the subject constitutional provision.
The term served must be one for which [the official concerned] was
elected.
The private respondent (Capco) therein, before he assumed
the position of mayor, first served as the vice-mayor of his local
government unit. The nature of the responsibilities and duties of the
vicemayor is wholly different from that of the mayor. The vice-mayor
does not hold office as chief executive over his local government
unit.
The term limit for elective local officials must be taken to
refer to the right to be elected as well as the right to serve the same
elective position. Consequently, it is not enough that an individual
has served three consecutive terms in an elective local office, he
must also have been elected to the same position for the same
number of times before the disqualification can apply. Capco was
qualified to run again as mayor in the next election because he was
not elected to the office of mayor in the first term but simply found
himself thrust into it by operation of law. Neither had he served the
full term because he only continued the service, interrupted by the
death, of the deceased mayor. The vice-mayors assumption of the
mayorship in the event of the vacancy is more a matter of chance
than of design. Hence, his service in that office should not be
counted in the application of any term limit.

The policy embodied in the constitutional provision (Art. X,


8) is not only to prevent the establishment of political dynasties but
also to enhance the freedom of choice of the people. A consideration
of the historical background of Art. X, 8 of the Constitution reveals
that the members of the Constitutional Commission were as much
concerned with preserving the freedom of choice of the people as
they were with preventing the monopolization of political power. In
discussing term limits, the drafters of the Constitution did so on the
assumption that the officials concerned were serving by reason of
election. To consider Capco to have served the first term in full and
therefore ineligible to run a third time for reelection would be not
only to falsify reality but also to unduly restrict the right of the
people to choose whom they wish to govern them.

19. ADORMEO vs COMELEC

limit rule of Art. X, sec. 8 of the Constitution since his 2004-2007


term was not interrupted by the preventive suspension imposed on
him, the SC granted the petition of Simon B. Aldovino, Danilo B.
Faller, and Ferdinand N. Talabong seeking Asilos disqualification.
Preventive suspension, by its nature, does not involve an effective
interruption of service within a term and should therefore not be a
reason to avoid the three-term limitation, held the Court. It noted
that preventive suspension can pose as a threat more potent than
the voluntary renunciation that the Constitution itself disallows to
evade the three-term limit as it is easier to undertake and merely
requires an easily fabricated administrative charge that can be
dismissed soon after a preventive suspension has been imposed.

21.

ABUNDO VS COMELEC

Whether or not an assumption to office through a recall


election should be considered as one term in applying the three-term
limit rule?

ISSUE: Whether or not Abundo has consecutively served for three


terms.

NO.

HELD: The petition is partly meritorious.

The term limit for elective local officials must be taken to refer to the
right to be elected as well as the right to serve in the same elective
position. It is not enough that an individual has served three
consecutive terms in an elective local office, he must also have been
elected to the same position for the same number of times before
the disqualification can apply.

CONSTITUTIONAL LAW: Involuntary Interruption of Service

The two conditions for the application of the disqualification


must concur:
1)

that the official concerned has been elected for three consecutive
terms in the same local government post; and

2)

that he has fully served three consecutive terms. COMELECs ruling


that private respondent was NOT elected for three consecutive
terms should be upheld. The continuity of his mayorship was
disrupted by his defeat in the 1998 elections.
Voluntary renunciation of office for any length of time shall not
be considered as an interruption in the continuity of service for the
full term for which he was elected. Voluntary renunciation of a term
does not cancel the renounced term in the computation of the three
term limit; conversely, involuntary severance from office for any
length of time short of the full term provided by law amounts to an
interruption of continuity of service.

The consecutiveness of what otherwise would have been Abundos


three successive, continuous mayorship was effectively broken
during the 2004- 2007 term when he was initially deprived of title to,
and was veritably disallowed to serve and occupy, an office to which
he, after due proceedings, was eventually declared to have been the
rightful choice of the electorate.
The declaration of being the winner in an election protest grants the
local elected official the right to serve the unexpired portion of the
term. Verily, while he was declared winner in the protest for the
mayoralty seat for the 2004-2007 term, Abundos full term has been
substantially reduced by the actual service rendered by his opponent
(Torres). Hence, there was actual involuntary interruption in the term
of Abundo and he cannot be considered to have served the full 20042007 term.
Prior to the finality of the election protest, Abundo did not serve in
the mayors office and, in fact, had no legal right to said position.
During the pendency of the election protest, Abundo ceased from
exercising power or authority. Consequently, the period during which
Abundo was not serving as mayor should be considered as a rest
period or break in his service because prior to the judgment in the
election protest, it was Abundos opponent, Torres, who was
exercising such powers by virtue of the still then valid proclamation.
Petition is PARTLY GRANTED.

20. ALDOVINO VS COMELEC


ISSUE:
WON the suspensive condition interrupts the three-term limitation
rule of COMELEC?

ADDITIONAL NOTES/REVIEW:

NO. The preventive suspension of public officials does not interrupt


their term for purposes of the threeterm limit rule under the
Constitution and the Local Government Code (RA 7160).

On the issue of recall elections, Adormeo v. Commission on


Elections and Socrates v. Commission on Elections held that an
elective official, who has served for three consecutive terms and who
did not seek the elective position for what could be his fourth term,
but later won in a recall election, had an interruption in the
continuity of the officials servicefor he had become in the interim a
private citizen.

The candidacy of Lucena City Councilor Wilfredo F. Asilo for a fourth


term in the 2007 elections was in contravention of the three-term

Latasa v. Commission on Elections ruled that the abolition of an


elective office due to the conversion of a municipality to a city does

RULING:

not, by itself, work to interrupt the incumbent officials continuity of


service.
As mentioned above, Aldovino, Jr. v. Commission on Elections
states that preventive suspension is not a term interrupting event as
the elective officers continued stay and entitlement to the office
remain unaffected during the period of suspension, although he is
barred from exercising the functions of the office during this period.
Lonzanida v. Commission on Elections and Dizon v. Commission
on Elections continued on to rule that when a candidate is
proclaimed as winner for an elective position and assumes office, his
term is interrupted when he loses in an election protest and is ousted
from office. An interruption for any length of time, provided the
cause is involuntary is sufficient to break the continuity of service.
Lastly, Ong v. Alegre and Rivera III v. Commission on Elections
declared when an official is defeated in an election protest and

decision becomes final only after the official had served the full term
for the office, then his loss in the election contest does not constitute
an interruption since he has managed to serve the term from start to
finish. His full service should be counted in the application of term
limits because the nullification of his proclamation came after the
expiration of the term. (GR No. 201716, Abundo v. Commission
on Elections, January 8, 2013)

You might also like